You are on page 1of 92

JEE (MAIN & ADV.

), MEDICAL + BOARD, NDA, X & IX


Enjoys unparalleled reputation for best results
in terms of percentage selection
www.newtonclasses.net

TRIGONOMETRIC FUNCTION ( )
Only one option is correct.
1. One radian ≈
(a) 57°17 ' 44.8" (b) 58°37 ' 22.4" (c) 59°27 '33.6" (d) 56°57 '11.2"
3
2. The radius of the circle whose arc of length 15 cm makes an angle of radian at the centre, is
4
1 1
(a) 10 cm (b) 11 cm (c) 20 cm (d) 22 cm
4 2
3. The length of an are of a circle of radius 28 cm, that subtends an angle of 45° at the centre, is
(a) 12 cm (b) 16 cm (c) 22 cm (d) 24 cm
4. The angle subtended at the centre of a circle of diameter 50 cm by an arc of length 11 cm is
(a) 20°15' (b) 22°48 ' (c) 25°12 ' (d) 27°24 '
5. The acute angle in radians between the minute and the hour hand of a clock when the time is 4 : 20 is
π π π π
(a) (b) (c) (d)
18 9 6 3
π
6. The angles of a triangle are in A.P. , the smallest angle being , the largest angle is
4
π 2π 4π 5π
(a) (b) ( c) (d)
3 3 9 12
7. A circular wire of radius 15 cm is cut and bent so as to lie along the circumference of a loop of radius
120 cm. The angle subtended by it at the centre is
(a) 30° (b) 45° (c) 60° (d) None of these
8. If the interior angles of a polygon are in A.P. with common difference 5° and the smallest angle 120° ,
then the number of sides of the polygon is
(a) 7 (b) 9 (c) 7 or 15 (d) 9 or 16
9. A horse is tied to a post by a rope. If it moves along a circular path, keeping the rope tight and
describes 132 m when it has traced out an angle of 108° at the centre, then the length of the rope is
(a) 54 m (b) 66 m (c) 70 m (d) 81 m
Find the measures of ∠C of ∆ABC in degrees and radians, if m∠A = 42° and m∠B = (π / 6 )
c
10.
c c c c
 3π  π   3π   5π 
(a)   (b)   (c)   (d)  
 5  5  2   3 
11. A semicircle is divided into two sectors whose angles are in the ratio 4 : 5. Find the ratio of their areas
(a) 5 :1 (b) 4 : 5 (c) 5 : 4 (d) 3 : 4
12. Two concentric circles with the centre O have radii 18 cm and 36 cm. A line OX cuts the circles in
A and A′ respectively. This line is rotated through an angle of 80° and, in its final position, it cuts the
circles in B and B′ respectively. Find the perimeter and the area of the region between the circles and
lines
(a) 70π cm 2 (b) 288π cm 2 (c) 216π cm 2 (d) 210π cm 2
13. Find the measure of the angle between the hour-hand and the minute-hand of a clock at twenty minutes
past two.

th
Office.: 606 , 6 Floor, Hariom Tower, Circular Road, Ranchi-1, Ph.: 0651-2562523, 9835508812, 8507613968
2 ( BY R. K. MALIK’S NEWTON CLASSES
)
(a) 50° (b) 60° (c) 54° (d) 65°
14. The angle between the hands of a clock when the time is 3.20 p.m. is
π π π π
(a) (b) (c) (d)
18 9 6 8
15. If the angles of a triangle are in the ratio 1: 2 : 3 , then the smallest angle in radian is
π π π π
(a) (b) (c) (d)
3 6 2 9
16. A circular wire of radius 5 cm is cut and bent again into an arc of a circle of radius 10 cm. The angle
subtended by the arc at the centre in radian is
π π π
(a) (b) (c) (d) π
2 3 6
17. A circular wire of diameter 40 cm is cut and placed along the circumference of a circle of radius 1
metre, the angle subtended by the wire at the centre of the circle is equal to
π π 2π π
(a) (b) (c) (d)
4 3 5 6
18. The central angle of a sector of circle of area 9 π sq. cm is 60° , the perimeter of the sector is
(a) π (b) 3 + π (c) 6 + π (d) 6
19. The angles of a triangle are in A.P., and the greatest angle is double the least then the angles in degrees
are
(a) 30°, 60°, 90° (b) 40°, 60°, 80° (c) 20°, 40°, 80° (d) 30°, 60°, 80°
20. A wheel makes 120 revolutions in one minute. Then the angle in radian covered by the wheel in 2
seconds is
(a) 4π (b) 6π (c) 8π (d) 2π
21. The ratio of the number of sides of two regular polygons is 3 : 4 and the difference between their
angles is 15°. The number of sides of the polygon having larger angle is
(a) 6 (b) 8 (c) 10 (d) 12
cos A cos B 1 π π
22. If = = , − < A < 0, − < B < 0, then the value of 2 sin A + 4 sin B is equal to
3 4 5 2 2
(a) −4 (b) 0 (c) 2 (d) 4
p sin θ + cos θ
23. If tan θ = , what is the value of ?
q sin θ − cos θ
p+q p−q p q
(a) 1 (b) (c) (d) +
p−q p+q q p
cos 9° + sin 9°
24. is equal to
cos 9° − sin 9°
(a) tan 26° (b) tan 36° (c) tan 54° (d) tan 81°
25. If cos θ − sin θ = 1and 0 < θ ≤ π then θ is equal to
π π
(a) (b) (c) π (d) None
6 3
26. The value of cos1° ⋅ cos 2°.....cos100° is

th
Office.: 606 , 6 Floor, Hariom Tower, Circular Road, Ranchi-1, Ph.: 0651-2562523, 9835508812, 8507613968
( ) BY R. K. MALIK’S NEWTON CLASSES 3
(a) −1 (b) 0 (c) 1 (d) None of these
27. If p = cos 2 θ + sec2 θ , then
(a) p < 1 (b) 0 < p ≤ 1 (c) 1 ≤ p ≤ 2 (d) p ≥ 2
28. The value of cos ( 270° + θ ) cos ( 90° − θ ) − sin ( 270° − θ ) cos θ is
1
(a) −1 (b) 0 (c) (d) 1
2
29. The value of the expression cos1°.cos 2°..cos179° equals
1
(a) −1 (b) 0 (c) (d) 1
2
tan A 1 + sec A
30. + is equal to
1 + sec A tan A
(a) 2 cosec A (b) 2 sec A (c) 2 sin A (d) 2 cos A
4 4
31. cos θ − sin θ is equal to
θ  θ 
(a) 1 − 2sin 2   (b) 1 + 2 sin 2   (c) 2 cos 2 θ − 1 (d) 1 + 2 cos 2 θ
2 2
 π   5π 
32. Evaluate 2 sin   sin  
 12   12 
1 3 1
(a) − (b) − (c) 1 (d)
4 2 2
π
33. If α is a root of 25 cos 2 θ + 5cos θ − 12 = 0, < α < π , then sin 2α is equal to :
2
24 13 13 24
(a) − (b) − (c) (d)
25 18 18 25
1
34. If sin ( x − y ) = cos ( x + y ) = , then values of x and y ( Given, x, y ∈ [ 0, 180°]) are given by
2
(a) x = 180°, y = 135° (b) x = 165°, y = 15° (c) x = 45°, y = 135° (d) x = 45°, y = 15°

 1 − sin θ   1 + sin θ 
35. If θ lies in the second quadrant, then the value of  +   is equal to
 1 + sin θ   1 − sin θ 
(a) 2sec θ (b) −2sec θ (c) 2 cos ec θ (d) −2 cos ec θ
1 + tan 2 θ
36. If θ = 60°, then is equal to
2 tan θ
3 2 1
(a) (b) (c) (d) 3
2 3 3
1  1 
37. If sec θ = m and tan θ = n, then ( m + n ) +  is equal to
m  ( m + n ) 
(a) 2 (b) mn (c) 2m (d) 2n
1 + sin θ − cos θ   π 
38. The expression is equal to  where A ∈  0,  
1 + sin θ + cos θ   2 
θ θ θ θ θ
(a) 1 − sin cos (b) 1 + sin cos (c) tan (d) 2(cot 2 θ − 1) .
2 2 2 2 2
39. If A =`130° and x = sin A + cos A, then
th
Office.: 606 , 6 Floor, Hariom Tower, Circular Road, Ranchi-1, Ph.: 0651-2562523, 9835508812, 8507613968
4 ( ) BY R. K. MALIK’S NEWTON CLASSES
(a) x < 0 (b) x = 0 (c) x > 0 (d) x ≥ 0
π 3π 3 3 3 tan α + 2 tan β
40. If <α < π, π < β < and cos α = − , sin β = − then is equal to
2 2 2 5 cot 2 α + cos β
3 5 4 7
(a) (b) (c) (d)
20 22 25 27
1 cosec 2θ − sec2 θ
41. If θ is an acute angle and tan θ = , then the value of is
7 cosec2θ + sec 2 θ
1 3 5
(a) (b) (c) (d) 2
2 4 4
42. If A lies in the second quadrant and 3 tan A + 4 = 0, then the value of 2 cot A − 5cos A + sin A is equal
to
53 7 17 23
(a) − (b) − (c) (d)
10 10 10 10
x 1
43. If tan α = and tan β = , then α + β is where α and β are acute angles
x +1 2x +1
π π π
(a) 0 (b) (c) (d)
4 3 2
cot 54° tan 20°
44. The value of + is
tan 36° cot 70°
(a) 0 (b) 1 (c) 2 (d) 3
5 1  π
45. If tan α = and tan β = , where  0 < α , β <  , then
6 11  2
π π π π
(a) α + β = (b) α + β = (c) α + β = (d) α + β =
6 4 3 2
tan 70° − tan 20°
46. The value of is equal to
tan 50°
(a) 0 (b) 1 (c) 2 (d) 3
47. The value of tan ( −945° ) is
1
(a) −3 (b) − 3 (c) − (d) −1
3
48. sin 200° + cos 200° is
(a) negative (b) zero (c) positive (d) either zero or positive
49. If A, B, C , D are the angles of a cyclic quadrilateral, then cos A + cos B + cos C + cos D is equal to
(a) −1 (b) 0 (c) 1 (d) 4
tan 255° + tan 345°
50. Evaluate if cot15° = m
tan195° − tan105°
m −1 2m m −1
(a) (b) 2 (c) 2 (d) None of these
m +1 m +1 m +1
5
51. If cosec θ + cot θ = , then the value of tan θ is
2
14 20 21 15
(a) (b) (c) (d)
25 21 20 16
52. cos θ cos ( 90° − θ ) − sin θ sin ( 90° − θ ) equals

th
Office.: 606 , 6 Floor, Hariom Tower, Circular Road, Ranchi-1, Ph.: 0651-2562523, 9835508812, 8507613968
( ) BY R. K. MALIK’S NEWTON CLASSES 5
(a) −1 (b) 0 (c) 1 (d) 2
1
53. If cos θ = − and 0° < θ < 360° , then the values of θ are
2
(a) 60° and 120° (b) 60° and 240° (c) 120° and 240° (d) 120° and 300°
sin 2 θ 1 + cos θ sin θ
54. 1− + = equals
1 + cos θ sin θ 1 − cos θ
(a) 0 (b) 1 (c) sin θ (d) cos θ
6 6 2 2
55. The value of the expression sin θ + cos θ + 3sin θ cos θ equals
(a) 0 (b) 1 (c) 2 (d) greater than 3
56. If cosec θ − cot θ = q , then the value of cot θ is
2q 2q 1 − q2 1 + q2
(a) (b) (c) (d)
1 + q2 1 − q2 2q 2q
3π 1 + tan A + cosecA
57. If sec A = 2 and < A < 2π , then is equal
2 1 + cot A − cosec A
1
(a) − 2 (b) −1 (c) 0 (d)
2
5sin θ − 3cos θ
58. If θ lies in the first quadrant and 5 tan θ = 4 , then is equal to
sin θ + 2 cos θ
1 3 5
(a) 0 (c) (c) (d)
14 14 14
π π π
59. The cotangent of the angles , and are in
3 4 6
(a) A.P. (b) G.P. (c) H.P. (d) None of these
3cos θ + cos 3θ
60. is equal to
3sin θ − sin 3θ
(a) 2 cot θ (b) cot 2 θ + 1 (c) cot 3 θ (d) cot 4 θ
61. The value of log 3 tan1° + log 3 tan 2° + .... + log 3 tan 89° is
(a) 0 (b) 1 (c) 2 (d) 3
62. Which of the following is correct?
π
(a) sin1° = sin1 (b) sin1° =sin1 (c) sin1° < sin1 (d) sin1° > sin1
180
63. The value of tan1° ⋅ tan 2° tan 3°....tan 89° is equal to
π
(a) −1 (b) 1 (c) (d) 2
2
12  π −3  3π 
64. If sin θ =  0 < θ <  and cos φ = , π < φ <  , then sin (θ + φ ) will be
13  2 5  2 
1 56 56 1
(a) − (b) − (c) − (d)
56 65 61 65
1 − tan 2 15°
65. The value of is
1 + tan 2 15°
3
(a) 1 (b) (c) 3 (d) 2
2

th
Office.: 606 , 6 Floor, Hariom Tower, Circular Road, Ranchi-1, Ph.: 0651-2562523, 9835508812, 8507613968
6 ( BY R. K. MALIK’S NEWTON CLASSES
)
cos (θ1 − θ 2 )
66. If tan θ1 = k cot θ 2 , then is equal to
cos (θ1 + θ 2 )
k −1 1− k k +1 1+ k
(a) (c) (c) (d)
k +1 1+ k k −1 1− k
67. The value of cos 480° ⋅ sin150° + sin 600° ⋅ cos 390° is
1 1
(a) −1 (b) − (c) 0 (d)
2 2
2
68. Find the value of k for which ( cos x + sin x ) + k sin x cos x − 1 = 0 is an identity.
(a) −2 (b) −1 (c) 1 (d) 2
69. The value of cot105° is
(a) 3−2 (b) 2 − 3 (c) 2 +3 (d) 3+2
sin (α − β ) sin ( β − γ ) sin ( γ − α )
70. The expression + + is equal to
sin α sin β sin β sin γ sin γ sin α
(a) 0 (b) 1 (c) sin (α + β + γ ) (d) sin α + sin β + sin γ
71. If sin x + sin 2 x = 1 , then the value of cos 2 x + cos 4 x is
(a) 1 (b) 0 (c) −1 (d) 2
a b a2 b2
72. If x = h + , y=k+ , then 2
+ 2
is equal to
cos θ sin θ ( x − h) ( y − k )
ab
(a) ab (b) (d) 2ab (d) 1
a+b
tan θ − cot ϕ
73. The expression is equal to
tan ϕ − cot θ
tan θ 1 + tan θ 1 − tan θ tan ϕ
(a) (b) (c) (d)
tan ϕ 1 + tan ϕ 1 − tan ϕ tan θ
74. The value of tan 5° ⋅ tan15° ⋅ tan 25° ⋅ tan 45° ⋅ tan 35° ⋅ tan 55° ⋅ tan 65° ⋅ tan 75° ⋅ tan 85° is
(a) ∞ (b) 0 (c) 1 (d) −1
75. The value of cos ( 54° + θ ) cos ( 54° − θ ) + cos ( 36° − θ ) cos ( 36° + θ ) is

(a) cos θ (b) cos 2θ (c) cos 3θ (d) sin 2θ


3 12 π 3π
76. If sin A = , cos B = , where < A < π and π < B < , then the value of tan ( A + B ) is
5 13 2 2
16 63 63 16
(a) (b) (c) − (d) −
63 16 16 63
2x  π
77. If x = (1 − cos θ ) / (1 + cos θ ) then 2
is equal to: where θ ∈  0, 
(1 − x )  2
where θ lies in first quadratnt
(a) sin θ (b) cos θ (c) tan θ (d) cot θ .

th
Office.: 606 , 6 Floor, Hariom Tower, Circular Road, Ranchi-1, Ph.: 0651-2562523, 9835508812, 8507613968
( ) BY R. K. MALIK’S NEWTON CLASSES 7
θ
sin + sin θ
78. The expression 2 is equal to
θ
1 + cos + cos θ
2
θ θ θ θ
(a) tan (b) cot (c) sin (d) cos
2 2 2 2
1 + cos 2θ + sin 2θ
79. The value of is equal to
1 − cos 2θ + sin 2θ
(a) cot θ (b) tan θ (c) − secθ (d) cos ec θ

 π
80. The value of 2 + 2 + 2cos 4θ is where θ ∈  θ , 
 4
(a) cos θ (b) 2 cos θ (c) cos 2θ (d) 2 cos 2θ
sin 2 x + sin 5 x − sin x
81. The value of is
cos 2 x + cos5 x + cos x
(a) cot 2 x (b) tan x (c) tan 2 x (d) cot x
sin ( n + 1) α − sin ( n − 1) α
82. The value of is
cos ( n + 1) α + 2 cos nα + cos ( n − 1) α

α α α
(a) tan α (b) tan (c) cot (d) sec
2 2 2
sin 70° − cos 40°
83. The value of is
cos 70° + sin 40°
1 1
(a) 3 (b) (c) (d) tan10°
3 3
n n
 cos A + cos B   sin A + sin B 
84. The value of  + , when n is odd, is
 sin A − sin B   cos A − cos B 
 A− B
(a) 0 (b) −1 (c) 1 (d) 2 cot n 
 2 
cos (α + β )
85. If cot α cot β = 2 then is
cos (α − β )

1 1
(a) 3 (b) 2 (c) − (d)
3 2
86. The value of cos 0° + cos1° + cos 2° + ... + cos179° is
(a) 0 (b) 1 (c) −1 (d) 1/ 2
87. If A + B + C = 90° then cos 2 A + cos 2 B + cos 2 C is
(a) 2 (1 + cos A cos B cos C ) (b) 2 + 2 sin A sin B sin C

(c) 2 (1 − sin A sin B sin C ) (d) 2 (1 − cos A cos B cos C )


88. If tan ( A + B ) = p , tan ( A − B ) = q , then the value of tan 2 A in terms of p and q is

th
Office.: 606 , 6 Floor, Hariom Tower, Circular Road, Ranchi-1, Ph.: 0651-2562523, 9835508812, 8507613968
8 ( ) BY R. K. MALIK’S NEWTON CLASSES
p−q p+q p+q 1 − pq
(a) (b) (c) (d)
1 + pq p−q 1 − pq 1 + pq
cos10° + sin10°
89. is equal to
cos10° − sin10°
(a) − cot 35° (b) − tan 35° (c) tan 55° (d) cot 55°
90. The value of 12 sin 40° − 16sin 3 40° is
(a) −3 2 (b) −2 3 (c) 2 3 (d) 3 2
1 1
91. In a triangle ABC , let tan A = and tan B = . The value of angle C is …….
2 3
π π 3π 2π
(a) (b) (c) (d)
4 3 4 3
92. If A lies in the third quadrant and 3 tan A − 4 = 0 , then 5sin 2 A + 3sin A + 4 cos A is equal to
24 24 48
(a) − (b) 0 (c) (d)
5 5 5
8
93. If cos θ = and θ lies in the Ist quadrant, then the value of cos ( 30° + θ ) + cos ( 45° − θ )
17
+ cos (120° − θ ) is

23  3 − 1 1  23  3 + 1 1  23  3 − 1 1  23  3 + 1 1 
(a)  −  (b)  −  (c)  +  (d)  + 
17  2 2  17  2 2  17  2 2  17  2 2 
94. sin 2 17.5° + sin 2 72.5° is equal to
(a) cos 2 30° (b) sin 2 45° (c) tan 2 45° (d) cos 2 90°
95. The value of 4sin A cos3 A − 4 cos A sin 3 A is equal to
(a) sin 2 A (b) sin 3 A (c) cos 3 A (d) sin 4 A
3 3
cos θ − cos 3θ sin θ + sin 3θ
96. + is equal to
cos θ sin θ
(a) 0 (b) 1 (c) 3 (d) 5
2 2
97. If tan α = 2 tan β + 1 , then cos 2α is equal to
(a) − tan β (b) tan 2 β (c) − sin β (d) − sin 2 β
cos C − cos A
98. If A + C = 2 B, then is equal to:
sin A − sin C
(a) tan B (b) cot B (c) tan 2 B (d) cot 2 B .
1 13
99. If cos P = and cos Q = , where P and Q both are acute angles. Then, the value of P − Q is
7 14
(a) 30° (b) 45° (c) 60° (d) 75°
100. sin120° cos150° − cos 240° sin 330° is equal to
3 +1 2
(a) − (b) −1 (c) 1 (d)
4 3
π  π 
101. cos 4   − sin 4   is equal to
 24   24 
5 −1 5 −1 3 +1 2+ 2
(a) (b) (c) (d)
2 2 4 2 2 2

th
Office.: 606 , 6 Floor, Hariom Tower, Circular Road, Ranchi-1, Ph.: 0651-2562523, 9835508812, 8507613968
BY R. K. MALIK’S NEWTON CLASSES
( ) 9
sin 300° tan 330° sec 420°
102. The value of is equal to
tan135° sin 210° sec 315°
1 1
(a) (b) (c) 2 (d) 3
3 2
 π  3π   5π   7π 
103. 1 + cos  1 + cos  1 + cos  1 + cos  is equal to:
 8 8  8  8 
1 1 1+ 2 π
(a) (b) (c) (d) cos .
8 2 2 2 8
104. Find the maximum value of sin x ⋅ sin y if x + y = 90° .
1 1 2
(a) (b) (c) (d) 1
3 2 3
105. If tan α − tan β = m and cot α − cot β = n , then cot (α − β ) is equal to
1 1 1 1 1 m n
(a) 2
− 2 (b) (c) − (d) −
m n m − n2
2
m n n m
m +1
106. If sec θ − tan θ = , then cos θ is equal to
m −1
2m 2m m2 − 1 m2 + 1
(a) (b) (c) (d)
m2 − 1 m2 + 1 m2 + 1 m2 − 1
1 1
107. If tan α = and tan β = then ( 2α + β ) is equal to (where α , β are acute)
3 7
π π π π
(a) (b) (c) (d)
6 4 3 2
2
108. If 12 cot θ − 31cosec θ + 32 = 0 , then the value of sin θ is
2 3 1 4 3
(a) ± (b) or 1 (c) ± (d) or
3 5 2 5 4
1 1
109. − is equal to
sec θ − tan θ cos θ
1 1 1 1
(a) + (b) −
sin θ sec θ + tan θ sin θ sec θ + tan θ
1 1 1 1
(c) + (d) −
cos θ sec θ + tan θ cos θ sec θ + tan θ
α
110. If α is an acute angle and 2 x ⋅ sin 2 + 1 = x , then tan α is
2
1
(a) x2 − 1 (b) x2 + 1 (c) x2 − 2 (d) x2 −
2
sin θ + sin 2θ
111. is equal to
1 + cos θ + cos 2θ
(a) sin θ (b) cos θ (c) tan θ (d) cot θ
3π 1
112. If < x < π then 2 cot x + is equal to
4 sin 2 x
− sin x − cos x − sin x + cos x sin x − cos x sin x + cos x
(a) (b) (c) (d)
sin x sin x sin x sin x

th
Office.: 606 , 6 Floor, Hariom Tower, Circular Road, Ranchi-1, Ph.: 0651-2562523, 9835508812, 8507613968
10 ( BY R. K. MALIK’S NEWTON CLASSES
)
113. cos1° + cos 2° + cos 3° + ..... + cos180° is equal to
(a) −1 (b) 0 (c) 1 (d) 2
114. If angle C of a triangle ABC satisfies the equation 5cos C + 3 = 0 , then tan C and sin C are the roots
of the equation
(a) 15 x 2 − 8 x − 16 = 0 (b) 15 x 2 − 8 x + 16 = 0 (c) 15 x 2 + 8 x − 16 = 0 (d) 15 x 2 + 8 x + 16 = 0
1
115. If sin (α + β ) = 1, sin (α − β ) = then tan (α + 2 β ) ⋅ tan ( 2α + β ) is equal to (where α and β are
2
acute)
(a) −1 (b) 0 (c) 1 (d) None of these
116. Which of the following is incorrect?
2 2
(a) ( cot θ + tan θ ) = sec2 θ + cosec 2θ (b) ( cot θ + tan θ ) = sec2 θ ⋅ cosec 2θ
2
(c) ( cot θ − tan θ ) = 4 cot 2 2θ (d) None of these
 π
117. The set of all values of x in the interval 0,  for which 2 sin 2 x − 3sin x + 1 ≥ 0 is
 2
 π π π   π π π 
(a) 0,  (b)  ,  (c) 0,  (d)  , 
 6 6 3  4 4 3
1 + sin 2 θ sin 2 θ sin 2 θ
118. If cos 2 θ 1 + cos 2 θ cos 2 θ = θ , then sin 4θ is equal to
4 sin 4θ 4 sin 4θ 1 + 4 sin 4θ
1 1
(a) −1 (b) − (c) (d) 1
2 2
π A π A
119. sin 2  +  − sin 2  −  is equal to
8 2 8 2
3 1 1 1
(a) sin A (b) sin A (c) sin A (d) sin A
2 2 2 2 2
b
120. If tan x = , then the value of a cos 2 x + b sin 2 x is
a
(a) a − b (b) a (c) b (d) a + b
121. If tan α + cot α = m , then tan 3 α + cot 3 α is equal to
(a) m ( m 2 − 3m + 3) (b) m ( m 2 − 3) (c) m 2 ( m + 3) (d) m ( m 2 − m + 3)

π  π 
122. cos 2  + θ  − sin 2  − θ  is equal to
6  6 
1 1 1 θ
(a) − cos 2 θ (b) 0 (c) cos 2θ (d) cos 2
2 2 2 2
sin 2 A + sin 2 B + sin 2C
123. If A + B + C = π , then is equal to
cos A + cos B + cos C − 1
 A+ B   B+C  C + A A B C
(a) 2 cos   cos   cos   (b) 4 sin sin sin
 2   2   2  2 2 2
A B C  A+ B   B+C  C + A
(c) 8 cos cos cos (d) 4 tan   tan   tan  
2 2 2  2   2   2 
124. If A + B + C = π , then sin 2 A + sin 2 B + sin 2C , is equal to

th
Office.: 606 , 6 Floor, Hariom Tower, Circular Road, Ranchi-1, Ph.: 0651-2562523, 9835508812, 8507613968
BY R. K. MALIK’S NEWTON CLASSES
( ) 11
(a) 2sin A sin B sin C (b) 2 cos A cos B cos C (c) 4sin A sin B sin C (d) 4 cos A cos B cos C
125. The expression 2 cos θ − cos 3θ − cos 5θ = ……
(a) 8 cos 2 θ sin 3 θ (b) 2 cos 2θ cos 3θ (c) 16 sin 2 θ cos3 θ (d) 4 sin 3θ sin 2θ
126. If 2 cos x + 2 cos 3 x = cos y and 2sin x + 2 sin 3x = sin y, then the value of cos 2 x is
7 1 1 7
(a) − (b) − (c) (d)
8 8 8 8
θ x −1
127. If θ is an acute angle and sin = then tan θ is equal to
2 2x
(a) x2 − 1 (b) x 2 − 1 (c) x2 + 1 (d) x 2 + 1
sin 3θ + sin 5θ + sin 7θ + sin 9θ
128. = ....
cos 3θ + cos 5θ + cos 7θ + cos 9θ
(a) cot 5θ tan 7θ (b) 2 tan 2θ (c) 2 cot 6θ (d) tan 6θ
 cos α sin α 
129. If A (α ) =  , then A (α ) . A ( β ) is equal to
 − sin α cos α 
(a) A (α − β ) (b) A (α ) − A ( β ) (c) A (α ) + A ( β ) (d) A (α + β )
130. The value of cos15° − sin15° is
1 1 1
(a) − (b) 0 (c) (d)
2 2 2
131. For all values of x, the expression sin x + cos x is always
1
(a) ≥ 2 (b) ≥ 1 (c) ≤ 2 (d) ≥
2
3 tan ( A / 3) − tan 3 ( A / 3)
132. is equal to
1 − 3 tan 2 ( A / 3)
(a) sin A (b) 2 A − tan A (c) cot A (d) tan A
C 1 0
133. If C = 2 cos θ , then the value of the determinant ∆ = 1 C 1 is
6 1 C
sin 4θ 2sin 2 2θ
(a) 4 cos 2 θ ( 2 cos θ − 1) (b) (c) (d) None of these
sin θ sin θ
134. The expression cos 20° cos 40°, cos 60° cos80° is equal to
1 3 5 1
(a) (b) (c) (d)
4 8 12 16
sin α − sin γ
135. If α , β , γ are in A.P. then is equal to
cos γ − cos α
(a) sin α (b) cos α (c) tan β (d) cot β
136. cos 52° + cos 68° + cos172° is equal to
(a) 0 (b) 1 (b) 2 (d) None of these
2 2 2
137. If A + B + C = 2π , then cos A + cos B − sin C is equal to
(a) 2 cos A cos B sin C (b) 2 sin A sin B cos C
(c) 2 cos A cos B cos C (d) 2sin A sin B sin C

th
Office.: 606 , 6 Floor, Hariom Tower, Circular Road, Ranchi-1, Ph.: 0651-2562523, 9835508812, 8507613968
12 ( ) BY R. K. MALIK’S NEWTON CLASSES
2 4
138. If x = a ( sec θ + tan θ ) , y = b ( sec θ − tan θ ) , then x 4 y 2 is equal to
(a) a 2b 4 (b) a 4b 2 (c) a 2b 2 tan 2 θ (d) ab sec θ
139. The value of (1 + tan A tan B ) 2 + (tan A − tan B ) 2 is:
(a) cos 2 A cos 2 B (b) tan 2 A tan 2 B (c) cot 2 A cot 2 B (d) sec 2 A sec2 B .
140. The maximum value of the expression 3sin θ + cos 2θ is
13 11 17 4
(a) (b) (c) (d)
7 15 8 5
2π π
141. The value of sin 2 − sin 2 is
15 30
5 −1 5 −1 5 +1 5 +1
(a) (b) (c) (d)
8 6 3 4
3
 π
142. ∑ cos ( 2k − 1) 12  is equal to
k =1
2

1 1 3
(a) − (b) 0 (c) (d)
2 2 2
π
143. The minimum value of f ( x ) = sin 4 x + cos 4 x, 0 ≤ x ≤ , is
2
1 1 1 1
(a) − (b) (c) (d)
4 2 2 4 2
144. If A + B + C = π , then sin A + sin 2 B + sin 2 C is equal to
2

(a) 2 − cos A cos B cos C (b) 1 − 2sin A sin B sin C


(c) 2 + sin A sin B sin C (d) 2 + 2 cos A cos B cos C
π 3π 15 12
145. If < α < π , π < β < ; sin α = and tan β = , then the value of sin ( β − α ) is
2 2 17 5
171 21 21 171
(a) − (b) − (c) (d)
221 221 221 221
146. The expression cos 2θ + 2 cos θ has values lying in the interval
 3   3 1  1 
(a)  − , 3 (b)  − ,  (c)  − , 2 (d) [ −2, 2]
 2   2 2  2 
147. cos 2 α + cos 2 (α − 120° ) + cos 2 (α + 120° ) is equal to
1 3
(a) 0 (b) (c) 1 (d)
2 2
148. If A = 35°, B = 15° and C = 40° then tan A. tan B + tan B.tan C + tan C. tan A is equal to
(a) 0 (b) 1 (c) 2 (d) 3
149. The value of 2 ( sin 6 θ + cos 6 θ ) − 3 ( sin 4 θ + cos 4 θ ) + 1 is equal to
(a) 0 (b) 2 (c) 4 (d) 6
sin 5θ
150. is equal to
sin θ
(a) 16 cos 4 θ − 12 cos 2 θ − 1 (b) 16 cos 4 θ − 12 cos 2 θ + 1
(c) 16 cos 4 θ + 12 cos 2 θ − 1 (d) 16 cos 4 θ + 12 cos 2 θ + 1
151. If 1 + cos α + cos 2 α + ...... = 2 − 2, then α ( 0 < α < π ) is

th
Office.: 606 , 6 Floor, Hariom Tower, Circular Road, Ranchi-1, Ph.: 0651-2562523, 9835508812, 8507613968
( ) BY R. K. MALIK’S NEWTON CLASSES 13
π π π 3π
(a) (b) (c) (d)
8 6 4 4
 1   1 
152. cos 2  7 °  − cos 2  37 °  is equal to
 2   2 
3 1 1 1
(a) (b) (c) (d)
4 2 2 2 2
π   3π   5π   7π   9π 
153. The expression tan   tan   tan   tan   tan   is equal to
 20   20   20   20   20 
2
(a) (b) 1 (c) 2 3 (d) 2
3
π π 5π
154. The value of cos 2 + cos 2 + cos 2 is
12 4 12
3+ 3 3 2 2
(a) (b) (c) (d)
2 2 3+ 3 3
155. If A + B + C = 0 , then sin A + sin B + sin C is equal to
A B C A B C
(a) −4sin sin sin (b) 1 − 4 cos cos cos
2 2 2 2 2 2
A B C A B C 1
(c) −2sin sin sin − 1 (d) −2 cos cos cos +
2 2 2 2 2 2 2
156. The value of cos 35° + cos85° + cos155° is:
1 1
(a) − (b) 0 (c) (d) 1.
2 2
1
157. If sin ( A + B + C ) = 1, tan ( A − B ) = and sec ( A + C ) = 2, then where A, B, C are acute
3
(a) A = 45°, B = 15°, C = 30° (b) A = 60°, B = 30°, C = 0°
(c) A = 90°, B = 60°, C = 30° (d) A = 30°, B = 60°, C = 0°
158. Which of the following statements is incorrect?
A B C
(a) sin A + sin B + sin C = 4 cos cos cos when A + B + C = π
2 2 2
A B C
(b) sin A + sin B + sin C = 4sin sin sin when A + B + C = 2π
2 2 2
A B C
(c) sin A + sin B + sin C = 4sin sin sin when A + B + C = 0
2 2 2
(d) sin 2 A + sin 2 B + sin 2C = 4sin A sin B sin C when A + B + C = π
π  π 
159. The expression sin θ sin  − θ  sin  + θ  is equal to:
3  3 
1 1 1 1
(a) sin 2θ (b) sin 2 θ (c) sin 3θ (d) sin 3θ .
3 4 3 4
1 1
160. If θ and φ are angles in the 1st quadrant such that tan θ = and sin φ = , then
7 10
(a) θ + 2φ = 30° (b) θ + 2φ = 45° (c) θ + φ = 90° (d) θ + 2φ = 90°
161. In ∆ABC , the value of cosec A ( sin B cos C + cos B sin C ) is equal to

th
Office.: 606 , 6 Floor, Hariom Tower, Circular Road, Ranchi-1, Ph.: 0651-2562523, 9835508812, 8507613968
14 ( BY R. K. MALIK’S NEWTON CLASSES
)
1
(a) −1 (b) 0 (c) 1 (d)
2
162. The expression cos 6θ + cos 4θ + cos 2θ + 1 is equal to
(a) 2 sin θ sin 3θ (b) 4 sin 2 3θ (c) 4 cos θ cos 2θ cos 3θ (d) 2sin 3θ cos 3θ
2 2
163. If ( sin α + cosecα ) + ( cos α + sec α ) = k + tan 2 α + cot 2 α , then k is equal to
(a) 3 (b) 5 (c) 7 (d) 9
sin 70° + cos 40°
164. The value of is equal to
cos 70° + sin 40°
1 1
(a) (b) (c) 1 (d) 3
2 3
π  π 
165. The value of cos 2  + θ  − sin 2  − θ  is
4  4 
(a) 0 (b) cos θ (c) sin 2θ (d) cos 2θ
7
166. The ratio of the maximum value of sin 2 θ − cos θ + to its minimum value is
4
11
(a) 2 (b) 4 (c) 7 (d)
4
 π
167. The maximum value of 5sin θ + 3sin  θ +  + 3 is
 3
(a) 9 (b) 10 (c) 11 (d) 12
168. The expression tan 20°.tan 40°. tan 60°.tan 80° is equal to
(a) 3 (b) 2 3 (c) 3 (d) 3 3
π  π 
169. The expression cos θ cos  − θ  cos  + θ  is equal to:
3  3 
1 1 1 1
(a) cos 2θ (b) cos 2 θ (c) sin 3θ (d) cos 3θ .
3 4 3 4
170. sin163° cos 347° + sin 73° sin167° is equal to
1
(a) 0 (b) (c) 1 (d) None of these
2
π  π 
171. The maximum value of 2 + sin  + θ  + 2 cos  − θ  is:
4  4 
(a) 2 (b) 3 + 2 (c) 2+ 5 (d) 7.
π
172. If A − B = , then (1 + tan A )(1 − tan B ) is equal to
4
(a) −2 (b) −1 (c) 1 (d) 2
173. The value of sin 600° cos 330° + cos120° sin150° is
1 3
(a) −1 (b) (c) 1 (d)
2 2
π
174. If A + B + C = , find sin 2 A + sin 2 B + sin 2C .
2
(a) 4 cos A sin B sin C (b) 4sin A sin B sin C (c) 4 cos A cos B cos C (d) 4 cos A cos B sin C
175. The value of sin12°.sin 48°.sin 54° is:

th
Office.: 606 , 6 Floor, Hariom Tower, Circular Road, Ranchi-1, Ph.: 0651-2562523, 9835508812, 8507613968
(BY R. K. MALIK’S NEWTON CLASSES
) 15
1 1 1 1
(a) (b) (c) (d) .
8 6 3 2
x y z
176. If = = , then the value of x + y + z is
cos θ  2π   2π 
cos  θ +  cos  θ − 
 3   3 
1 1
(a) 1 (b) 0 (c) (d) −
2 2
sin 2 A − sin 2 B
177. is equal to
sin A cos A − sin B cos B
(a) tan ( A − B ) (b) cot ( A − B ) (c) tan ( A + B ) (d) cot ( A + B )
sin 85° − sin 35°
178. The value of is
cos 65°
(a) −1 (b) 0 (c) 1 (d) 2
sin ( −660° ) tan (1050° ) sec ( −420° )
179. is equal to
cos ( 225° ) cosec ( 315° ) cos ( 510° )

4 2 3 3
(a) (b) (c) (d)
3 3 2 4
3 A 5A
180. If cos A = , then the value of 32sin sin is equal to
4 2 2
(a) −11 (b) − 11 (c) 11 (d) 11
181. The minimum value of ( 4 tan 2 θ + 9 cot 2 θ ) is
(a) 4 (b) 6 (c) 9 (d) 12
182. Which is not true in a triangle?
(a) sin A = sin B cos C + cos B sin C (b) cos C = sin A sin B − cos A cos B
(c) tan A + tan B + tan C = tan A ⋅ tan B ⋅ tan C (d) cot A + cot B + cot C = cot A ⋅ cot B ⋅ cot C
 x+ y
183. If cos x + cos y + cos α = 0 and sin x + sin y + sin α = 0 , then cot   is equal to
 2 
 x+ y
(a) sin α (b) cos α (c) cot α (d) sin  
 2 
4 xy
184. sec2 θ = 2
is true if and only if
( x + y)
(a) x ≠ 0, y ≠ 0 (b) x = y, x ≠ 0 (c) x = y (d) x + y ≠ 0
A B
185. If A + B + C = 180° , then ∑ tan 2 tan 2 is equal to

(a) 0 (b) 1 (c) 2 (d) 3


π   π 
186. If tan θ ⋅ tan  + θ  ⋅ tan  − + θ  = k tan 3θ , then the value of k is
3   3 
1
(a) −1 (b) (c) 3 (d) None of these
3
n −1  A+ B 
187. If sin A = n sin B , then tan   is equal to
n +1  2 

th
Office.: 606 , 6 Floor, Hariom Tower, Circular Road, Ranchi-1, Ph.: 0651-2562523, 9835508812, 8507613968
16 ( ) BY R. K. MALIK’S NEWTON CLASSES
 A− B   A− B   A− B   A− B 
(a) sin   (b) tan   (c) cot   (d) cos  
 2   2   2   2 
188. If A + B + C = 180° , then cos 2 A + cos 2 B + cos 2C is equal to
(a) −1 + 4sin A sin B sin C (b) 1 − 4 cos A cos B cos C
(c) 1 + 4 cos A cos B cos C (d) None of these
 π tan 2 α
189. If α ∈  0,  , then x2 + x + is always greater than or equal to
 2 x2 + x
(a) 1 (b) 2 (c) 2 tan α (d) sec2 α
190. The quadratic equation whose roots are sin 2 18° and cos 2 36° is
(a) 16 x 2 − 12 x − 1 = 0
(b) 16 x 2 − 12 x + 1 = 0 (c) 16 x 2 + 12 x − 1 = 0 (d) 16 x 2 + 12 x + 1 = 0
A B
191. If A + B + C = 180° , then ∑ tan ⋅ tan is equal to
2 2
(a) 0 (b) 1 (c) 2 (d) 3
192. If sin α = sin β and cos α = cos β , then:
α − β  α − β  α + β  α + β 
(a) sin  =0 (b) cos  =0 (c) sin  =0 (d) cos  =0.
 2   2   2   2 
193. If A + B + C = 270°, then cos 2 A + cos 2 B + cos 2C +4sin A sin B sin C is equal to:
(a) 0 (b) 1 (c) 2 (d) 3.
sec8 A − 1
194. is equal to
sec 4 A − 1
cot 8 A tan 8 A tan 2 A
(a) (b) (c) (d) None of these
cot 2 A tan 2 A tan 8 A
195. The expression sin 36° sin 72° sin108° sin144° is equal to
3 5 7 3
(a) (b) (c) (d)
8 16 12 4
π   2π 
196. The maximum value of cos 2  + x  + cos 2  + x  is
3   3 
3 2 2 3
(a) − (b) − (c) (d)
2 3 3 2
197. If x = h + a sec θ and y = k + bcosec θ , then
2 2

(a)
a2
+
b2
=1 (b)
( x − y) +
(y −k) =1
2 2
( x − h) ( y −k) a2 b2
2 2

(c)
a2

b2
=1 (d)
( x − y) −
( y −k) =1
2 2
( x − h) ( y −k) a2 b2

π  π 
198. The extreme values of 4 cos ( x 2 ) cos  + x 2  cos  − x 2  are
3  3 
(a) −1, 1 (b) −2, 2 (c) −3, 3 (d) −4, 4
 π    3π  
199. The expression 6 sin 4 ( 5π + θ ) + sin 4  + θ   −4 sin 6  + θ  + sin 6 ( 3π + θ )  is equal to
 2    2  
1 1
(a) 2 (b) 2sin 2θ (c) sin 2 cos 2 θ (d) cos 2θ
2 2

th
Office.: 606 , 6 Floor, Hariom Tower, Circular Road, Ranchi-1, Ph.: 0651-2562523, 9835508812, 8507613968
( ) BY R. K. MALIK’S NEWTON CLASSES 17

θ 
200. If tan β = cos θ tan α , then tan 2   is equal to
2
cos (α + β ) sin (α − β )
(a) (b) cot 2 ( β − α ) (c) (d) tan (α − β ) cot (α + β )
cos (α − β ) sin (α + β )
3π 4π
201. The value of cos 2 + cos 2 is equal to
5 5
3 4 5 5
(a) (b) (c) (d)
4 5 4 2
cos (α − β ) p +1
202. If = , then p is equal to
sin (α + β ) p −1
π  π  π  π 
(a) tan  − α  tan  − β  (b) tan  − α  tan  + β 
4  4  4  4 
π  π  π  π 
(c) tan  + α  tan  − β  (d) tan  + α  tan  + β 
4  4  4  4 
 π
203. If sin 4 A − cos 2 A = cos 4 A − sin 2 A,  0 < A <  , then the value of tan 4A is
 4
1 3 +1
(a) 1 (b) (c) 3 (d)
3 3 −1
 B + 2C + 3 A   A− B 
204. In a ∆ABC , cos   + cos   is equal to
 2   2 
(a) −1 (b) 0 (c) 1 (d) 2
2 2
205. Evaluate cot 36° cot 72°
5 −1 1 5 +1 5
(a) (b) (c) (d)
4 5 4 2
206. If tan A − tan B = x and cot B − cot A = y, then cot( A − B ) is equal to:
1 1 y−x 1 1 y−x
(a) − (b) (c) + (d) .
x y y+x x y 2y − x
3 −1
207. If tan A = , (where A is acute angle) then, cos A =
3 +1
3 −1 3 +1
(a) (b) (c) 3 +1 (d) 3 −1
2 2 2 2
1 cosec2θ − sec 2 θ
208. If tan θ = , (where θ is acute angle) then,
7 cosec2θ + sec 2 θ
1 3 5 7
(a) (b) (c) (d)
2 4 6 8
209. If tan θ + cot θ = 2, (where θ is acute angle) then, sin θ + cos θ =
(a) 5 (b) 2 (c) 3 (d) 2
210. If 2 cos A ⋅ tan B − 2 cos A − tan B + 1 = 0, (where A and B are acute) then the measures of angles A
and B are respectively
(a) 60°, 45° (b) 30°, 45° (c) 45°, 75° (d) 45°, 60°

th
Office.: 606 , 6 Floor, Hariom Tower, Circular Road, Ranchi-1, Ph.: 0651-2562523, 9835508812, 8507613968
18 BY R. K. MALIK’S NEWTON CLASSES
( )
sin θ + cos θ sin θ − cos3 θ
3 3 3
211. + =
sin θ + cos θ sin θ − cos θ
(a) 0 (b) 1 (c) 2 (d) 3
212. ( sec θ + tan θ − 1)( secθ − tan θ + 1) =
(a) 2sin θ (b) 2 cos θ (c) 2sec θ (d) 2 tan θ
2 2
213. If ( sin θ + cosec θ ) + ( cos θ + sec θ ) = tan 2 θ + cot 2 θ + k then, k =
(a) 1 (b) 3 (c) 5 (d) 7
214. If sin 4 θ + cos 4 θ + sin 2 θ .cos 2 θ = 1 − u 2 , then u =
(a) sin θ ⋅ cos θ (b) sec θ ⋅ cosec θ (c) sec θ ⋅ tan θ (d) cosec θ ⋅ cot θ
215. If ( sin θ + cos θ )(1 − sin θ ⋅ cos θ ) = sin θ + cos θ , then
n n
n=
(a) 0 (b) 1 (c) 2 (d) 3
216. If p ⋅ cos θ = q 2 − p 2 ⋅ sin θ , (where θ is acute angle) then q ⋅ sin θ =

(a) p (b) p2 − q2 (c) p 2 + q 2 ⋅ cos θ (d) None of these


217. If sin θ = 3 ⋅ cos θ , (where θ is acute angle) then, 2 ( sin θ + cos θ ) − 1 =

(a) 3 (b) 2 (c) 1 (d) 0


218. If sec θ + cos θ = 2, then sec 2 θ − sec4 θ =
(a) 1 (b) cos 2 θ + cos 4 θ (c) −1 (d) cos 4 θ − cos 2 θ
219. If tan θ + cot θ = 2, then tan 2 θ − tan 3 θ =
(a) cot 3 θ − cot 2 θ (b) 1 (c) cot 2 θ + cot 3 θ (d) 2
220. If 1 + sin α ⋅ sin β − cos α ⋅ cos β = 0 then tan α .cot β =
(a) −3 (b) −2 (c) −1 (d) 0
A B+C A+ B C
221. If A, B, C are the angles of a triangle, then tan ⋅ tan + cos ⋅ cosec =
2 2 2 2
(a) 1 (b) 2 (c) 0 (d) 3
222. If x = r ⋅ cos A ⋅ cos B, y = r ⋅ cos A ⋅ sin B, z = r ⋅ sin A, then x 2 + y 2 + z 2 =
(a) r 2 ( sin 2 A + cos 2 B ) (b) r 2 ( cos 2 A + sin 2 B ) (c) r 2 (d) 2r 2

223. If x = a ⋅ cos 2 θ + b ⋅ sin 2 θ and ( x − a )( b − x ) = c 2 ⋅ sin 2 θ ⋅ cos 2 θ , then c =


(a) a − b or b − a (b) a + b (c) a 2 − b 2 or b 2 − a 2 (d) a 2 + b 2
224. If x = cos ( A + B ) ⋅ cos ( A − B ) and y = sin ( A + B ) ⋅ sin ( A − B ) , then x − y =
(a) sin 2 A (b) cos 2A (c) sin 2B (d) cos 2B
π  π  π  π 
225. If x = sin  + A  ⋅ cos  + B  and y = cos  + A  ⋅ sin  + B  , then x − y =
3  3  3  3 
(a) sin ( A − B ) (b) cos ( A − B ) (c) sin ( A + B ) (d) cos ( A + B )
 A
226. 1 + tan A ⋅ tan   =
2
(a) sin A (b) cos A (c) cosec A (d) sec A
227. If x ⋅ cos A ⋅ cos B = sin ( A − B ) , y ⋅ cos B ⋅ cos C = sin ( B − C ) , z ⋅ cos A cos C = sin ( C − A) ,
then x + y + z =

th
Office.: 606 , 6 Floor, Hariom Tower, Circular Road, Ranchi-1, Ph.: 0651-2562523, 9835508812, 8507613968
( ) BY R. K. MALIK’S NEWTON CLASSES 19
(a) −1 (b) 0 (c) sin ( A + B + C ) (d) cos ( A − B − C )
228. If x = sin ( B − C ) ⋅ cos A, y = sin ( C − A) ⋅ cos B, and z = sin ( A − B ) ⋅ cos C , Then x + y + z =
(a) sin ( A − B − C ) (b) cos ( A − B − C ) (c) 0 (d) −1
 π  π
229. If 2 sin  x +  = cos  x −  , then tan x =
 3  6
(a) 1 / 3 (b) −1/ 3 (c) 3 (d) − 3
230. cos x + cos (120° − x ) + cos (120° + x ) =

(a) −1 (b) 0 (c) 3 (d) 1 / 3


231. tan ( 45° + A) ⋅ tan ( 45° − A ) =

(a) −1 (b) 0 (c) 1 (d) 2


232. tan 75° + tan15° =
(a) 1 (b) 2 (c) 3 (d) 4
233. If a = tan x, b = tan y, c = 1 + a 2 and d = 1 + b 2 (where x, y ∈ 1st quadrant) then cd ⋅ sin ( x + y ) =
(a) a − b (b) a + b (c) c − d (d) c + d
π  π 
234. cos 2  − θ  + cos 2  + θ  =
4  4 
(a) sin 2θ (b) cos 2θ (c) 1 (d) 0
π 3π 9π 11π
235. cos + cos + cos + cos =
12 12 12 12
6π 5π
(a) cos 2π (b) cos π (c) cos (d) cos
12 12
π 3π 5π 7π 9π
236. cot cot ⋅ cot ⋅ cot ⋅ cot =
20 20 20 20 20
(a) 1 (b) 2 (c) 3 (d) 4
π 3π 5π 7π
237. sin 2 + sin 2 + sin 2 + sin 2 =
8 8 8 8
(a) 1 (b) 2 (c) 3 (d) 4
1 + sin θ + cos θ θ  θ 
238. If = f   , then f   = ……..
1 + sin θ − cos θ 2 2
θ  θ  θ  θ 
(a) sin   (b) cos   (c) tan   (d) cot  
2 2 2 2
1 − tan 2 ( 45° − A )
239. =
1 + tan 2 ( 45° − A )
(a) sin 2 A (b) cos 2A (c) tan 2A (d) cot 2 A
sin 3θ cos 3θ
240. − =
sin θ cos θ
(a) sin 2θ (b) cos 4θ (c) 1 (d) 2
sin 2θ cos 2θ
241. − =
sin θ cos θ
(a) sec θ (b) tan θ (c) 1 (d) 2

th
Office.: 606 , 6 Floor, Hariom Tower, Circular Road, Ranchi-1, Ph.: 0651-2562523, 9835508812, 8507613968
20 ( BY R. K. MALIK’S NEWTON CLASSES
)
242. 4 sin θ ⋅ cos θ − 4 cos θ ⋅ sin 3 θ =
3

(a) 4 cos θ (b) cos 4θ (c) 4sin θ (d) sin 4θ


243. 1 − 8 cos 2 θ + 8cos 4 θ =
(a) sin 4θ (b) cos 4θ (c) sin 4 θ + cos 4 θ (d) sin 4 θ − cos 4 θ
244. If cos 3 A + cos 5 A + cos 7 A + cos15 A = 4 cos mA ⋅ cos nA ⋅ cos pA then, the value of m + n + p is
(a) 14 (b) 15 (c) 16 (d) 17
245. If sin 40° − cos 70° = k ⋅ cos80°, then k =
(a) 1 (b) 2 (c) 3 (d) 4
246. cos 20° + cos100° + cos140° =
(a) cos130° (b) cos 45° (c) 0 (d) None of these
247. cos 55° + cos 65° + cos175° =
(a) 0 (b) 1 (c) sin18° (d) cos 36°
248. sin10° + sin 20° + sin 40° + sin 50° =
(a) sin15° + sin 75° (b) cos15° + cos 75° (c) sin 70° + sin 80° (d) cos 70° + cos80°
cos 21° − sin 21°
249. =
cos 21° + sin 21°
(a) tan 21° (b) cot 66° (c) tan 42° (d) cot 42°
250. sin 7θ ⋅ sin θ + sin11θ ⋅ sin 3θ =
(a) sin 4θ ⋅ sin10θ (b) cos 4θ ⋅ cos10θ (c) sin 8θ ⋅ sin14θ (d) cos 8θ ⋅ cos14θ
π 2π 3π 4π 5π
251. 16 cos cos cos cos cos
12 12 12 12 12
2 3
(a) 3 (b) (c) (d) 2
3 2
252. If A + B + C = π , then sin 2 A + sin 2 B − sin 2C =
(a) 4 sin A ⋅ cos B ⋅ cos C (b) 4 cos A ⋅ sin B ⋅ cos C (c) 4 cos A ⋅ cos B ⋅ sin C (d) 4 cos A ⋅ cos B ⋅ cos C
253. sin 2 A + sin 2 B + sin 2 ( A − B ) =
(a) 4sin A.sin B ⋅ sin ( A − B ) (b) 4sin A ⋅ cos B ⋅ cos ( A − B )
(c) 4 cos A ⋅ sin B ⋅ cos ( A − B ) (d) 4 cos A ⋅ cos B ⋅ sin ( A − B )

254. tan 4 A + cot 4 A + 2 =


(a) 1 + sec 2 A + cosec 2 A (b) 2 + sec 2 A + cosec 2 A
(c) sec2 A ⋅ cosec2 A − 2 (d) 3 − sec 2 A ⋅ cosec 2 A
π  π 
255. If tan  + θ  − tan  − θ  = m. tan ( nθ ) , then
4  4 
(a) m < n (b) m = n (c) m > n (d) m n < 1
π θ  π θ 
256. sin 2  +  − cos 2  +  =
 4 2  4 2
(a) sin θ (b) cos θ (c) sin 2θ (d) cos 2θ
a
257. If cot θ = , then a ⋅ cos 2θ + b ⋅ sin 2θ =
b
(a) a 2 + b 2 (b) a 2 − b 2 (c) a (d) b

th
Office.: 606 , 6 Floor, Hariom Tower, Circular Road, Ranchi-1, Ph.: 0651-2562523, 9835508812, 8507613968
(BY R. K. MALIK’S NEWTON CLASSES
) 21
π 9π 3π 5π
258. The expression 2 cos cos + cos + cos is:
13 13 13 13
(a) – 1 (b) 0 (c) 1 (d) None of these.
259. The angle subtended at the centre of a circle of radius 3 m by an arc of length 1 m is equal to
1
(a) 20° (b) 60° rad (d) 3 rad
3
260. cos ( 540° − θ ) − sin ( 630° − θ ) =
(a) 0 (b) 2 cos θ (c) 2sin θ (d) sin θ + cos θ
tan155° − tan115°
261. If x = tan 25°, then =
1 + tan155° ⋅ tan115°
1 − x2 1 + x2 1 + x2 1 − x2
(a) (b) (c) (d)
2x 2x 1 − x2 1 + x2
262. If sin (120° − α ) = sin (120° − β ) , (where 0 < α , β < π ) then
π π
(a) α + β = (b) α = β or α + β = (c) α = β (d) α + β = 0
3 3
263. If 3sin 2θ = 5 + 4 cos 2θ , then tan θ =
(a) 1 (b) 3 (c) 4 (d) 5
tan160° − tan110°
264. If p = cot 20°, then =
1 + tan160° ⋅ tan110°
p2 −1 p2 +1 1 − p2 2p
(a) (b) (c) (d)
2p 2p 2p 1 + p2
sin ( x + y ) a+b tan x
265. If = , then =
sin ( x − y ) a −b tan y
b a
(a) (b) (c) ab (d) None of these
a b
266. Which of the following number is rational ?
(a) sin15° (b) cos15° (c) sin15° ⋅ cos15° (d) sin15° ⋅ cos 75°
1 − cos α 1 + cos α
267. If angle α is in third quadrant, then + =
1 + cos α 1 − cos α
(a) 2cosecα (b) −2 cosecα (c) cosec α (d) −cosecα
268. If cot (α + β ) = 0, (where α , β ∈ 1st quadrant), then sin (α + 2β ) =
(a) − sin α (b) sin β (c) cos α (d) cos β
269. If cosec θ − cot θ = p , then cosec θ =
1 1 1 1 1 1
(a) θ + (b) θ − (c)  p+  (d)  p− 
p p 2 p 2 p
270. If sec θ − tan θ = x, (Where x ∈ ( 0, 1) and θ lies in 1st quadrant) then sin θ =
1 + x2 x2 −1 1
2 1 − x2
(a) (b) 2 (c) x + 2 (d)
1 − x2 x +1 x 1 + x2
271. cos105° + sin105° =
(a) sin 30° (b) cos 45° (c) sin 60° (d) cos 90°

Office.: 606 , 6th Floor, Hariom Tower, Circular Road, Ranchi-1, Ph.: 0651-2562523, 9835508812, 8507613968 21
22 BY R. K. MALIK’S NEWTON CLASSES
( )
3  A  A  5A 
272. If cos A = , then 16 cos 2   − 32sin   ⋅ sin  =
4 2 2  2 
(a) −4 (b) −3 (c) 3 (d) 4
1
273. If sin (α + β ) = 1 and sin (α − β ) = , (where α and β are acute angles), then
2
tan (α + 2β ) ⋅ tan ( 2α + β ) =
(a) −1 (b) 0 (c) 1 (d) 2
274. If cos 25° + sin 25° = p, then cos 50° =

(a) 2 − p2 (b) − 2 − p 2 (c) p ⋅ 2 − p 2 (d) − p ⋅ 2 − p 2


275. Which of the following is correct ?
(a) sin1° > sin1c (b) sin1° < sin1c (c) sin1° = sin1c (d) sin1° = (π /180 ) sin1c

1 − sin x 1 + sin x
276. If x is in the second quadrant, then + =
1 + sin x 1 − sin x
(a) 2sec x (b) −2sec x (c) 2 cosec x (d) −2 cosec x
π  π  π  π 
277. sin  + A  ⋅ cos  + B  − cos  + A  ⋅ sin  + B  = ...
3  3  3  3 
(a) cos ( A − B ) (b) sin ( A − B ) (c) cos ( A + B ) (d) None of these
π π
278. If 0 < A < and 0 < B < , then angle ( A − B ) lies surely in ….. quadrant
2 2
(a) First (b) Second (c) Third (d) None of these
279. If sin A = 3 / 5 and cos B = 9 / 41, (where A, B are both lie in the first quadrant), then sin ( A − B ) = ...
(a) −133 / 205 (b) −84 / 205 (c) 124 / 205 (d) None of these \
280. 1 + tan A ⋅ tan ( A / 2 ) = ....
(a) sec A (b) tan A (c) cosec A (d) cot A
sin ( A − B ) sin ( B − C ) sin ( C − A )
281. For any angle A, B, C , + + =
cos A ⋅ cos B cos B ⋅ cos C cos C ⋅ cos A
(a) 0 (b) sin ( A − B − C ) (c) tan ( A − B − C ) (d) None of these
282. If 2sin ( x + 60° ) = cos ( x − 30° ) , then tan x = …….

(a) 3 (c) 2 (c) − 2 (d) − 3


283. If sin (θ + φ ) = 2 ⋅ sin (θ − φ ) , then tan θ = K ⋅ tan φ , where K = ...
(a) 1 (b) 2 (c) 3 (d) 4
cos (θ − φ )
284. If 3 tan θ ⋅ tan φ = 1, then = ...
cos (θ + φ )
(a) 1 (b) 2 (c) 3 (d) 4
cot 54° tan 20°
285. + = ...
tan 36° cot 70°
(a) 1 (b) 2 (c) 0 (d) 3
π  π 
286. cos 2  − θ  + cos 2  + θ  = ...
4  4 

th
22 Office.: 606 , 6 Floor, Hariom Tower, Circular Road, Ranchi-1, Ph.: 0651-2562523, 9835508812, 8507613968
BY R. K. MALIK’S NEWTON CLASSES
( ) 23
(a) 1 (b) 2 (c) 3 (d) None of these
π 3π 5π 7π 9π
287. cot ⋅ cot ⋅ cot ⋅ cot ⋅ cot = ...
20 20 20 20 20
(a) −1 (b) 0 (c) 1 (d) None of these
288. sin 2 5° + sin 2 10° + sin 2 15° + ..... + sin 2 90° = ...
1 1 1 1
(a) 6 (b) 7 (c) 8 (d) 9
2 2 2 2
289. If sec θ = −13 / 5, where 90° < θ < 180°, then sin 2θ = ...
(a) 102 /191 (b) 120 / 199 (c) −120 / 169 (d) −119 / 169
290. If cos θ = 3 / 7, and θ lies in the fourth quadrant, then cos (θ / 2 ) = ...

(a) (2 / 7) (b) − ( 2 / 5) (c) − (5 / 7 ) (d) None of these

π  π 
291. For any angle A, 4 sin A ⋅ sin  − A  ⋅ sin  + A  = ...
3  3 
(a) sin 2 A (b) sin 3 A (c) sin 4 A (d) None of these
292. cos 20° ⋅ cos 40° ⋅ cos 60° ⋅ cos80° = ……
(a) 1 / 2 (b) 1/ 22 (c) 1/ 23 (d) 1/ 24
293. tan 20° ⋅ tan 40° ⋅ tan 60° ⋅ tan 80° = ...
(a) 1 (b) 2 (c) 3 (d) 4
2sin 2α − 3cos 2α
294. If tan α = 3 , then = …..
4sin 2α + 5 cos 2α
(a) 1 / 3 (b) 4 / 9 (c) −9 / 4 (d) −4 / 9
π 3π 5π 7π
295. cos 2 + cos 2 + cos 2 + cos 2 = ...
8 8 8 8
(a) 1 (b) 2 (c) 3 (d) 4
296. cos 40° + cos80° + cos160° + cos 240° = ...
(a) −2 (b) −1/ 2 (c) − 3 (d) −1/ 3
sin 70° + cos 40°
297. = ...
cos 70° + sin 40°
(a) 2 (b) 1/ 2 (c) 3 (d) 1 / 3
α + β  α − β 
298. If 3sin α = 5sin β , then tan   ÷ tan   = ...
 2   2 
(a) 1 (b) 2 (c) 3 (d) 4
299. cos 52° + cos 68° + cos172° = ...
(a) −1 (b) 0 (c) 1 (d) None of these
4 2 2 2
sin α + sin α ⋅ cos α + cos α
300. If α is any real number, then = ..
cos 4 α + sin 2 α ⋅ cos 2 α + sin 2 α
(a) 1 (b) 2 (c) 3 (d) None of these
301. If α is any real number, then ( sin α + cosecα ) + ( cos α + sec α ) − ( tan 2 α + cot 2 α ) = ...
2 2

(a) 3 (b) −5 (c) 7 (d) 2 sin 2 α + 3cos 2 α


302. sin163° ⋅ cos 347° + sin 73° ⋅ sin167° = ....
(a) 2 (b) 1 / 2 (c) 3 (d) 1 / 3

Office.: 606 , 6th Floor, Hariom Tower, Circular Road, Ranchi-1, Ph.: 0651-2562523, 9835508812, 8507613968 23
24 BY R. K. MALIK’S NEWTON CLASSES
( )
303. If sin a + sin b + sin 99 c = 3, (where a, b, c are acute angles) then cos100 a + cos100 b + cos100 c = ...
99 99

(a) −3 (b) −2 (c) −1 (d) 0


sin 2 A − sin 2 B
304. = ……
sin A ⋅ cos A − sin B ⋅ cos B
(a) sin ( A − B ) (b) cos ( A + B ) (c) tan ( A + B ) None of these
 π
305. If sin ( A + B + C ) = 1, tan ( A − B ) = 1/ 3 and sec ( A + C ) = 2, (where, A, B, C ∈ 0,  ) then
 2
(a) A = 90°, B = 60°, C = 30° (b) A = 120°, B = 60°, C = 0°
(c) A = 60°, B = 30°, C = 0° (d) None of these
306. If ∆ABC is a right-angled at A then cos 2 B + cos 2 C =
(a) −2 (b) −1 (c) 1 (d) 0
π ∞ ∞ ∞
307. For 0 ≤ θ ≤ and x = ∑ cos 2 nθ , y = ∑ sin 2 n θ , and z = ∑ cos 2 n θ ⋅ sin 2 n θ
2 n=0 n=0 n=0

(a) xyz = xz + y (b) xyz = yz + x (c) xyz = xy + z (d) x + y + z = xyz


308. The circular wire of diameter 10 cm is cut and placed along the circumference of a circle of diameter 1
m. The angle subtended by the wire at the centre of the circle is equal to
π π π π
(a) rad (b) rad (c) rad (d) rad
4 3 5 10
309. The maximum value of 3cos θ + 4sin θ is
(a) 3 (b) 4 (c) 5 (d) None of these
310. If sin θ + cos θ = m and sec θ + cosecθ = n, then n ( m + 1)( m − 1) is equal to
(a) m (b) n (c) 2m (d) 2n
311. What is the value of sin 420°⋅ cos 390° + cos ( −300° ) ⋅ sin ( −330° ) ?
(a) 0 (b) 1 (c) 2 (d) −1
312. The value of cot ( 45° + θ ) cot ( 45° − θ ) is
(a) −1 (b) 0 (c) 2 (d) −1
313. The value of sin A sin ( 60° − A ) sin ( 60° + A ) is equal to
sin 3 A sin 3 A
(a) sin 3 A (b) (c) (d) None of these
2 4
314. What is the value of sin A cos A tan A + cos A sin A cot A ?
(a) sin A (b) cos A (c) tan A (d) 1
315. What is the maximum value of 3cos x + 4 sin x + 5 ?
(a) 5 (b) 7 (c) 10 (d) 12
sin x 1 + cos x
316. What is the value of + ?
1 + cos x sin x
(a) 2 tan x (b) 2 cosec x (c) 2 cos x (d) 2 sin x
317. What is the maximum value of sin 3θ cos 2θ + cos 3θ sin 2θ ?
(a) 1 (b) 2 (c) 4 (d) 10
cos12° − sin12° sin147°
318. The value of + is equal to
cos12° + sin12° cos147°
(a) 1 (b) −1 (c) 0 (d) None of these

th
24 Office.: 606 , 6 Floor, Hariom Tower, Circular Road, Ranchi-1, Ph.: 0651-2562523, 9835508812, 8507613968
( BY R. K. MALIK’S NEWTON CLASSES
) 25
1
319. One of the angles of a triangle is rad and the other is 99° The third angle in radian measure is ….
2
9π − 10 90π − 100 90π − 10
(a) (b) (c) (d) None of these
π 7π π
π
320. If y = sec2 θ + cos 2 θ , where 0 < θ < , then which one of the following is correct ?
2
(a) y = 0 (b) 0 ≤ y ≤ 2 (c) y ≥ 2 (d) None of these
3 12
321. If tan A = and tan B = − , then how many values can cot ( A − B ) have depending on the actual
4 5
values of A and B ?
(a) 1 (b) 2 (c) 3 (d) 4
sin θ + cos θ − tan θ 3π
322. What is the value of , when θ = ?
sec θ + cosecθ − cot θ 4
(a) 0 (b) 1 (c) −1 (d) None of these
323. Which one of the following is correct ?
π
(a) sin1° > sin1 (b) sin1° < sin1 (c) sin1° = sin1 (d) sin1° = sin1
180
cos15° sin15° cos 45° cos15°
324. What is the value of ×
cos 45° sin 45° sin 45° sin15°
1 2 1 3
(a) (b) (c) − (d) −
4 2 4 4
325. The angle A lies in the third quadrant and it satisfies the equation 4 ( sin 2 x + cos x ) = 1 What is the
measure of the ∠A ?
(a) 225° (b) 240° (c) 210° (d) None of these
1 1 1 1 
326. If cos θ =  x +  , then  x 2 + 2  , is equal to
2 x 2 x 
(a) sin 2θ (b) cos 2θ (c() tan 2θ (d) scθ
327. The value of x for the maximum values of 3 cos x + sin x, is
(a) 30° (b) 45° (c) 60° (d) 90°
328. If A, B, C , D are the angles of a cyclic quadrilateral, then cos A + cos B + cos C + cos D is equal to
(a) 2 ( cos A + cos C ) (b) 2 ( cos A + cos B ) (c) 2 ( cos A + cos D ) (d) 0
41π 1 − 3 tan 2 A
329. If A = , then what is the value of ?
12 3 tan A − tan 2 A
1
(a) −1 (b) 1 (c) (d) 3
3
330. The length of arc of a circle of radius 5 cm subtending a central angle measuring 15° is ?
5π 7π π π
(a) cm (b) cm (c) cm (d) cm
12 12 12 5
331. What is the value of 1 − sin10° sin 50° sin 70° ?
1 3 5 7
(a) (b) (c) (d)
8 8 8 8

Office.: 606 , 6th Floor, Hariom Tower, Circular Road, Ranchi-1, Ph.: 0651-2562523, 9835508812, 8507613968 25
26 ( BY R. K. MALIK’S NEWTON CLASSES
)
5 99
332. The sines of two angles of a acute angled triangle are equal to and . What is the cosine of the
13 101
third angle ?
255 265 275 770
(a) (b) (c) (d)
1313 1313 1313 1313
333. x = sin θ cos θ and y = sin θ + cos θ are satisfied by which one of the following equations ?
(a) y 2 − 2 x = 1 (b) y 2 + 2 x = 1 (c) y 2 − 2 x = −1 (d) y 2 + 2 x = −1
334. IF sin 4 x − cos 4 x = p, then which one of the following is correct?
(a) p = 1 (b) p = 0 (c) p >1 (d) p ≤ 1
1 1
335. If sin A = and sin B = , where A and B are positive actue angles, then A + B is equal to
10 5
π π π
(a) π (b) (c) (d)
2 3 4
sin1°
336. The value of , where 1c represents 1 rad, is
sin1c

(a) greater than 1 (b) less than 1 (c) equal to 1 (d) equal to π / 180
337. The value of sin θ + sin (θ + 120° ) + sin (θ + 240° ) is equal to

(a) 0 (b) 1 (c) 3 (d) 2


338. The measure of the angle 114°35′30′′ in radian is …………
(a) 1 rad (b) 2 rad (c) 3 rad (d) 4 rad
339. If X = sin ( A + B ) sin ( A − B ) and Y = cos ( A + B ) cos ( A − B ) , then which one of the following is not
correct?
(a) X ′ + Y ′ > 0, if 0° < B < 45° for any A (b) X + Y = 0, if B = 45° for any A
(c) X + Y is a rational number of any A and B (d) X + Y < 0, if 45° < B ≤ 90° for any A
π 
340. What is the value of tan   ?
 12 
(a) 2 − 3 (b) 2 + 3 (c) 2 − 3 (d) 3− 2
341. Which one of the following pairs is not correctly matched ?
(a) sin 2π : sin ( −2π ) (b) tan 45° : tan ( −315° )
(c) cot ( tan −1 0.5 ) : tan ( cot −1 0.5 ) (d) tan 420° : tan ( −60° )

x2 y2 z 2
342. If x = a sec θ cos φ , y = b sec θ sin φ , z = c tan θ , then what is the value of + − ?
a2 b2 c2
(a) 1 (b) 0 (c) −1 (d) a 2 + b 2 − c 2
π
343. If A + B = , The greatest and the least values of cos A cos B, are respectively ……….
2
1 1 −1
(a) and 0 (b) 0 and −1/ 2 (c) and (d) 0 and −1
2 2 2
 1 1
344. Given that p = tan α + tan β and q = cot α + cot β ; then what is the value of  −  ?
 p q
(a) cot (α − β ) (b) tan (α − β ) (c) tan (α + β ) (d) cot (α + β )

th
26 Office.: 606 , 6 Floor, Hariom Tower, Circular Road, Ranchi-1, Ph.: 0651-2562523, 9835508812, 8507613968
( ) BY R. K. MALIK’S NEWTON CLASSES 27

cosec (π + θ ) cot {( 9π / 2 ) − θ } cosec 2 ( 2π − θ )


345. What is the value of
cot ( 2π − θ ) sec2 (π − θ ) sec {( 3π / 2 ) + θ }
(a) 0 (b) 1 (c) −1 (d) ∞
346. The value of sin 20° sin 40° sin 60° sin 80° is equal to
(a) −3 /16 (b) 5 / 16 (c) 3 /16 (d) −5 /16
  3π    π  
347. The value of 3 sin 4  − α  + sin 4 ( 3π + α )  −2 sin 6  + α  + sin 6 ( 5π − α )  is equal to
  2    2  
(a) 0 (b) 1 (c) 3 (d) sin 4α + sin 6α
348. What is the value of tan15° ⋅ tan195° ?
(a) 7 − 4 3 (b) 7 + 4 3 (c) 7 + 2 3 (d) 7 + 6 3

349. If 2 + 2 + 2 + 2 + .......∞ = cosec θ , then the value of sin θ is equal to


1 1 1
(a) 1 (b) (c) (d)
4 2 2
350. Which one of the following statements is correct ?
(a) The squares of the tangents of the angles 30°, 45°, 60° are in GP
(b) The squares of the sines of the angles 30°, 45°, 60° are in GP
(c) The squares of the secants of the angles 30°, 45°, 60° are in AP
(d) The squares of the tangents of the angles 30°, 45°, 60° are in AP
351. Consider the following statements
−1
I. If θ = 1200° the ( sec θ + tan θ ) is positive
II. If θ = 120°, then ( cosec θ -cotθ ) is negative
Which of the statements given above is/are correct ?
(a) Only I (b) Only II (c) Both I and II (d) Neither I hor II
352. Match List I with List II and select the correct answer using the code given below the lists
List I List II
A. tan15° 1. −2 − 3
B. tan 75° 2. 2 + 3
C. tan105° 3. 2 − 3
Codes
A B C A B C A B C A B C
(a) 4 1 2 (b) 4 2 1 (c) 3 2 1 (d) 2 1 4
353. If θ = 18°, then the value of 4 sin 2 θ + 2sin θ is …………
(a) −1 (b) 1 (c) 0 (d) 2
354. The correct sequence of the following values is ………..
π  π  π 
I. sin   II. cos   III. cot  
 12   12   12 
Select the correct answer using the code given below
(a) III > II > I (b) I > II > III (c) I > III > II (d) III > I > II
355. Consider the following statements
I. 1° in radan measure is less than .0.02 radians

Office.: 606 , 6th Floor, Hariom Tower, Circular Road, Ranchi-1, Ph.: 0651-2562523, 9835508812, 8507613968 27
28 BY R. K. MALIK’S NEWTON CLASSES
( )
II. I radian in degree measure is greater than 45°.
Which of the above statements is/are correct?
(a) Only I (b) Only II (c) Both I and II (d) Neither I nor II
356. Let A and B be obtuse angles such that sin A = 4 / 5 and cos B = −12 /13. What is the value of
sin ( A + B ) ?
(a) −63 / 65 (b) −33 / 65 (c) 33 / 65 (d) 63 / 65
357. If a = cos 2 and b = sin 7, then
(a) a > 0, b > 0 (b) ab < 0 (c) a > b (d) a < b

π 3π 1 − sin θ
358. If <θ < , then is equal to
2 2 1 + sin θ
(a) sec θ − tan θ (b) sec θ + tan θ (c) tan θ − sec θ (d) None of these

π y +1 1 + sin θ
359. If 0 < θ < , and if = , then y is equal to
2 1− y 1 − sin θ
θ θ θ θ θ θ
(a) cot (b) tan (c) cot + tan (d) cot − tan
2 2 2 2 2 2

1 − sin α
360. The set of all possible values of α in [ −π , π ] such that is equal to sec α − tan α , is
1 + sin α
(a) [ 0, π / 2 ) (b) [ 0, π / 2 ) ∪ (π / 2, π ] (c) [ −π , 0] (d) ( −π / 2, π / 2 )

361. If a = sin x cos3 x and b = cos x sin 3 x , then


(a) a − b > 0 for x ∈ ( 0, π / 4 ) (b) a − b < 0 for x ∈ ( 0, π / 4 )

(c) a + b < 0 for x ∈ ( 0, π / 2 ) (d) a + b < 0 for x ∈ ( 0, π / 4 )

sin 3 A − cos3 A sin A


362. If A is an obtuse angle, then + − 2 tan A cot A is always equal to
sin A − cos A 1 + tan 2 A
(a) 1 (b) −1 (c) 2 (d) none of these
363. If tan θ = −4 / 3, then sin θ is
(a) −4 / 5 but not 4 / 5 (b) −4 / 5 or 4 / 5 (c) 4 / 5 but not −4 / 5 (d) none of these
sin ( x + y ) a+b tan x
364. ]If = ' then is equal to
sin ( x − y ) a −b tan y

b a
(a) (b) (c) ab (d) none of these
a b
x x
365. If tan = cos ecx − sin x, then the value of tan 2 , is
2 2
(a) 2 − 5 (b) 2 + 5 (c) −2 − 5 (d) −2 + 5
3  A   5A   5A 
366. If cos A = ' then 32sin   sin   sin  =
4 2  2   2 
(a) 7 (b) 8 (c) 11 (d) none of these

th
28 Office.: 606 , 6 Floor, Hariom Tower, Circular Road, Ranchi-1, Ph.: 0651-2562523, 9835508812, 8507613968
(BY R. K. MALIK’S NEWTON CLASSES
) 29
367. If sin 2θ = cos 3θ and θ is an acute angle, then sin θ equals

5 −1  5 −1  5 +1 − 5 −1
(a) (b) −   (c) (d)
4  4  4 4

2π 4π 8π 14π
368. The value of cos cos cos cos is
15 15 15 15
(a) 1 (b) 1/2 (c) 1/4 (d) 1/16
π 2π 3π 4π 5π 6π 7π
369. The value of cos cos cos cos cos cos cos is
15 15 15 15 15 15 15
1 1 1
(a) (b) (c) (d) None of these
26 27 28
370. The value of tan 5θ is
5 tan θ − 10 tan 3 θ + tan 5 θ 5 tan θ + 10 tan 3 θ − tan 5 θ
(a) (b)
1 − 10 tan 2 θ + 5 tan 4 θ 1 − 10 tan 2 θ − 5 tan 4 θ
5 tan 5 θ − 10 tan 3 θ − tan θ
(c) (d) none of these
1 − 10 tan 2 θ + 5 tan 4 θ
3π π α 
371. If π < α < , then the expression 4sin 4 α + sin 2 2α + 4 cos 2  −  is equal to
2 4 2
(a) 2 + 4sin α (b) 2 − 4 sin α (c) 2 (d) none of these

α x −1
372. If α is an acute angle and sin = , then tan α is
2 2x

x −1 x −1
(a) (b) (c) x2 − 1 (d) x2 + 1
x +1 x +1
373. If sin A + cos B = m and sin 3 A + cos 3 A = n, then

(a) m3 − 3m + n = 0 (b) n3 − 3n + 2m = 0 (c) m3 − 3m + 2n = 0 (d) m3 + 3m + 2n = 0



374. If A + B + C = , then cos 2 A + cos 2 B + cos 2C =
2
(a) 1 − 4 cos A cos B cos C (b) 4sin A sin B sin C (c) 1 + 2 cos A cos B cos C (d) 1 − 4sin A sin B sin C
375. The maximum value of sin ( x + π / 6 ) + cos ( x + π / 6 ) in the interval ( 0, π / 2 ) is attained at

(a) π /12 (b) π / 6 (c) π / 3 (d) π / 2

1 + cos θ
376. If π < θ < 2π , then is equal to
1 − cos θ
(a) cos ecθ + cot θ (b) cos ec θ − cot θ (c) − cos ec θ + cot θ (d) − cos ec θ − cot θ

π 1 − sin θ 1 + sin θ
377. If < θ < π , then + is equal to
2 1 + sin θ 1 − sin θ
(a) 2sec θ (b) −2sec θ (c) sec θ (d) − sec θ

Office.: 606 , 6th Floor, Hariom Tower, Circular Road, Ranchi-1, Ph.: 0651-2562523, 9835508812, 8507613968 29
30 ( ) BY R. K. MALIK’S NEWTON CLASSES
tan α + tan β
+ {cos (α − β ) sec (α + β ) + 1} = 1. then tan α tan β is equal to
−1
378. If
cot α + cot β
(a) 1 (b) −1 (c) 2 (d) −2
tan 70° − tan 20°
379. The value of =
tan 50°
(a) 2 (b) 1 (c) 0 (d) 3
1
380. The minimum value of ' is
3sin θ − 4 cos θ + 7
1 5 7 1
(a) (b) (c) (d)
12 12 12 6
 cos x 1 − sin x 
381. The maximum value of cos x  +  ' is
1 − sin x cos x 
(a) 1 (b) 3 (c) 2 (d) 4

b a+b a −b  π
382. If tan x = . then + = where x =  0, 
a a −b a+b  4
2 sin x 2 cos x 2 cos x 2 sin x
(a) (b) (c) (d)
sin 2 x cos 2 x sin 2 x cos 2 x
383. Let 0 < x ≤ π / 4, then ( sec 2 x − tan 2 x ) equals

(a) tan 2 ( x + π / 4 ) (b) tan ( x + π / 4 ) (c) tan (π / 4 − x ) (d) tan ( x − π / 4 )

cos θ sin θ a b
384. If = , then + is equal to
a b sec 2θ cos ec 2θ

a
(a) a (b) b (c) (d) a + b
b
385. If α + β − γ = π , then sin 2 α + sin 2 β − sin 2 γ is equal to
(a) 2 sin α sin β sin γ (b) 2 cos α cos β cos γ (c) 2 sin α sin β sin γ (d) none of these

 απ   βπ 
386. If tan   = cot   , then
 4   4 
(a) α + β = 0 (b) α + β = 2n (c) α + β = 2n + 1 (d) α + β = 2 ( 2n + 1) , n ∈ Z .

sin A sin A
387. If = n and = m, then ( m 2 − n 2 ) sin 2 B =
cos B cos B
(a) 1 − n 2 (b) 1 + n 2 (c) 1 − n (d) 1 + n
388. If cos (θ + φ ) = m cos (θ − φ ) , then tan θ is equal to

1+ m 1− m 1− m 1+ m
(a) tan φ (b) tan φ (c) cot φ (d) sec φ
1− m 1+ m 1+ m 1− m
cos x sin x
389. If = a and = b, then : ( a 2 − b 2 ) ⋅ sin 2 y = ...
cos y sin y

th
30 Office.: 606 , 6 Floor, Hariom Tower, Circular Road, Ranchi-1, Ph.: 0651-2562523, 9835508812, 8507613968
( ) BY R. K. MALIK’S NEWTON CLASSES 31
(a) a 2 − 1 (b) b 2 − 1 (c) x 2 − 1 (d) y 2 − 1

390. If π < 3θ < , then 2 + 2 + 2 cos 4θ is equal to
2
(a) −2 cos θ (b) −2sin θ (c) 2 cos θ (d) 2sin θ
3cos θ + cos 3θ
391. is equal to
3sin θ − sin 3θ
(a) 1 + cot 2 θ (b) cot 4 θ (c) cot 3 θ (d) 2 cot θ
3π 4π
392. cos 2 + cos 2 is equal to
5 5
4 5 5 3
(a) (b) (c) (d)
5 2 4 4
α +β
tan
393. If 3sin α = 5sin β , then 2 is equal to
α −β
tan
2
(a) 1 (b) 2 (c) 3 (d) 4
1  π
Directions (Q. Now. 394-396) Let sin ( A + B ) = 1 and sin ( A − B ) = , where A, B ∈ 0, 
2  2
394. What is the value of A?
π π π π
(a) (b) (c) (d)
6 3 4 8
395. What is the value of tan ( A + 2 B ) tan ( 2 A + B ) ?
(a) −1 (b) 0 (c) 1 (d) 2
396. What is the value of sin 2 A − sin 2 B ?
1
(a) 0 (b) (c) 1 (d) 2
2

Office.: 606 , 6th Floor, Hariom Tower, Circular Road, Ranchi-1, Ph.: 0651-2562523, 9835508812, 8507613968 31
32 ( ) BY R. K. MALIK’S NEWTON CLASSES

th
32 Office.: 606 , 6 Floor, Hariom Tower, Circular Road, Ranchi-1, Ph.: 0651-2562523, 9835508812, 8507613968
JEE (MAIN & ADV.), MEDICAL + BOARD, NDA, X & IX
Enjoys unparalleled reputation for best results
in terms of percentage selection
www.newtonclasses.net

SOLUTION OF TRIGONOMETRIC FUNCTION ( )


Only one option is correct.
o o o
180   180 × 7   630 
1. Ans. (a), ∵ π radians = 180° , ∴ 1 radian =   =  =  ≈ 57°17′ 45′′
 π   22   11 
3
2. Ans. (c), Given, radian is the angle subtended at the centre by arc of length 15 cm
4
 15 
∴ 2π radians is the angle subtended by arc of length =  × 2π  cm = 40π cm
 3/ 4 
∴ Circumference of the circle = 40π cm ⇒ 2π r = 40π where r is the radius of the circle
 40π 
⇒ r =  cm = 20 cm
 2π 
π rθ
3. Ans. (c), Length of the arc l = [Where θ is in degrees]
180
 22 1 
=  × 28 × 45 ×  cm = 22 cm
 7 360 
4. Ans. (c), Let θ be the angle subtended at the centre.
50
∴ Radius of the circle r = cm = 25 cm, Length of the arc l = 11 cm
2
o o o
π rθ l × 180   11× 180 × 7   126 
Now, l = , where θ is in degrees ⇒ θ =   =  =  = 25°12′
180  π r   22 × 25   5 
5. Ans. (a), At the time 4 : 20 , the minutes hand is exactly at 4. The hour hand is at 4 at time 4 : 00.
c

The hour hand traces an angle of ( 2π ) in 12 hours, i.e. it traces an angle   in 1 hour, i.e., in 60
c

 12 
c
2π 20 π
min. So, the angle traced by the hour hand in 20 min =  ×  = radians.
 12 60  18
π
∴ The angle between the minute hand and the hour hand at time 4 : 20 is radians.
18
π π π
6. Ans. (d), Let the three angles of the triangle be , + θ and + 2θ .
4 4 4
π π  π 
Then, +  + θ  +  + 2θ  = π [∵ Sum of angles of a triangle = π ]
4 4  4 
3π π π π  π  5π
⇒ + 3θ = π ⇒ 3θ = ⇒θ = ∴ The largest angle = + 2   = .
4 4 12 4  12  12
7. Ans. (b), Length of the wire = circumference of a circle of radius 15 cm = 2π × 15 = 30π cm
∴ Length of the arc formed by the wire when placed on the loop = 30 π cm
Now, the length of arc of a circle of radius r , subtending an angle θ at the circle is given by
π rθ
S= where θ is in degrees θ
180
12
0c

( 2π ) ×120 × θ
m

⇒ 30π = ⇒ θ = 45° O
360
th
Office.: 606 , 6 Floor, Hariom Tower, Circular Road, Ranchi-1, Ph.: 0651-2562523, 9835508812, 8507613968
2 ( BY R. K. MALIK’S NEWTON CLASSES
)
∴ Angle subtended at the circle = 45°
8. Ans. (b), Let the polygon have n sides. Then, the sum of its interior angles
= Sum to n terms of an A.P. with first term 120° and common difference 5°
o o
n   5 
=  {( 2 × 120 ) + ( n − 1) ⋅ 5} = 120n + n ( n − 1) 
2   2 
Also, sum of the interior angles = ( 2n − 4 ) × 90°
[∵ Sum of the interior angles of a polygon of n sides = ( 2n − 4 ) × 90° ]
5
∴ ( 2n − 4 ) × 90 = 120n + n ( n − 1) ⇒ 360n − 720 = 240n + 5n 2 − 5n
2
⇒ 5n 2 − 125n + 720 = 0 ⇒ n 2 − 25n + 144 = 0 ⇒ ( n − 16 )( n − 9 ) = 0 ⇒ n = 9 or n = 16
But when n = 16 , then the largest angle is 195° which is more than 180° . Hence, n ≠ 16
∴ The polygon has 9 sides.
9. Ans. (c), The length ‘ l ’ of the arc of a circle of radius r , subtending an angle
132 m
2π rθ
θ at the centre is given by, l = where θ is in degrees
360 108°
22 108 132 × 7 × 360
⇒ 132 = 2 × × r × ⇒r= = 70 m.
7 360 2 × 22 × 108
c o
 π   180 π 
10. Ans. (a), We have m∠A = 42° and m∠B =   =  ×  = 30°
6  π 6
But, in ∆ABC , m∠A + m∠B + m∠C = 180°
c c
 π   3π 
∴ 42° + 30° + m∠C = 180° ∴ m∠C = 108° =  × 108  =  
 180   5 
c
 3π 
Thus, m∠C = 108° =  
 5 
θ1 4 C
11. Ans. (b), We are given = …(i) Y
θ2 5 X
1 2 θ1 θ 2
rθ A B
Area of sector OAXC 2 1 θ1 4 r O r
∴ = = = [Using (i)]
Area of sector OBYC 1 r 2θ θ2 5
2
2
Clearly, ratio of areas = ratio of angles = 4 : 5
12. Ans. (c), From the figure OA = 18 cm and OA′ = 36 cm Y
∴ AA′ = OA′ − OA = 36 − 18 = 18 cm similarly, BB′ = 18 cm B′
80°
∴ AA′ = BB′ = 18 cm B C • • C′
c c 4π/9
 π   4π  X
Then, θ = m∠AOB = m∠A′OB′ = 80° =  × 80  =  
O A A′
 180   9 

∴ Length of ( arc ACB ) = l ( arc A′C ′B′ ) = ( OA )(θ ) = 18 × = 8π cm
9

and length of ( arc A′C ′B′ ) = l ( arc A′C ′B′ ) = ( OA′ )(θ ) = 36 × = 16π cm
9
∴ Perimeter of required region AA′C ′B′BCA = AA′ + BB′ + l ( ACB ) + l ( A′C ′B′ )

Office.: 606 , 6th Floor, Hariom Tower, Circular Road, Ranchi-1, Ph.: 0651-2562523, 9835508812, 8507613968
( ) BY R. K. MALIK’S NEWTON CLASSES 3
= 18 + 18 + 8π + 16π = 36 + 24π = 6 ( 6 + 4π ) cm
1 2 1 2 4π
Further, Area of ( sector OACB ) = A ( sector OACB ) = ( OA ) ⋅θ = × (18 ) × = 72π cm 2
2 2 9
1 2 1 2 4π
Area of ( sector OA′C ′B′ ) = A ( sector OA′C ′B′ ) = ( OA′ ) ⋅ θ = × ( 36 ) × = 216 π cm 2
2 2 9
∴ Area of shaded region = A ( OA′C ′B′ ) − A ( OACB ) = 288π − 72π = 216 cm 2
13. Ans. (a), When the time is 2 : 20 : 00 (twenty minutes past two), the minute-hand is exactly at mark 4
and the hour-hand has crossed one-third of the angle between mark 2 and mark 3
Now, the hour-hand moves through one complete rotation ( 360° ) in 12 hours
12
∴ In 1 hour, the hour-hand moves through ( 360 / 12 ) ° 11 1
∴ In 60 minutes, the hour-hand moves through 30° 10 2
o HH
 20  9 3
∴ In 20 minutes, the hour-hand moves through  × 30  , i.e. 10°
 30 

M
H
8 4
∴ angle between hour-hand and mark 3 is 30° − 10° = 20°
7 5
Also, angle between mark 3 and mark 4 is 30° 6
∴ Angle between the hour-hand and minute-hand at 2 : 20 is 20° + 30° = 50°
14. Ans. (b), Angle between the hands at 3 is 90° . In 20 minutes, minute hand traces an angle
o
1
6°× 20 = 120° , and hour hand traces an angle × 20 = 10°
2
π
∴ Angle between the hands = 120° − 90° − 10° = 20° =
9
15. Ans. (b), Let the angles be A, B and C ∴ Given information ⇒ A : B : C = 1: 2 : 3
Therefore, A = k , B = 2k , C = 3k and A + B + C = π ⇒ k + 2k + 3k = π
π π π π π
⇒ k= ∴ Angles are , and , Clearly the smallest angle is
6 6 3 2 6
16. Ans. (d), Perimeter of the circle of radius 5 cm is 2π .5 = 10π cm = Arc length of the new circle (10π
cm)
Given, radius r of the new circle = 10 cm. Now we know that s = r θ ⇒ 10π = 10θ ⇒ θ =π

17. Ans. (c), Given, length of wire is 40π cm, we know that s = r θ ⇒ 40π = (1× 100 ) θ ⇒ 40 π = 100θ


⇒θ =
5
18. Ans. (c), As area of a circle is π r 2 ∴ 9π = π r 2 ⇒ r = 3 , Hence, perimeter of the sector

π π 
= 2r + rθ = 2 × 3 + 3 × 60°× = 6 + 3  = 6 + π
180° 3
19. Ans. (b), As three terms in A.P. are a − d , a, a + d
So, angles be a − d , a and a + d , As we know that sum of the three angles of a triangle = 180°
a
⇒ a − d + a + a + d = 180° ⇒ a = 60° , But given a + d = 2 ( a − d ) ⇒d= ⇒ d = 20°
3

Office.: 606 , 6th Floor, Hariom Tower, Circular Road, Ranchi-1, Ph.: 0651-2562523, 9835508812, 8507613968
4 ( BY R. K. MALIK’S NEWTON CLASSES
)
∴ The angles are 60° − 20°, 60°, 60° + 20° = 40°, 60°, 80° , Hence, the option (b) is correct.
Alternatively :
If angles of a triangle are in A.P., then one of the angles must be 60° .
∴ Option (c) is not a solution
Option (d) is not a solution because their sum is not equal to 180° .
In option (a), the angles are in A.P, but greatest angle ≠ double the least angle.
∴ Option (b) is the required solution.
20. Ans. (c), The wheel makes 120 revolutions in 60 seconds.
∴ In 1 second wheel makes 2 revolutions, and in 2 seconds it makes 4 revolutions.
Also in one revolution second wheel max 2π radian, ∴ Angle covered in 2 seconds = 3 × 2π = 8π
21. Ans. (b), Let the two polygons A and B have 3 x and 4 x sides respectively.
2 × 3x − 4 6x − 4
An interior angle of polygon A = × 90° = × 90°
3x 3x
2 × 4x − 4 8x − 4
and an interior angle of polygon B = × 90° = × 90°
4x 4x
 8x − 4   6x − 4 
∴  × 90°  −  × 90°  = 15°
 4x   3x 
[Note : The polygon with larger no. of sides has larger interior angle]
 8 x − 4 6 x − 4  15° 24 x − 12 − 24 x + 16 1 4 1
⇒ − = ⇒ = ⇒ = ⇒ x = 2.
 4x 3 x  90° 12 x 6 12 x 6
∴ The two polygons have 6 and 8 sides respectively.
∴ The polygon with larger interior angle has 8 sides.
 π
 Note that sum of all interior angles of any polygon of n sides = ( 2n − 4 ) 2 
π
22. Ans. (a), Since − < A < 0, we have sin A < 0 i.e., sin A is − ve
2
π
Since − < B < 0 , we have sin B < 0 i.e., sin B is − ve
2
cos A cos B 1 3 4
Now, = = ⇒ cos A = and cos B = .
3 4 5 5 5
9 4 16 3
∴ sin A = − 1 − cos 2 A = − 1 − = − and sin B = − 1 − cos 2 B = − 1 − =−
25 5 25 5
 4  3
∴ 2 sin A + 4sin B = 2  −  + 4  −  = −4.
 5  5
sin θ + cos θ tan θ + 1
23. Ans. (b), ∵ = [Dividing Nr. and Dr. by cos θ ]
sin θ − cos θ tan θ − 1
p
+1
q p+q
= =
−1 p − q
p
q

Office.: 606 , 6th Floor, Hariom Tower, Circular Road, Ranchi-1, Ph.: 0651-2562523, 9835508812, 8507613968
( ) BY R. K. MALIK’S NEWTON CLASSES 5
cos 9° + sin 9° 1 + tan 9°
24. Ans. (c), = [Dividing Nr. And Dr. by cos 9° ]
cos 9° − sin 9° 1 − tan 9°
tan 45° + tan 9°  tan A + tan B 
= = tan ( 45° + 9° ) = tan 54°. [∵ tan 45° = 1] and ∵ tan ( A + B ) =
1 − tan 45°. tan 9°  1 − tan A ⋅ tan B 
25. Ans. (d), Given, cos θ − sin θ = 1 ⇒ cos θ = 1 + sin θ
∵ When 0 < θ ≤ π , then sin θ is + ve and so 1 + sin θ ≥ 1
Clearly, sin θ = 0  But since cos θ ≤ 1, so we have sin θ = 0 
 
But for θ = π cos θ = −1 , Hence no solution
26. Ans. (b), cos1°.cos 2°........cos100° = cos1°.cos 2°........cos89°.cos 90°.cos 91°.....cos100°
=0 [∵ cos 90° = 0]
2
27. Ans. (d), p = cos 2 θ + sec2 θ = ( cos θ − sec θ ) + 2 cos θ sec θ

[∵ cos θ ⋅ secθ = 1]
2
= ( cos θ − sec θ ) + 2 ≥ 2 , ∴ p ≥ 2
28. Ans. (d), cos ( 270° + θ ) cos ( 90° − θ ) − sin ( 270° − θ ) cos θ
= sin θ .sin θ − ( − cos θ ) cos θ = sin 2 θ + cos 2 θ = 1
29. Ans. (b), cos1°.cos 2°....cos179° = cos1°. cos 2°......cos89°. cos 90°. cos 91°.......cos179°
=0 [∵ cos 90° = 0]
tan A 1 + sec A tan 2 A + 1 + sec2 A + 2 sec A 2sec 2 A + 2sec A
30. Ans. (a), + = =
1 + sec A tan A tan A (1 + sec A ) tan A (1 + sec A )
2sec A (1 + sec A ) 2sec A
= = = 2 cosec A
tan A (1 + sec A ) tan A

Ans. (c), ∵ cos 4 θ − sin 4 θ = ( cos 2 θ ) − ( sin 2 θ )


2 2
31.

{ }
= ( cos 2 θ − sin 2 θ )( cos 2 θ + sin 2 θ ) = ( cos 2 θ − sin 2 θ ) = cos 2 θ − (1 − cos 2 θ ) = 2 cos 2 θ − 1

 π   5π  1
32. Ans. (d), 2 sin   sin   = 2 sin15° sin 75° = 2sin15° cos15° = sin 30° =
 12   12  2
33. Ans. (a), Since α is a root of 25cos 2 θ + 5cos θ − 12 = 0 , so we have 25cos 2 α + 5cos α − 12 = 0
−5 ± 25 + 1200 −5 ± 35 4  π 
⇒ cos α = = ⇒ cos α = − ∵ 2 < α < π ⇒ cos α is -ve 
50 50 5
16 3  π 
∴ sin α = 1 − = ∵ 2 < α < π ⇒ sin α is +ve 
25 25
3  4 24
and so, sin 2α = 2 sin α cos α = 2 × ×  −  = − .
5  5 25
1 1
34. Ans. (d), sin ( x − y ) = ⇒ x − y = 30° or150° and cos ( x + y ) = ⇒ x + y = 60° or 300°
2 2
Now, we have
(i) When x − y = 30°, x + y = 60°, then x = 45°, y = 15°
(ii) When x − y = 150°, x + y = 60°, then x = 105°, y = −45°
(iii) When x − y = 30°, x + y = 300°, then x = 165°, y = 135°

Office.: 606 , 6th Floor, Hariom Tower, Circular Road, Ranchi-1, Ph.: 0651-2562523, 9835508812, 8507613968
6 BY R. K. MALIK’S NEWTON CLASSES
( )
(iv) When x − y = 150°, x + y = 300°, then x = 225°, y = 75°
Since x and y lie between 0° and180° we have, ( x = 45°, y = 15° ) or ( x = 165°, y = 135° )
Out of the given alternatives we have x = 45°, y = 15°. So, option (d) is correct
1 − sin θ 1 + sin θ  1 − sin θ   1 − sin θ   1 + sin θ   1 + sin θ 
35. Ans. (b), + =  × +   
1 + sin θ 1 − sin θ  1 + sin θ   1 − sin θ   1 − sin θ   1 + sin θ 

=
(1 − sin θ ) + (1 + sin θ ) =
2
= 2 sec θ
1 − sin θ 2 cos θ
But since θ lies in the second quadrant, sec θ is negative and so sec θ = − sec θ

1 − sin θ 1 + sin θ
∴ + = −2sec θ
1 + sin θ 1 − sin θ

( 3)
2
1 + tan 2 θ 1 + ( tan 60° ) 1 +
2
4 2
36. Ans. (b), = = = = .
2 tan θ 2 ( tan 60° ) 2 ( 3) 2 3 3

Alternatively :
1 + tan 2 θ sec2 θ 1 1 1 1 1 2
= = = = = = =
2 tan θ 2 tan θ 2 sin θ cos θ sin 2θ sin120° sin (180° − 60° ) sin 60° 3

37. Ans. (a), Given, sec θ = m and tan θ = n ∴


1
=
1
=
1
.
( sec θ − tan θ )
m + n ( sec θ + tan θ ) ( sec θ + tan θ ) ( sec θ − tan θ )

=
( sec θ − tan θ ) = ( sec θ − tan θ ) ∵ sec 2 θ − tan 2 θ = 1
( sec2 θ − tan 2 θ )
1  1  1 1
Now, ( m + n ) +  = {( secθ + tan θ ) + ( secθ − tan θ )} = .2 sec θ = 2.
m  ( m + n )  secθ sec θ
1 + sin θ − cos θ sin θ + (1 − cos θ )
38. Ans. (c) =
1 + sin θ + cos θ sin θ + (1 + cos θ )
θ θ θ θ θ θ
2sin cos + 2 sin 2 2sin  sin + cos 
2 2 2 2 2 2 θ
= = = tan .
θ θ 2 θ θ θ θ 2
2sin cos + 2 cos 2 cos  sin + cos 
2 2 2 2 2 2
39. Ans. (c), Given, x = sin A + cos A = sin130° + cos130° = sin (180° − 50° ) + cos ( 90° + 40° )
= sin 50° − sin 40° > 0 [∵ sin 50° > sin 40°]
[Note: In the 1st quadrant sin θ is always increasing]
3 3 1 1  π 
40. Ans. (b), Given, cos α = − ⇒ sin α = 1 − = = ∵ sin α is +ve when 2 < α < π 
2 4 4 2
sin α 1/ 2 1 1
∴ tan α = = =− and cot α = =− 3
cos α − 3 / 2 3 tan α
3 9 16 4  3π 
and sin β = − ⇒ cos β = − 1 − =− =− ∵ cos β is -ve when π < β < 2 
5 25 25 5

Office.: 606 , 6th Floor, Hariom Tower, Circular Road, Ranchi-1, Ph.: 0651-2562523, 9835508812, 8507613968
( BY R. K. MALIK’S NEWTON CLASSES
) 7
 1  3 3
3.  −  + 2.   −1 +
sin β −3 / 5 3 3 tan α + 2 tan β  3 4
  2= 5 .
and tan β = = = ∴ = =
cos β −4 / 5 4 2
cot α + cos β  4 4 22
( )
2
− 3 +−  3−
 5 5
1 1 8
41. Ans. (b), Given, tan θ = ⇒ sec 2 θ = 1 + tan 2 θ = 1 + = .
7 7 7
8
2 2 8−
cosec θ − sec θ 7 = 48 = 3
Also, cos θ = 7 ⇒ cosec 2θ = 1 + cot 2 θ = 1 + 7 = 8 , ∴ =
cosec θ + sec θ 8 + 8 64 4
2 2

7
Alternatively :
1 1

cosec θ − sec θ sin θ cos 2 θ cos 2 θ − sin 2 θ
2 2 2 1 − tan 2 θ
= = = [Dividing Nr. and Dr. by cos 2 θ ]
cosec 2θ + sec 2 θ 1 1 cos 2 θ + sin 2 θ 1 + tan 2 θ
2
+ 2
sin θ cos θ
1
1−
= 7 = 6 = 3.
1 8 4
1+
7
4 1 3
42. Ans. (d), 3 tan A + 4 = 0 ⇒ tan A = − , ∴ cot A = =− ,
3 tan A 4
16 5
Also, sec A = − 1 + tan 2 A = − 1 + =− [Note : sec A is −ve since A lies in 2nd quadrant]
9 3
1 3 9 4
cos A = = − and sin A = 1 − cos 2 A = 1 − =
sec A 5 25 5
[Note : sin A is +ve since A lies in 2nd quadrant]
 3  3 4 3 4 23
Now, 2 cot A − 5cos A + sin A = 2  −  − 5  −  + = − + 3 + =
 4  5 5 2 5 10
x 1
+
tan α + tan β x +1 2x +1
43. Ans. (b), ∵ tan (α + β ) = =
1 − tan α tan β  x  1 
1−   
 x +1   2x + 1 
 ( 2 x 2 + x ) + ( x + 1) 
 
 ( x + 1)( 2 x + 1)  2 x 2 + 2 x + 1 π
= = =1 ∴α + β =
 ( 2 x 2 + 3 x + 1) − x  2 x 2 + 2 x + 1 4
 
 (
x + 1)( 2 x + 1) 

cot 54° tan 20° cot ( 90° − 36° ) tan 20° cot ( 90° − 36° ) tan 20°
44. Ans. (c), + = + = +
tan 36° cot 70° tan 36° cot ( 90° − 20° ) tan 36° cot ( 90° − 20° )
tan 36° tan 20°
= + = 1+1 = 2 ∵ cot θ = tan ( 90° − θ ) 
tan 36° tan 20°

Office.: 606 , 6th Floor, Hariom Tower, Circular Road, Ranchi-1, Ph.: 0651-2562523, 9835508812, 8507613968
8 ( BY R. K. MALIK’S NEWTON CLASSES
)
5 1
+
5 1 tan α + tan β
45. Ans. (b), Given, tan α = and tan β = , ∵ tan (α + β ) = = 6 11
6 11 1 − tan α tan β 5 1 
1−  × 
 6 11 
 61 
  π π
66
=   = 1 = tan , ∴α + β =
 61  4 4
 
 66 
tan 70° − tan 20°
46. Ans. (c), ∵ tan 50° = tan ( 70° − 20° ) =
1 + tan 70° tan 20°
tan 70° − tan 20°
⇒ = 1 + tan 70° tan 20° = 1 + tan 70° cot 70° ∵ cot θ = tan ( 90° − θ ) 
tan 50°
= 1+1 = 2 [∵ tan θ .cot θ = 1]
47. Ans. (d),. ∵ tan ( −945° ) = − tan ( 945° ) ∵ tan ( −θ ) = − tan θ 

= − tan (1080° − 135° ) = − tan ( 3 × 360° − 135° ) = tan135° ∵ tan ( 6π − θ ) = − tan θ 

= tan (180° − 45° ) = − tan 45° = −1 ∵ tan (π − θ ) = − tan θ 


48. Ans. (a)
Clearly, the angle 200° lies in the III rd quadrant and so both sin 200° and cos 200° are negative.
∴ sin 200° + cos 200° is negative.
49. Ans. (b), We have: A + C = 180° and B + D = 180°
[∵ The opposite angles of a cyclic quadrilateral are supplementary]
∴ cos A + cos B + cos C + cos D = cos A + cos B + cos (180° − A ) + cos (180° − B )
= cos A + cos B − cos A − cos B = 0.
tan 255° + tan 345° tan ( 270° − 15° ) + tan ( 360° − 15° )
50. Ans. (d), =
tan195° − tan105° tan (180° + 15° ) − tan ( 90° + 15° )
cot15° − tan15° cot 2 15° − 1
= = [Dividing Nr. & Dr. by tan15° ]
tan15° + cot15° 1 + cot 2 15°
m2 − 1
= .
m2 + 1
5
51. Ans. (b), It is given that cosec θ + cot θ = …(i)
2
2  1 
cosecθ − cot θ = …(ii) ∵ cosec θ − cot θ = cosec θ + cot θ 
5
5 2 21 21 20
Subtracting (ii) from (i) we get, 2 cot θ = − = ⇒ cot θ = ⇒ tan θ = .
2 5 10 20 21
52. Ans. (b), cos θ cos ( 90° − θ ) − sin θ sin ( 90° − θ )

= cos θ sin θ − sin θ cos θ = 0 ∵ cos ( 90° − θ ) = sin θ & sin ( 90° − θ ) = cos θ 
1
53. Ans. (c), Given, cos θ = − , Since cos θ is –ve, so θ lies in II or III quadrant.
2

Office.: 606 , 6th Floor, Hariom Tower, Circular Road, Ranchi-1, Ph.: 0651-2562523, 9835508812, 8507613968
( BY R. K. MALIK’S NEWTON CLASSES
) 9
1 1 1
Also, since cos 60° = , so we have, cos (180° − 60° ) = − and cos (180° + 60° ) = −
2 2 2
1 1
i.e., cos120° = − and cos 240° = − i.e., θ = 120° or 240°
2 2

54. Ans. (d), 1 −


sin 2 θ
+
1 + cos θ

sin θ
= 1−
(1 − cos 2 θ ) (1 + cos θ )(1 − cos θ ) − sin 2 θ
+
1 + cos θ sin θ 1 − cos θ (1 + cos θ ) sin θ (1 − cos θ )

= 1−
(1 − cos θ ) + (1 − cos θ ) − sin θ
2 2 2

= 1 − (1 − cos θ ) +
sin 2 θ − sin 2 θ
= cos θ + 0 = cos θ .
(1 + cosθ ) ( sin θ )(1 − cosθ ) ( sin θ )(1 − cos θ )
55. Ans. (b), sin 6 θ + cos 6 θ + 3sin 2 θ cos 2 θ
= sin 6 θ + cos 6 θ + 3sin 2 θ cos 2 θ ( sin 2 θ + cos 2 θ ) ∵ sin 2 θ + cos 2 θ = 1

= ( sin 2 θ + cos 2 θ ) = 13 = 1.
3
∵ sin 2 θ + cos 2 θ = 1
56. Ans. (c), cosec θ − cot θ = q …(i)
1
Taking inverse an both dies we get, cosec θ + cot θ = …(ii)
q
1 1− q2
Subtracting (i) from (ii) we get, 2 cot θ = −q ⇒ cot θ = .
q 2q
1
57. Ans. (b), Given, sec A = 2 ⇒ cos A = ∵ cos A is + ve since A lies in IV th quadrant 
2
1 1 −1
∴sin A = − 1 − =− = ∵ sin A is − ve when A lies in IV th quadrant 
2 2 2
sin A −1/ 2
∴ tan A = = = −1 ∵ tan A is − ve when A lies in IV th quadrant 
cos A 1/ 2
1 1
cot A = = = −1 ∵ cos A is − ve when A lies in IV th quadrant 
tan A −1
1 1
and cosec A = = =− 2 ∵ cosec A is − ve when A lies in IV th quadrant 
sin A −1/ 2

Now,
1 + tan A + cosec A 1 + ( −1) + − 2
=
( ) =
− 2
= −1.
1 + cot A − cosec A 1 + ( −1) − − 2 ( ) 2

4
58. Ans. (d), Given, 5 tan θ = 4 ⇒ tan θ =
5
5sin θ − 3cos θ 5 tan θ − 3
Now, = [Dividing Nr. & Dr. by cos θ ]
sin θ + 2 cos θ tan θ + 2
 4
 5×  − 3 1 5
5
= = = .
4   14  14
 + 2  
5   5
π 1 π π
59. Ans. (b), ∵ cot = , cot = 1and cot = 3.
3 3 4 6

Office.: 606 , 6th Floor, Hariom Tower, Circular Road, Ranchi-1, Ph.: 0651-2562523, 9835508812, 8507613968
10 ( BY R. K. MALIK’S NEWTON CLASSES
)
2
 π π 1  π π π π
Clearly,  cot  .  cot  = . 3 = 1 =  cot  ∴ cot , cot and cot are in G.P.
 3  6 3  4 3 4 6
3cos θ + cos 3θ 3cos θ + 4 cos3 θ − 3cos θ 4 cos3 θ
60. Ans. (c), = 3
= 3
= cot 3 θ .
3sin θ − sin 3θ 3sin θ − 3sin θ + 4 sin θ 4sin θ
61. Ans. (a), log 3 tan1° + log 3 tan 2° + ..... + log 3 tan 89°
= ( log3 tan1° + log3 tan 89° ) + ( log3 tan 2° + log 3 tan 88° ) + ....
+ ( log3 tan 44° + log3 tan 46° ) + log 3 tan 45°
= log 3 ( tan1°. tan 89° ) + log3 ( tan 2°. tan 88° ) + ..... + log 3 ( tan 44°. tan 46° ) + log 3 tan 45°
= log 3 ( tan1°. cot1° ) + log 3 ( tan 2° cot 2° ) + .....
∵ tan θ = cot ( 90° − θ ) 
+ log 3 ( tan 44°. cot 44° ) + log 3 tan 45°
= ( log3 1 + log 3 1 + .... + log 3 1) + log 3 1 [∵ tan θ .cot θ = 1and tan 45° = 1]
= ( 0 + 0 + ... + 0 ) + 0 = 0.
62. Ans. (c), We have 1c ≈ 57° i.e. 1c > 1°
Also sin θ being an increasing function we have , sin1c > sin1° i.e. sin1° < sin1
63. Ans. (b), tan1°. tan 2°. tan 3°.....tan 89°
( tan1°.tan 89°) . ( tan 2° .tan 88°) . ( tan 3°. tan 87°) .... ( tan 44°. tan 46° ) .tan 45°
= ( tan1°. cot1° ) . ( tan 2° cot 2° ) . ( tan 3°.cot 3° ) .... ( tan 44°. cot 44° ) . tan 45° ∵ tan ( 90° − θ ) = cot θ 
= 1.1.1...1.1 [∵ tan θ .cot θ = 1and tan 45° = 1]
= 1.
144 5
64. Ans. (b), cos θ = 1 − sin 2 θ = 1 − =  Note : cos θ is + ve since θ lies in 1st quadrant 
169 13
9 4
sin φ = − 1 − cos 2 φ = − 1 − =−  Note: sin φ is − ve since φ lies in 2nd quadrant 
25 5
 12   3   5   4  56
Now, sin (θ + φ ) = sin θ cos φ + cos θ sin φ =   .  −  +   .  −  = − .
 13   5   13   5  65
1 − tan 2 15° 3
65. Ans. (b), 2
= cos ( 2 ×15° ) = cos 30° = .
1 + tan 15° 2
66. Ans. (d), Given, tan θ1 = k cot θ 2 ⇒ tan θ1 ⋅ tan θ 2 = k
cos (θ1 − θ 2 ) cos θ1 cos θ 2 + sin θ1 sin θ 2 1 + tan θ1 tan θ 2
Hence = = [ Dividing Nr.and Dr.by cos θ1 cosθ 2 ]
cos (θ1 + θ 2 ) cos θ1 cos θ 2 − sin θ1 sin θ 2 1 − tan θ1 tan θ 2
1+ k
= . [∵ tan θ1 = k cot θ 2 ⇒ tan θ1 tan θ 2 = k ]
1− k
67. Ans. (a), cos 480°. sin150° + sin 600°. cos 390° = cos ( 360° + 120° ) .sin (180° − 30° )
+ sin ( 540° + 60° ) .cos ( 360° + 30° ) = cos120°. sin 30° − sin 60°. cos 30°
= − cos 60°. sin 30° − sin 60°. cos 30° ∵ cos120° = cos (180° − 60° ) = − cos 60° 
= − ( sin 60°.cos 30° + cos 60°.sin 30° ) = − sin ( 60° + 30° ) = − sin 90° = −1.

Office.: 606 , 6th Floor, Hariom Tower, Circular Road, Ranchi-1, Ph.: 0651-2562523, 9835508812, 8507613968
( ) BY R. K. MALIK’S NEWTON CLASSES 11
2
68. Ans. (a), Given, ( cos x + sin x ) + k sin x cos x − 1 = 0
⇒ cos 2 x + sin 2 x + 2sin x cos x + k sin x cos x − 1 = 0
⇒ 1 + 2sin x cos x + k sin x cos x − 1 = 0 ∵ cos 2 x + sin 2 x = 1

⇒ ( k + 2 ) sin x cos x = 0 which can be true for all x if k = −2


 tan 45° − tan 30° 
69. Ans. (a), ∵ cot105° = cot ( 90° + 15° ) = − tan15° = − tan ( 45° − 30° ) = −  
 1 + tan 45° tan 30° 
 1 
 1− 3
= −

=−
3 −1
=−
( 3 −1 )( 3 −1 ) = − 4−2 3
= 3 − 2 , ∴ cot105° = 3 − 2
 1 + 1. 1



3 +1 ( 3 + 1)( 3 − 1) 2
 3 
sin (α − β ) sin ( β − γ ) sin ( γ − α )
70. Ans. (a), + +
sin α sin β sin β sin γ sin γ sin α
sin α cos β − cos α sin β sin β cos γ − cos β sin γ sin γ cos α − cos γ sin α
= + +
sin α sin β sin β sin γ sin γ sin α
= ( cot β − cot α ) + ( cot γ − cot β ) + ( cot α − cot γ ) = 0
71. Ans. (a), Given, sin x + sin 2 x = 1 ⇒ sin x = 1 − sin 2 x = cos 2 x

Squaring on both sides, we get sin 2 x = cos 4 x , ∴ cos 2 x + cos 4 x = sin x + sin 2 x = 1
a a a
72. Ans. (d), Given, x = h + ⇒ x−h = ⇒ = cos θ …(i)
cos θ cos θ x−h
b b b
and y = k + ⇒ y−k = ⇒ = sin θ …(ii)
sin θ sin θ y−k

a2 b2
Squaring and adding (i) and (ii) we get, 2
+ 2
=1
( x − h) ( y −k)
1 tan θ tan ϕ − 1
tan θ −
tan θ − cot ϕ tan ϕ tan ϕ tan θ tan ϕ − 1 tan θ tan θ
73. Ans. (a), = = = ⋅ =
tan ϕ − cot θ tan ϕ − 1 tan θ tan ϕ − 1 tan ϕ tan θ tan ϕ − 1 tan ϕ
tan θ tan θ
74. Ans. (c), ∵ We know that tan 85° = tan ( 90° − 5° ) = cot 5° ∴ tan 5° ⋅ tan 85° = tan 5° ⋅ cot 85° = 1

Now, let us group the terms as


( tan 5°⋅ tan 85° ) ⋅ ( tan15°⋅ tan 75° ) ⋅ ( tan 25°⋅ tan 65° ) ⋅ ( tan 35°⋅ tan 55° ) ⋅ tan 45° = 1
75. Ans. (b), Given, = cos 2 54° − sin 2 θ + cos 2 36° − sin 2 θ
= sin 2 36° + cos 2 36° − 2 sin 2 θ = 1 − 2 sin 2 θ = cos 2θ
Alternatively : cos ( 54° + θ ) cos ( 54° − θ ) + cos ( 36° − θ ) ⋅ cos ( 36° + θ )

= cos ( 54° + θ ) ⋅ cos [90° − 90° + 54° − θ ] + cos [ 90° − 90° + 36° − θ ] cos ( 36° + θ )

= cos ( 54° + θ ) ⋅ sin ( 36° + θ ) + sin ( 54° + θ ) ⋅ cos ( 36° + θ )

= sin ( 54° + θ + 36° + θ ) = sin ( 90° + 2θ ) = cos 2θ

Office.: 606 , 6th Floor, Hariom Tower, Circular Road, Ranchi-1, Ph.: 0651-2562523, 9835508812, 8507613968
12 ( ) BY R. K. MALIK’S NEWTON CLASSES
[As we know that sin A cos B + cos A sin B = sin ( A + B ) ]

3 9 4
76. Ans. (d), sin A = ⇒ cos A = − 1 − sin 2 A = − 1 − =−
5 25 5

144 5 sin A 3 sin B 5


and sin B = − 1 − cos 2 B = − 1 − = − , Hence tan A = = − and tan B = =
169 13 cos A 4 cos B 12
3 5 −9 + 5
− +
tan A + tan B 4 12 4 48 16
Hence tan ( A + B ) = = = 12 =− × =−
1 − tan A. tan B  3   5  48 + 15 12 63 63
1−  −   
 4   12  48
Alternatively :
Y Y
B
5 −12
3 A X
−5 O
−4 O
X 13

3 3 −5 5
From figure it is clear that tan A = = − and tan B = =
−4 4 −12 12
3 5 −9 + 5
− +
tan A + tan B 4 12 4 48 16
Therefore, tan ( A + B ) = = = 12 =− × =−
1 − tan A. tan B  3   5  48 + 15 12 63 63
1−  −   
 4   12  48

θ
2 sin 2
1 − cos θ 2 = tan θ
77. Ans. (c) x = =
1 + cos θ θ 2
2 cos 2
2
θ
2 tan
Hence
2x
= 2 = tan  2 × θ  = tan θ .
 
(1 − x ) 1 − tan 2 θ
2
 2
2

θ θ θ θ θ θ θ θ
sin + sin θ sin sin 1 + 2 cos 
+ 2 sin cos sin  1 + 2 cos 
2 2 2 2 = 2 2 2 2 θ
78. Ans. (a), = = = tan
θ θ θ θ θ θ θ 2
1 + cos + cos θ 1 + cos + 2 cos 2 − 1 cos + 2 cos 2 cos  1 + 2 cos 
2 2 2 2 2 2 2

1 + cos 2θ + sin 2θ 2 cos 2 θ + 2sin θ cos θ 2cos θ ( cos θ + sin θ )


79. Ans. (a), = = = cot θ
1 − cos 2θ + sin 2θ 2 sin 2 θ + 2sin θ cos θ 2sin θ ( sin θ + cos θ )

80. Ans. (b), 2 + 2 + 2cos 4θ = 2 + 2 + 2 ( 2 cos 2 2θ − 1) = 2 + 2 + 4 cos 2 2θ − 2

 π  π
= 2 + 2 cos 2θ = 2 {1 + 2 cos 2 θ − 1} [ ∵ θ ∈  0,  ⇒ 2θ ∈  0,  ,Hence cos 2 2θ = cos 2θ ]
 4  2

= 4 cos 2 θ = 2 cos θ = 2 cos θ ∵ θ is in 1st quadrant 

Office.: 606 , 6th Floor, Hariom Tower, Circular Road, Ranchi-1, Ph.: 0651-2562523, 9835508812, 8507613968
( BY R. K. MALIK’S NEWTON CLASSES
) 13
sin 2 x + ( sin 5 x − sin x ) sin 2 x + 2 cos 3 x sin 2 x sin 2 x (1 + 2 cos3 x )
81. Ans. (c), Given, = = = tan 2 x
cos 2 x + ( cos 5 x + cos x ) cos 2 x + 2 cos 3 x cos 2 x cos 2 x (1 + 2 cos3 x )

sin ( n + 1) α − sin ( n − 1)α


82. Ans. (b),
cos ( n + 1) α + 2cos nα + cos ( n − 1)α

 ( n + 1) α + ( n − 1) α   ( n + 1) α − ( n − 1) α 
2 cos   ⋅ sin  
 2   2 
=
 ( n + 1) α + ( n − 1) α   ( n + 1) α − ( n − 1) α 
2 cos   ⋅ cos   + 2 cos nα
 2   2 
α α
2sin ⋅ cos
2 cos n α sin α sin α 2 2 = tan α
= = =
2 cos nα .cos α + 2 cos nα cos α + 1 2α 2
2 cos
2
α α
2sin ⋅ cos
sin 2α − 0 2 2
Alternatively : Substituting n = 1, in given expression we get, =
cos 2α + 2 cos α + 1 2α
2 cos
2
2sin α cos α sin α α
= 2
= = tan
2 cos α + 2 cos α cos α + 1 2
sin 70° − cos 40° sin 70° − cos ( 90° − 50° )
83. Ans. (d), =
cos 70° + sin 40° cos 70° + sin ( 90° − 50° )

 70° + 50°   70° − 50° 


2 cos   ⋅ sin  
sin 70° − sin 50°  2   2  = 2 cos 60° sin10° = sin10° = tan10°
= =
cos 70° + cos 50°  70° + 50°   70° − 50°  2 cos 60° cos10° cos10°
2 cos   ⋅ cos  
 2   2 
A+ B A− B
2 cos
cos
cos A + cos B 2 2 A− B
84. Ans. (a), ∵ = = cot
sin A − sin B 2 cos A + B A − B 2
sin
2 2
A+ B A− B
2sin cos
sin A + sin B 2 2 A− B
and = = − cot
cos A − cos B −2sin A + B sin A − B 2
2 2
n n
 cos A + cos B   sin A + sin B   A− B  n  A− B 
 + ( −1) cot 
n
Therefore   +  = cot n  
 sin A − sin B   cos A − cos B   2   2 

But given, n is odd, ∴ ( −1) = −1 . Therefore, the value of the expression is 0.


n

85. Ans. (c), Given, cot α cot β = 2 …(1)

cos (α + β ) cos α cos β − sin α sin β


Now =
cos (α − β ) cos α cos β + sin α sin β

Office.: 606 , 6th Floor, Hariom Tower, Circular Road, Ranchi-1, Ph.: 0651-2562523, 9835508812, 8507613968
14 BY R. K. MALIK’S NEWTON CLASSES
( )
cos (α + β ) 1 − cot α cot β 1 − 2 −1
⇒ = = = [Dividing Nr. and Dr. by cos α ⋅ cos β ]
cos (α − β ) 1 + cot α cot β 1+ 2 3

86. Ans. (b), cos 0° + cos1° + ... + cos89° + cos 90° + cos 91° + ... + cos178° + cos179°
= 1 + cos1° + cos 2° + ... + cos89° + 0 + cos (180° − 89° ) + .. + cos (180° − 2° ) + cos (180° − 1° )

= 1 + cos1° + cos 2° + ... + cos89° − cos 89° − cos 88°... − cos 2° − cos1° = 1
87. Ans. (b), Given = cos 2 A + cos 2 B + 1 − sin 2 C = cos 2 A + cos ( B + C ) cos ( B − C ) + 1

= 1 − sin 2 A + cos ( 90° − A) cos ( B − C ) + 1 = 2 − sin 2 A + sin A cos ( B − C )

= 2 − sin A ( sin A − cos ( B − C ) ) = 2 − sin A cos ( B + C ) − cos ( B − C ) 

= 2 + sin A ⋅ 2 sin B sin C = 2 + 2 sin A sin B sin C


Alternatively : By setting A = B = C = 30° , we get cos 2 A + cos 2 B + cos 2 C
2
2
 3 9
= 3cos 30° = 3   =
 2  4

 1  1  1  1 9
Option (b) gives 2 + 2 sin 30° sin 30° sin 30° = 2 + 2       = 2 + =
 2  2  2  4 4
tan ( A + B ) + tan ( A − B ) p+q
88. Ans. (c), tan 2 A = tan {( A + B ) + ( A − B )} = = .
1 − tan ( A + B ) tan ( A − B ) 1 − pq
cos10° + sin10° 1 + tan10°
89. Ans. (c), = [Dividing Nr. & Dr. by cos10° ]
cos10° − sin10° 1 − tan10°
tan 45° + tan10°
= [Using tan 45° = 1 ]
1 − tan 45° tan10°
 tan A + tan B 
= tan ( 45° + 10° ) ∵ 1 − tan A tan B = tan ( A + B ) 
= tan 55°
90. Ans. (c), 12 sin 40° − 16sin 3 40° = 4 ( 3sin 40° − 4sin 3 40° )

3
= 4sin ( 3 × 40° ) = 4sin120° = 4sin (180° − 60° ) = 4sin 60° = 4 × = 2 3.
2
1 1
+
tan A + tan B  1 1
91. Ans. (c), tan ( A + B ) = = 2 3 ∵ tan A = 2 , tan B = 3 
1 − tan A tan B 1 1
1−  × 
 2 3
5
6 5/6 π
= = =1 ⇒ tan (π − C ) = 1 = tan [∵ A + B + C = π ⇒ A + B = π − C]
 1 5/6 4
1 − 
 6
π π 3π
⇒ π −C = ⇒ C =π − =
4 4 4

Office.: 606 , 6th Floor, Hariom Tower, Circular Road, Ranchi-1, Ph.: 0651-2562523, 9835508812, 8507613968
( BY R. K. MALIK’S NEWTON CLASSES
) 15
4

4 2 tan A 3 = 8 × 9 = 24 .
92. Ans. (b), ∵ 3 tan A − 4 = 0 ⇒ tan A = ∴ sin 2 A = =
3 1 + tan A 1 + 16 3 25 25
2

9
16 5
Also, sec A = − 1 + tan 2 A = − 1 + =− sec A is − ve since A lies in III rd quadrant 
9 3
3 −4
∴ cos A = − and so sin A = − 1 − cos 2 A =
5 5
 24   −4   −3  24 12 12
Now, 5sin 2 A + 3sin A + 4 cos A =  5 ×  +  3 ×  +  4 ×  = − − =0
 25   5   5  5 5 5
8
93. Ans. (c), cos θ = ⇒ sin θ = 1 − cos 2 θ
17
64 15
= 1− = .  Note : sin θ is + ve since θ lies in 1st quadrant 
289 17
Now, cos ( 30° + θ ) + cos ( 45° − θ ) + cos (120° − θ )
= cos 30° cos θ − sin 30° sin θ + cos 45° cos θ + sin 45° sin θ + cos120° cos θ + sin120° sin θ
3 1 1 1 1 3
= cos θ − sin θ + cos θ + sin θ − cos θ + sin θ
2 2 2 2 2 2
3 1 1  3 −1 1 
= ( sin θ + cos θ ) − ( sin θ + cos θ ) + ( sin θ + cos θ ) = ( sin θ + cos θ )  + 
2 2 2  2 2 

 15 8   3 − 1 1  23  3 − 1 1 
=  +   +  =  + .
 17 17   2 2  17  2 2 
94. Ans. (c), sin 2 17.5° + sin 2 72.5° = sin 2 17.5° + sin 2 ( 90° − 17.5° )
= sin 2 17.5° + cos 2 17.5° = 1 = 12 = tan 2 45° [∵ tan 45° = 1]
95. Ans. (d), 4sin A cos3 A − 4 cos A sin 3 A = 4 sin A cos A ( cos 2 A − sin 2 A )

= 2 ( 2sin A cos A ) .cos 2 A = 2sin 2 A.cos 2 A = sin 4 A.

cos3 θ − cos 3θ sin 3 θ + sin 3θ cos3 θ − ( 4 cos3 θ − 3cos θ ) sin 3 θ + ( 3sin θ − 4sin 3 θ )
96. Ans. (c), + = +
cos θ sin θ cos θ sin θ
−3cos 3 θ + 3cos θ 3sin θ − 3sin 3 θ
= + = 3 (1 − cos 2 θ ) + 3 (1 − sin 2 θ ) = 3sin 2 θ + 3cos 2 θ
cos θ sin θ
= 3 ( sin 2 θ + cos 2 θ ) = 3 × 1 = 3.

1 − tan 2 α 1 − ( 2 tan β + 1)
2
−2 tan 2 β −2 tan 2 β − tan 2 β
97. Ans. (d), cos 2α = = = = = = − sin 2 β .
1 + tan α 1 + ( 2 tan β + 1) 2 tan β + 2
2 2 2
2 ( tan β + 1) sec β
2 2

A+C
98. Ans. (a) A + C = 2 B ⇒ B =
2
 A+C   A−C 
2 sin   sin  
cos C − cos A  2   2   A+C 
Now, = = tan  
sin A − sin C  A+C   A−C   2 
cos  sin
  
 2   2 

Office.: 606 , 6th Floor, Hariom Tower, Circular Road, Ranchi-1, Ph.: 0651-2562523, 9835508812, 8507613968
16 ( ) BY R. K. MALIK’S NEWTON CLASSES
= tan B [Using (i)]
1 1 48 4 3
99. Ans. (c), Given, cos P = ⇒ sin P = 1 − cos 2 P = 1 − = =
7 49 49 7
[Since Q is acute, so sin P is positive]
13 169 27 3 3
and cos Q = ⇒ sin Q = 1 − cos 2 Q = 1 − = =
14 196 196 14
[Since Q is acute, so sin Q is positive]
4 3 13 1 3 3 52 3 − 3 3 49 3 3
Now, sin ( P − Q ) = sin P cos Q − cos P sin Q = . − . = = = = sin 60°
7 14 7 14 98 98 2
⇒ P − Q = 60°.
100. Ans. (b), sin120° cos150° − cos 240° sin 330°
= sin (180° − 60° ) cos (180° − 30° ) − cos (180° + 60° ) sin ( 360° − 30° )
 3  3  1  1 3 1 4
= ( sin 60° ) . ( − cos 30° ) − ( − cos 60° )( − sin 30° ) =   .  −  −  −  .  −  = − − = − = −1
 2   2   2  2 4 4 4

π  π   π   π   π   π 
101. Ans. (c), cos 4   − sin 4   = cos 2   − sin 2    . cos 2   + sin 2   
 24   24    24   24     24   24  
  π  π 3 +1
= cos 2    .1 = cos = cos15° =
  24   12 2 2
sin 300° tan 330° sec 420° sin ( 360° − 60° ) tan ( 360° − 30° ) sec ( 360° + 60° )
102. Ans. (c), =
tan135° sin 210° sec 315° tan (180° − 45° ) sin (180° + 30° ) sec ( 360° − 45° )
3 1
. .2
=
{ − sin 60°} .{− tan 30°} sec 60°
=
sin 60°.tan 30°.sec 60°
=
2 3 = 2
{− tan 45°}.{− sin 30°} .sec 45° tan 45° sin 30°.sec 45° 1. 1 . 2
2
 π  3π   5π   7π 
103. Ans. (a) 1 + cos  1 + cos  1 + cos  1 + cos 
 8 8  8  8 
  5π   3π  3π 
∵cos  8  = cos  π − 8  = − cos 8 and 
 π  3π   3π  π      
= 1 + cos  1 + cos  1 − cos 1 − cos 
 8  8  8  8   7π   π π 
 cos   = cos  π −  = − cos 
  8   8 8 
 π  3π  2 π 2 3π
= 1 − cos 2  1 − cos 2  = sin .sin
 8  8  8 8
π π  3π  π 3π  π
= sin 2 .cos 2 ∵sin 8 = cos  2 − 8  = cos 8 
8 8    
2 2
1 π π  1 π 
=  2 sin cos  =  sin  [∵2 sin A cos A = sin 2 A]
4 8 8 4 4
2
1  1  1
= . = .
4  2  8
1
104. Ans. (b), x + y = 90° ⇒ y = 90° − x ∵ sin x. sin y = sin x.sin ( 90° − x ) = sin x.cos x = sin 2 x.
2

Office.: 606 , 6th Floor, Hariom Tower, Circular Road, Ranchi-1, Ph.: 0651-2562523, 9835508812, 8507613968
( )BY R. K. MALIK’S NEWTON CLASSES 17
1 1 1 1 1
Now, −1 ≤ sin 2 x ≤ 1 ⇒ − ≤ sin 2 x ≤ ⇒ − ≤ sin x.sin y ≤ .
2 2 2 2 2
1
∴ Maximum value of sin x.sin y is .
2
105. Ans. (c), Given, cot α − cot β = n …(i)
1 1 cot β − cot α −n
∴ tan α − tan β = − = ⇒ m= [Using (i)]
cot α cot β cot α cot β cot α cot β
−n
⇒ cot α cot β = …(ii)
m
−n
+1
cot α cot β + 1 m 1 1
Now, cot (α − β ) = = = − [Using (i) and (ii)]
cot β − cot α −n m n
m +1
106. Ans. (c), sec θ − tan θ = …(i)
m −1
m −1
By taking inverse on both sides we get, sec θ + tan θ = …(ii)
m +1
( m + 1) ( m − 1) 2 ( m + 1)
2

Adding (i) and (ii) we get, 2 sec θ = + =


( m − 1) ( m + 1) ( m2 − 1)
m2 + 1 m2 − 1
⇒ sec θ = 2 ⇒ cos θ = 2 .
m −1 m +1
tan α + tan α + tan β − tan α tan α tan β
107. Ans. (b), tan ( 2α + β ) = tan (α + α + β ) =
1 − tan α tan α − tan α tan β − tan β tan α
 tan A + tan B + tan C − tan A tan B tan C 
∵ tan ( A + B + C ) = 1 − tan A tan B − tan B tan C − tan C tan A 
2
1 1 1 1
2 2.   +   −   .  
2 tan α + tan β − tan α .tan β 3 7 3 7
= 2
=   2     
1 − tan α − 2 tan α tan β 1 1 1
1 −   − 2.   .  
 3  3  7 
2 1 1  50 
+ −  
= 3 7 63 =  63  = 1 = tan π ⇒ ( 2α + β ) =
π
1 2  50  4 4
1− −  
9 21  63 
108. Ans. (d), 12 cot 2 θ − 31cosec θ + 32 = 0 ⇒ 12 cos 2 θ − 31sin θ + 32sin 2 θ = 0
⇒ 12 (1 − sin 2 θ ) − 31sin θ + 32 sin 2 θ = 0 ⇒ 20 sin 2 θ − 31sin θ + 12 = 0

31 ± 961 − 960 31 ± 1 3 4
⇒ sin θ = = = or
40 40 4 5
109. Ans. (d), Given, = sec θ + tan θ − sec θ = tan θ
For checking choice (a) Given = cosec θ + sec θ − tan θ ∴ It is wrong
For checking choice (b) Given cosec θ − ( sec θ − tan θ ) = cosec θ − sec θ + tan θ ∴ It is wrong
For checking choice (c) Given = sec θ + sec θ − tan θ ∴ It is wrong
For checking choice (d) Given = sec θ − ( sec θ − tan θ ) = tan θ , Hence choice (d) is correct

Office.: 606 , 6th Floor, Hariom Tower, Circular Road, Ranchi-1, Ph.: 0651-2562523, 9835508812, 8507613968
18 ( BY R. K. MALIK’S NEWTON CLASSES
)
α α x −1  1 − cos α  x − 1
110. Ans. (a), 2 x.sin 2 + 1 = x ⇒ sin 2 = ⇒ = ∵ 1 − cos 2 A = 2 sin 2 A
2 2 2x  2  2 x
1
⇒ cos α = ⇒ x must be positive
x
1 x2 −1 sin α
Now sin α = 1 − cos 2 α = 1 − 2
= and so, tan α = = x 2 − 1.
x x cos α
[Note sin α , cos α and tan α are all positive since α is acute]
sin θ + sin 2θ sin θ + 2 sin θ cos θ
111. Ans. (c), = ∵ 1 + cos 2θ = 2 cos 2 θ 
1 + cos θ + cos 2θ cos θ + 2 cos 2 θ
sin θ (1 + 2 cos θ ) sin θ
= = = tan θ .
cos θ (1 + 2 cos θ ) cos θ

1 2 cos x 1 2sin x cos x + 1


112. Ans. (a) , 2 cot x + 2
= + 2 =
sin x sin x sin x sin 2 x
2

=
2sin x cos x + sin 2 x + cos 2 x
=
( sin x + cos x ) =
sin x + cos x
=
− sin x − cos x
sin 2 x sin x2
sin x sin x
 3π 
∵ When 4 < x < π , we have, sin x ≥ 0 but cos x < 0 and cos x is greater than sin x in 
 
 magnitude i.e. cos x > sin x and so sin x + cos x is − ve ∴ sin x + cos x = − ( sin x + cos x ) 
113. Ans. (a), cos1° + cos 2° + cos 3° + ..... + cos180°
= {( cos1° + cos179° ) + ( cos 2° + cos178° ) + ( cos 3° + cos177° ) + ... + ( cos 89° + cos 91° )}
+ cos 90° + cos180°
= ( cos1° − cos1° ) + ( cos 2° − cos 2° ) + ( cos 3° − cos 3° ) + ... + ( cos 89° − cos89° ) + 0 + ( −1)

∵ cos (180° − θ ) = − cos θ 


= ( 0 + 0 + 0 + .... + 0 ) + 0 + ( −1) = −1.
3
114. Ans. (c), ∵ 5cos C + 3 = 0 ⇒ cos C = −
5
π
Since C is an angle of a ∆ABC and cos C is −ve, we have < C < π.
2
9 4  π 
Now, sin C = 1 − cos 2 C = 1 − = ∵ sin C is + ve since 2 < C < π 
25 5
sin C 4/5 4 4  4  −8
∴ tan C = = = − , We have sin C + tan C = +  −  = (Sum of the roots)
cos C −3 / 5 3 5  3  15
4  4 16
and sin C.tan C =   .  −  = − (Product of the roots)
5  3 15
 −8   16 
∴ Required quadratic equation is x 2 −   x +  −  = 0 ⇒ 15 x 2 + 8 x − 16 = 0 .
 15   15 
π 1 π
115. Ans. (c), sin (α + β ) = 1 ⇒ α + β = …(i) sin (α − β ) = ⇒ α −β = …(ii)
2 2 6
π π
Solving (i) and (ii) we get α = ,β= ∴ tan (α + 2β ) .tan ( 2α + β )
3 6

Office.: 606 , 6th Floor, Hariom Tower, Circular Road, Ranchi-1, Ph.: 0651-2562523, 9835508812, 8507613968
(BY R. K. MALIK’S NEWTON CLASSES
) 19

π π   2π π   2π   5π   π  π
= tan  +  .tan  +  = tan   . tan   = tan  π −  .tan  π − 
3 3  3 6  3   6   3  6
π π  1 

=  − tan  .  − tan  = − 3  −
 3  6 
(  =1
3
)
2
116. Ans. (d), For checking choice (a) ( cot θ + tan θ ) = cot 2 θ + tan 2 θ + 2

= ( cosec 2θ − 1) + ( sec2 θ − 1) + 2 = sec2 θ + cos ec 2θ , Hence, option (a) is correct.


2 2 2 2
2  cos θ sin θ   cos θ + sin θ 
For checking choice (b) ( cot θ + tan θ ) = + =
  
 sin θ cos θ   sin θ cos θ 
2
 1  2 2
=  = sec θ ⋅ cosec θ , Hence, option (b) is correct.
 cos θ sin θ 
2 2 2 2
2  cos θ sin θ   cos θ − sin θ 
For checking choice (c) ( cot θ − tan θ ) = − =
  
 sin θ cos θ   sin θ cos θ 
2
 
 cos 2θ  2
 = ( 2 cot 2θ ) = 4 cot 2θ , Hence, option (c) is correct, Hence, option (d) is correct
2
=
1
 sin 2θ 
2 
117. Ans. (a), 2 sin 2 x − 3sin x + 1 ≥ 0 ⇒ 2 sin 2 x − 2 sin x − sin x + 1 ≥ 0
⇒ 2sin x ( sin x − 1) − 1( sin x − 1) ≥ 0
( 2 p − 1)( p − 1)
⇒ ( 2sin x − 1)( sin x − 1) ≥ 0 ⇒ ( 2 p − 1)( p − 1) ≥ 0 where p = sin x
+ − +
1 1 p = 1/2 p =1
⇒ p ≤ or p ≥ 1 ⇒ sin x ≤ or sin x ≥ 1
2 2
1   π 
⇒ sin x ≤ ∵ sin x can not be greater than 1 and since x ∈ 0, 2  , so sin x ≠ 1
2    
π
⇒ 0≤ x≤
6
1 + sin 2 θ sin 2 θ sin 2 θ
118. Ans. (b), cos 2 θ 1 + cos 2 θ cos 2 θ =0 [C1 → C1 − C3 , C2 → C2 − C3 ]
4 sin 4θ 4 sin 4θ 1 + 4 sin 4θ

1 0 sin 2 θ
⇒ 0 1 cos 2 θ =0
 { }
⇒  (1 + 4sin 4θ ) − ( − cos 2 θ ) − 0 + sin 2 θ ( 0 + 1)  = 0

−1 −1 1 + 4sin 4θ
1
⇒ (1 + 4sin 4θ ) + cos 2 θ + sin 2 θ = 0 ⇒ 1 + 4sin 4θ + 1 = 0 ⇒ sin 4θ = −
2
π A π A π A π A π A π A
119. Ans. (c), sin 2  +  − sin 2  −  = sin  + + −  ⋅ sin  + − + 
8 2 8 2 8 2 8 2 8 2 8 2
π
= sin sin A ∵ sin ( A + B ) sin ( A − B ) = sin 2 A − sin 2 B 
4
1
= sin A.
2

Office.: 606 , 6th Floor, Hariom Tower, Circular Road, Ranchi-1, Ph.: 0651-2562523, 9835508812, 8507613968
20 ( ) BY R. K. MALIK’S NEWTON CLASSES
 1 − tan 2 x   2 tan x 
120. Ans. (b), a cos 2 x + b sin 2 x = a  2  + b 2 
 1 + tan x   1 + tan x 
 b 2   b   a 2 − b 2 2b 2 
a 1 − 2  + b  2.   +
a (1 − tan 2 x ) + b ( 2 tan x )  a   a  a

a  a 2 + b2 a2
= = = = ⋅ = a.
1 + tan 2 x b2  a 2 + b2  a a2 + b2
1+ 2  2 
a  a 
3
121. Ans. (b), tan α + cot α = m ⇒ ( tan α + cot α ) = m3
⇒ tan 3 α + cot 3 α + 3 tan α cot α ( tan α + cot α ) = m3
⇒ tan 3 α + cot 3 α + 3.1. m = m3 [∵ tan α cot α = 1]
⇒ tan 3 α + cot 3 α = m3 − 3m = m ( m 2 − 3 ) .

π  π 
122. Ans. (c), cos 2  + θ  − sin 2  − θ 
6  6 
π π  π π 
= cos  + θ + − θ  cos  + θ − + θ  ∵ cos 2 A − sin 2 B = cos ( A + B ) cos ( A − B ) 
6 6  6 6 
π 1
= cos . cos 2θ = cos 2θ .
3 2
 2 A + 2B   2 A − 2B 
123. Ans. (c), We have, sin 2 A + sin 2 B + sin 2C = 2 sin   .cos   + sin 2C
 2   2 
= 2sin ( A + B ) cos ( A − B ) + 2sin C cos C
= 2sin C cos ( A − B ) + 2sin C cos C ∵ sin ( A + B ) = sin (π − C ) = sin C 

= 2sin C cos ( A − B ) − cos ( A + B )  ∵ cos C = cos {π − ( A + B )} = − cos ( A + B ) 

= 2sin C [ 2sin A sin B ] = 4sin A sin B sin C.


 A+ B   A− B   2 C 
Also, cos A + cos B + cos C − 1 = 2cos   cos   + 1 − 2sin  −1
 2   2   2
C  A− B  2 C   A+ B  π C  C
= 2 sin
2
cos   − 2sin ∵ cos  2  = cos  2 − 2  = sin 2 
 2  2      
C   A− B   A + B   C π  A + B   A + B 
= 2sin cos   − cos   ∵ sin = sin  −    = cos  
2  2   2   2  2  2   2 
C A B A B C
= 4sin sin sin = 4sin sin sin
2 2 2 2 2 2
sin 2 A + sin 2 B + sin 2C 4sin A sin B sin C A B C
∴ = = 8cos cos cos
cos A + cos B + cos C − 1 4sin A sin B sin C 2 2 2
2 2 2

Office.: 606 , 6th Floor, Hariom Tower, Circular Road, Ranchi-1, Ph.: 0651-2562523, 9835508812, 8507613968
( )BY R. K. MALIK’S NEWTON CLASSES 21
 2 A + 2B   2 A − 2B 
124. Ans. (c), = 2 sin   .cos   + sin 2C
 2   2 
sin 2 A + sin 2 B + sin 2C = 2sin ( A + B ) cos ( A − B ) + 2sin C cos C

= 2sin (π − C ) cos ( A − B ) + 2sin C cos C = 2sin C {cos ( A − B ) + cos C} ∵ sin (π − C ) = sin C 

{ }
= 2 sin C cos ( A − B ) + cos (π − ( A + B ) ) = 2sin C {cos ( A − B ) − cos ( A + B )}

= 2sin C {2sin A sin B} = 4sin A sin B sin C .


 3θ + 5θ   3θ − 5θ 
125. Ans. (c), 2 cos θ − cos 3θ − cos 5θ = 2 cos θ − ( cos 3θ + cos 5θ ) = 2 cos θ − 2 cos   ⋅ cos  
 2   2 
= 2 cos θ − {2 cos 4θ cos θ } = 2 cos θ (1 − cos 4θ ) = 2 cos θ ( 2sin 2 2θ ) ∵ 1 − cos 2 A = 2 sin 2 A
2
= 4 cos θ sin 2 2θ = 4 cos θ ( 2 sin θ cos θ ) = 16sin 2 θ cos3 θ .
126. Ans. (a), We have, 2 cos x + 2 cos 3 x = cos y …(i) 2 sin x + 2sin 3 x = sin y …(ii)
Squaring and adding (i) and (ii) we get
( 4 cos x + 4 cos 3x + 8cos x cos 3x ) + ( 4 sin x + 4sin 3x + 8sin x sin 3x ) = cos
2 2 2 2 2
y + sin 2 y

⇒ 4 ( cos x + sin x ) + 4 ( cos 3 x + sin 3 x ) + 8 ( cos x cos 3 x + sin x sin 3 x ) = 1


2 2 2 2

⇒ 4 + 4 + 8cos ( 3x − x ) = 1 ∵ cos A cos B + sin A sin B = cos ( A − B ) 


−7
⇒ 8 + 8 cos 2 x = 1 ⇒ cos 2 x =
8
θ θ x −1 x +1
127. Ans. (a),∵ cos = 1 − sin 2 = 1− =
2 2 2x 2x
 π θ π θ θ 
 Note : 0 < θ < 2 ⇒ 0 < 2 < 4 ∴ cos 2 is + ve and tan 2 is also + ve 

θ x −1 x +1
θ
2sin cos . 2. 2
sin θ 2 2 = 2x 2 x = 2 x −1 = x2 − 1
∴ tan θ = =
cos θ θ  x −1  2x − 2x + 2
1 − 2sin 2 1 − 2.  
2  2x 

128. Ans. (d),


sin 3θ + sin 5θ + sin 7θ + sin 9θ
=
( sin 3θ + sin 9θ ) + ( sin 5θ + sin 7θ )
cos 3θ + cos 5θ + cos 7θ + cos 9θ ( cos 3θ + cos 9θ ) + ( cos 5θ + cos 7θ )
2 sin 6θ cos 3θ + 2sin 6θ cos θ 2 sin 6θ ( cos 3θ + cos θ )
= = = tan 6θ .
2 cos 6θ cos 3θ + 2 cos 6θ cos θ 2 cos 6θ ( cos 3θ + cos θ )
 cos α sin α   cos α sin α   cos β sin β 
129. Ans. (d), Given, A (α ) =   , ∴ A (α ) . A ( β ) = 
 − sin α cos α   − sin α cos α   − sin β cos β 
 cos α cos β − sin α sin β cos α sin β + sin α cos β   cos (α + β ) sin (α + β ) 
= =  = A (α + β ) .
 − sin α cos β − cos α sin β − sin α sin β + cos α cos β   − sin (α + β ) cos (α + β ) 
130. Ans. (d), cos15° − sin15° = cos15° − cos 75° = 2sin 45° sin 30°
  C + D   D − C 
∵ cos C − cos D = 2sin  2  sin  2  
    

Office.: 606 , 6th Floor, Hariom Tower, Circular Road, Ranchi-1, Ph.: 0651-2562523, 9835508812, 8507613968 21
22 ( ) BY R. K. MALIK’S NEWTON CLASSES
1 1 1
= 2× . =
2 2 2
 1 1  π π
131. Ans. (c), sin x + cos x = 2 sin x + cos x  = 2. sin x cos + cos x sin
 2 2  4 4

 π  π
= 2 sin  x +  , As −1 ≤ sin  x +  ≤ 1 [∵ − 1 ≤ sin θ ≤ 1]
 4  4
 π  π
⇒ 0 ≤ sin  x +  ≤ 1 ⇒ 0 ≤ 2 sin  x +  ≤ 2 ∴ 0 ≤ sin x + cos x ≤ 2.
 4  4
3 tan ( A / 3) − tan 3 ( A / 3)  A  3 tan θ − tan 3 θ 
132. Ans. (d), = tan  3  = tan A ∵ tan 3θ =
1 − 3 tan ( A / 3)
2
 3  1 − 3 tan 2 θ 
C 1 0 2 cos θ 1 0
133. Ans. (d), ∆ = 1 C 1 = 1 2 cos θ 1 = 2 cos θ ( 4 cos 2 θ − 1) − 1( 2 cos θ − 6 )
6 1 C 6 1 2 cos θ
= 8cos3 θ − 2 cos θ − 2 cos θ + 6 = 8cos3 θ − 4 cos θ + 6
π
(b) and (c) are wrong choices can be checked easily by supposing θ =
2
2
sin 4θ 2sin 2θ
Hence, given expression become 6 but becomes 0 and also becomes 0
sin 4θ sin θ
1
134. Ans. (d), cos 20°. cos 40°. cos 60°. cos80° = cos 20°. cos 40°. cos80°
2
1 1 1
= .cos 20°. cos ( 60° − 20° ) . cos ( 60° + 20° ) = . cos ( 3 × 20° )
2 2 4
 1 
 Using the formula cos θ .cos ( 60° − θ ) cos ( 60° + θ ) = 4 cos 3θ 
1 1 1 1
= cos 60° = × = .
8 8 2 16
α +γ
135. Ans. (d), Since α , β , γ are in A.P., so we have α + γ = 2 β ⇒β= …(i)
2
α +γ  α −γ  α +γ 
2 cos   sin   cos  
sin α − sin γ  2   2  =  2  = cos β
Now, = [Using(i)]
cos γ − cos α α +γ  α −γ   α + γ  sin β
2 sin   sin   sin  
 2   2   2 
= cot β .
136. Ans. (a), ( cos 52° + cos 68° ) + cos172° = ( 2cos 60° cos 8° ) + cos (180° − 8° )
 1 
=  2 × × cos 8°  − cos8° = cos 8° − cos8° = 0.
 2 
1 + cos 2 A 1 + cos 2 B cos 2C − 1
137. Ans. (c), cos 2 A + cos 2 B − sin 2 C = + +
2 2 2
1 1 1 1
= + ( cos 2 A + cos 2 B + cos 2C ) = + ( 2 cos ( A + B ) cos ( A − B ) + 2 cos 2 C − 1)
2 2 2 2
= cos ( A + B ) cos ( A − B ) + cos 2 C = cos ( 2π − C ) cos ( A − B ) + cos 2 C

22 Office.: 606 , 6th Floor, Hariom Tower, Circular Road, Ranchi-1, Ph.: 0651-2562523, 9835508812, 8507613968
( ) BY R. K. MALIK’S NEWTON CLASSES 23
= cos C {cos ( A − B ) + cos C } ∵ cos ( 2π − C ) = cos C 

= cos C {cos ( A − B ) + cos ( A + B )} ∵ cos C = cos {2π − ( A + B )} = cos ( A + B ) 

= cos C {2 cos A cos B} = 2 cos A cos B cos C.


2 2 x
138. Ans. (b), Given, x = a ( sec θ + tan θ ) ⇒ ( sec θ + tan θ ) = …(i)
a
4
4 b ( sec θ − tan θ ) 4
Now, y = b ( sec θ − tan θ ) = 4
. ( sec θ + tan θ )
( sec θ + tan θ )
b ( sec2 θ − tan 2 θ )
4
b b
= 4
= 4
= 2
[Using(i)]
( secθ + tan θ ) ( secθ + tan θ ) x
 
a
⇒ x 2 y = a 2b ⇒ x 4 y 2 = a 4b 2 .
139. Ans. (d) (1 + tan A tan B ) 2 + (tan A − tan B ) 2
= 1 + tan 2 A tan 2 B + 2 tan A tan B + tan 2 A + tan 2 B − 2 tan A tan B
= 1 + tan 2 A + tan 2 A tan 2 B + tan 2 B = (1 + tan 2 A) + tan 2 B (1 + tan 2 A) = (1 + tan 2 A) (1 + tan 2 B )
= sec 2 A.sec 2 B.
 3 
140. Ans. (c), Let P = 3sin θ + cos 2θ = 3sin θ + 1 − 2sin 2 θ = −2  sin 2 θ − sin θ  + 1
 2 
2
 3 9   9  3   17 
= −2  sin 2 θ − sin θ +  + 1 +  = −2  sin θ −  +   .
 2 16   8   4  8 
2
 3
Clearly, P has the maximum value when  sin θ −  has the
 4
2
  3  17
Minimum value i.e., 0. ∵  sin θ −  ≥ 0  ∴ The maximum value of P is .
  4  8
2π π
141. Ans. (a), sin 2 − sin 2
15 30
 2π π   2π π  π π
= sin  +  sin  −  = sin sin ∵ sin 2 A − sin 2 B = sin ( A + B ) sin ( A − B ) 
 15 30   15 30  6 10
1 5 −1 5 −1  π 5 − 1
= . = ∵ sin = sin18° = 
2 4 8  10 4 
3
 π π 3π 5π
142. Ans. (d), ∑ cos ( 2k − 1) 12  = cos
k =1
2 2

12
+ cos 2
12
+ cos 2
12
2
π π
π π  π  1  π 1 3
= cos 2
+ cos + sin  −  = cos 2 + 
2 2
 + sin 2 = 1+ = .
12 4  2 12  12  2  12 2 2
1
143. Ans. (d), f ( x ) = sin 4 x + cos 4 x = ( sin 2 x + cos 2 x ) − 2sin 2 x cos 2 x = 1 −
2 2
( sin 2 x ) .
2
2
Clearly, f ( x ) has a minimum value when ( sin 2 x ) has the maximum value.

Office.: 606 , 6th Floor, Hariom Tower, Circular Road, Ranchi-1, Ph.: 0651-2562523, 9835508812, 8507613968 23
24 ( BY R. K. MALIK’S NEWTON CLASSES
)
 π 
i.e., when sin 2 x has a maximum value ∵ 0 ≤ x ≤ 2 ⇒ 0 ≤ 2 x ≤ π ⇒ sin 2 x is + ve 
π π
i.e., when sin 2 x = 1 i.e., when 2 x = i.e. when x = .
2 4
4 4
π  π  π   1   1  1 1 1
∴ Minimum value of f ( x ) = f   = sin 4   + cos 4   =   +  = + =
4 4 4  2  2 4 4 2
1
Alternatively : f ( x ) = sin 4 x + cos 4 x = ( sin 2 x + cos 2 x ) − 2sin 2 x cos 2 x = 1 −
2 2
( sin 2 x ) .
2
1 2
Clearly, f ( x ) has a minimum value when ( sin 2 x ) has a maximum value.
2
2 1 2 1
We have −1 ≤ sin 2 x ≤ 1 ⇒ 0 ≤ ( sin 2 x ) ≤ 1 ⇒ 0 ≤ ( sin 2 x ) ≤
2 2
1 2 1 1 1
∴ Maximum value of ( sin 2 x ) is and so, maximum value of f ( x ) = 1 − = .
2 2 2 2
 1 − cos 2 A   1 − cos 2 B   1 − cos 2C 
144. Ans. (d), sin 2 A + sin 2 B + sin 2 C =  + + 
 2   2   2 
3 1 3 1
{
= − ( cos 2 A + cos 2 B + cos 2C ) = − ( 2 cos 2 A − 1) + 2 cos ( B + C ) cos ( B − C )
2 2 2 2
}
3 1
= − {−1 + 2 cos 2 A + 2 cos (π − A ) cos ( B − C )} [∵ A + B + C = π ⇒ B + C = π − A]
2 2
1
= 2 − {2 cos 2 A − 2 cos A cos ( B − C )} = 2 + cos A ⋅ {cos ( B + C ) + cos ( B − C )}
2
∵ cos A = cos {π − ( B + C )} = − cos ( B + C ) 
= 2 + 2 cos A cos B cos C.
π
145. Ans. (d), Since < α < π , so sin α is + ve and cos α is − ve
2

Since π < β < , so sin β is − ve and cos β is − ve
2
225 64 8
Now, cos α = − 1 − sin 2 α = − 1 − =− =− [∵ cos α is − ve]
289 289 17
144 169 13
sec β = − 1 + tan 2 β = − 1 + =− =− [∵ cos β is − ve and also sec β is − ve ]
25 25 5
1 5 25 144 12
∴ cos β = =− and so, sin β = − 1 − cos 2 β = − 1 − =− =− [∵ sin β is − ve]
sec β 13 169 169 13
 12   8   5   15  96 + 75 171
Now, sin ( β − α ) = sin β cos α − cos β sin α =  −  .  −  −  −  .   = =
 13   17   13   17  221 221
 1
146. Ans. (a), cos 2θ + 2 cos θ = 2 cos 2 θ − 1 + 2 cos θ = 2  cos 2 θ + cos θ − 
 2
2 2 2
 1  3   1 3  1
= 2  cos θ +  −  = 2  cos θ +  − Now, we have  cos θ +  ≥ 0 ∀ θ ∈ R
 2  4   2 2  2

24 Office.: 606 , 6th Floor, Hariom Tower, Circular Road, Ranchi-1, Ph.: 0651-2562523, 9835508812, 8507613968
( ) BY R. K. MALIK’S NEWTON CLASSES 25
2 2
 1  1 9
Also, maximum value of  cos θ +  is  1 +  i.e., [Obtained when cos θ = 1 ]
 2  2 4
2 2
 1 9  1 9
∴ 0 ≤  cos θ +  ≤ ⇒ 0 ≤ 2  cos θ +  ≤
 2 4  2 2
2
 3  1 3 9 3 3
⇒  0 −  ≤ 2  cos θ +  − ≤  −  ⇒− ≤ cos 2θ + 2 cos θ ≤ 3
 2  2 2 2 2 2
147. Ans. (d), cos 2 α + cos 2 (α − 120° ) + cos 2 (α + 120° )
2 2
= cos 2 α + {cos α cos120° + sin α sin120°} + {cos α cos`120° − sin α sin120°}
2 2
2
 1 3   1 3 
= cos α + − cos α + sin α  +  − cos α − sin α 
 2 2   2 2 
1 3 3 1 3 3
= cos 2 α + cos 2 α + sin 2 α − sin α cos α + cos 2 α + sin 2 α + sin α cos α
4 4 2 4 4 2
3 3 3 3 3
= cos 2 α + sin 2 α = ( cos 2 α + sin 2 α ) = × 1 = .
2 2 2 2 2
α β β γ γ α
148. Ans. (b), We know that tan ⋅ tan + tan ⋅ tan = 1 if α + β + γ = 180°
+ tan ⋅ tan
2 2 2 2 2 2
Hence, here tan A tan B + tan B tan C + tan C tan A = 1 [∵ A + B + C = 90° ]
⇒ tan A tan B + tan B tan C + tan C tan A = 1
149. Ans. (a), 2 ( sin 6 θ + cos 6 θ ) − 3 ( sin 4 θ + cos 4 θ ) + 1

{ } {
= 2 ( sin 2 θ + cos 2 θ ) − 3sin 2 θ cos 2 θ ( sin 2 θ + cos 2 θ ) −3 ( sin 2 θ + cos 2 θ ) − 2sin 2 θ cos 2 θ + 1
3 2
}
 Using ( a3 + b3 ) = ( a + b )3 − 3ab ( a + b ) and ( a 2 + b 2 ) = ( a + b ) 2 − 2ab 
 
= 2 {1 − 3sin 2 θ cos 2 θ } − 3{1 − 2 sin 2 θ cos 2 θ } + 1 = 0 = 2 − 6 sin 2 θ ⋅ cos 2 θ − 3 + 6sin 2 θ cos 2 θ + 1
sin 5θ sin ( 2θ + 3θ ) 1
150. Ans. (b), = = {sin 2θ cos 3θ + cos 2θ sin 3θ }
sin θ sin θ sin θ
1
=
sin θ
{
2 sin θ cos θ ( 4 cos3 θ − 3cos θ ) + ( 2 cos 2 θ − 1)( 3sin θ − 4 sin 3 θ ) }
= 2 cos θ ( 4 cos3 θ − 3cos θ ) + ( 2 cos 2 θ − 1) . ( 3 − 4sin 2 θ )

{ }
= 8cos 4 θ − 6 cos 2 θ + ( 2 cos 2 θ − 1) . 3 − 4 (1 − cos 2 θ ) = 8cos 4 θ − 6 cos 2 θ + ( 2 cos 2 θ − 1)( 4 cos 2 θ − 1)

= 8cos 4 θ − 6 cos 2 θ + 8 cos 4 θ − 6 cos 2 θ + 1 = 16 cos 4 θ − 12 cos 2 θ + 1.


151. Ans. (d), 1 + cos α + cos 2 α + .... = 2 − 2
 ∵ 1 + cos α + cos 2 α + ....is an infinite G.P. with common ratio 
1  
⇒ = 2− 2 1
1 − cos α  r = cos α < 1 and first term a = 1 and so S = a = 
 ∞
1 − r 1 − cos α 
1 1 2+ 2 2+ 2 2+ 2 1
⇒ 1 − cos α = = . = = = 1+
2− 2 2− 2 2+ 2 4−2 2 2
1 π  π 3π 3π
⇒ cos α = − = − cos = cos  π −  = cos ⇒α = .
2 4  4 4 4

Office.: 606 , 6th Floor, Hariom Tower, Circular Road, Ranchi-1, Ph.: 0651-2562523, 9835508812, 8507613968 25
26 ( BY R. K. MALIK’S NEWTON CLASSES
)
 1   1   1   1   1   1 
152. Ans. (d), cos 2  7 °  − cos 2  37 °  = 1 − sin 2  7 °  − 1 + sin 2  37 °  = sin 2  37 °  − sin 2  7 ° 
 2   2   2   2   2   2 
= sin ( 45° ) sin ( 30° ) ∵ sin 2 A − sin 2 B = sin ( A + B ) sin ( A − B ) 
1 1 1
= . = .
2 2 2 2
π   3π   5π   7π   9π 
153. Ans. (b), tan   tan   tan   tan   tan  
 20   20   20   20   20 
π   3π  π   π 3π  π π  π   3π   3π  π 
= tan   tan   tan   tan  −  tan  −  = tan   tan   .1.cot   .cot   = 1.
 20   20  4  2 20   2 20   20   20   20   20 
π π 5π π π  π 5π  π  
154. Ans. (b), cos 2 + cos 2 + cos 2 = sin 2  −  + cos 2 + cos 2 ∵ sin  2 − θ  = cos θ 
12 4 12  2 12  4 12    
2
5π  1 
2 5π 1
= sin +  + cos 2 = 1+ ∵ sin 2 A + cos 2 A = 1
12  2  12 2
3
= .
2
 A+ B   A− B 
155. Ans. (a), sin A + sin B + sin C = 2sin   cos   + sin C
 2   2 
C  A− B  C C   A+ B   C C
= −2sin
2
cos   + 2sin cos ∵ sin  2  = sin  − 2  = − sin 2 
 2  2 2      
C C  A − B 
= 2 sin  cos − cos  
2 2  2 
C   A+ B   A − B   C   A + B   A + B 
= 2 sin  cos   − cos   ∵ cos = cos  −    = cos  
2  2   2   2   2   2 
C  A B A B C
= 2 sin .  −2sin sin  = −4sin sin sin .
2  2 2 2 2 2
156. Ans. (b) cos 35° + cos85° + cos155°
1
= 2 cos 60° cos 25° + cos(180° − 25°) = 2. .cos 25° − cos 25° = cos 25° − cos 25° = 0
2
157. Ans. (b), sin ( A + B + C ) = 1 ⇒ A + B + C = 90° …(i)
1
tan ( A − B ) = ⇒ A − B = 30° …(ii)
3
1
sec ( A + C ) = 2 ⇒ cos ( A + C ) =⇒ A + C = 60° …(iii)
2
Subtracting (iii) from (i) we get B = 30°. Substituting B = 30° in (ii) we get A = 60°.
Substituting A = 60° in (iii) we get C = 0°.
158. Ans. (c)
(a) When A + B + C = π , we have
 A+ B   A− B  C C
sin A + sin B + sin C = 2sin   cos   + 2sin cos
 2   2  2 2
C  A− B  C C   A+ B  π C  C
= 2 cos
2
cos   + 2sin cos ∵ sin  2  = sin  2 − 2  = cos 2 
 2  2 2      

26 Office.: 606 , 6th Floor, Hariom Tower, Circular Road, Ranchi-1, Ph.: 0651-2562523, 9835508812, 8507613968
( ) BY R. K. MALIK’S NEWTON CLASSES 27
C   A− B   A + B   C π  A + B   A + B 
= 2 cos cos   + cos   ∵ sin = sin  −    = cos  
2  2   2   2  2  2   2 
C A B A B C
= 2 cos 2 cos cos  = 4 cos cos cos ∴ Statement (a) is correct.
2 2 2 2 2 2
 A+ B   A− B  C C
(b) When A + B + C = 2π , we have, sin A + sin B + sin C = 2sin   cos   + 2sin cos
 2   2  2 2
C  A− B  C C   A+ B    C  C
= 2 sin cos   + 2 sin cos ∵ sin   = sin π −    = sin 
2  2  2 2   2    2  2
C   A− B  C
= 2 sin cos   + cos 
2  2  2
C   A− B   A + B   C   A + B   A + B 
= 2 sin cos   − cos   ∵ cos = cos π −    = − cos  
2  2   2   2   2   2 
C A B A B C
= 2 sin 2sin sin  = 4sin sin sin ∴ Statement (b) is correct.
2 2 2 2 2 2
 A+ B   A− B  C C
(c) When A + B + C = 0, we have sin A + sin B + sin C = 2sin   cos   + 2sin cos
 2   2  2 2
C  A− B  C C   A+ B   C C
= −2sin
2
cos   + 2sin cos ∵ sin  2  = sin  − 2  = − sin 2 
 2  2 2      
C C  A − B 
= 2 sin cos − cos  
2 2  2 
C   A+ B   A − B   C   A + B   A + B 
= 2 sin cos   − cos   ∵ cos = cos −    = cos  
2  2   2   2   2   2 
C A B A B C
= 2 sin  −2 sin sin  = −4 sin sin sin ∴ Statement (c) is incorrect.
2 2 2 2 2 2
(d) When A + B + C = π , we have sin 2 A + sin 2 B + sin 2C = 2sin ( A + B ) cos ( A − B ) + 2sin C cos C
= 2sin C cos ( A − B ) + 2sin C cos C ∵ sin ( A + B ) = sin (π − C ) = sin C 

= 2sin C {cos ( A − B ) − cos ( A + B )} ∵ cos C = cos {π − ( A + B )} = − cos ( A + B ) 

= 2sin C {2sin A sin B} = 4 sin A sin B sin C ∴ Statement (d) is correct.


π  π 
159. Ans. (d) sin θ sin  − θ  sin  + θ 
3  3 
 π   3  1 1
= sin θ sin 2   − sin 2 θ  = sin θ  − sin 2 θ  = sin θ (3 − 4 sin 2 θ ) = (3sin θ − 4sin 3 θ )
 3  4  4 4
1
= sin 3θ
4
1 1 3
160. Ans. (b), Given, sin φ = ⇒ cos φ = 1 − sin 2 φ = 1 − = .
10 10 10

Office.: 606 , 6th Floor, Hariom Tower, Circular Road, Ranchi-1, Ph.: 0651-2562523, 9835508812, 8507613968 27
28 ( ) BY R. K. MALIK’S NEWTON CLASSES
 1 
sin φ  10  = 1
∴ tan φ = = 
3  3
[ Note:sin φ , cos φ , tan φ are all +ve since φ lies in1st quad.]
cos φ 
 
 10 
1 2
2.
 
2 tan φ 3 =  3  = 3 , Also, tan θ = 1 .
and so, tan 2φ = = 2
1 − tan 2 φ 1 8 4 7
1−    
3 9
1 3  25 
+  
tan θ + tan 2φ 28
∴ tan (θ + 2φ ) = = 7 4 =   = 1 ⇒ tan (θ + 2φ ) = tan 45° ⇒ θ + 2φ = 45°.
1 − tan θ tan 2φ  1 3   25 
1−  .   
 7 4   28 
161. Ans.(c), cosec A ( sin B cos C + cos B sin C )
= cosec A ⋅ sin ( B + C ) = cosec A ( sin (π − A) ) [∵ A + B + C = π ⇒ B + C = π − A]
= cosec A.sin A = 1
162. Ans. (c), ( cos 6θ + cos 4θ ) + ( cos 2θ + 1) = 2 cos 5θ cos θ + 2 cos 2 θ
= 2 cos θ {cos θ + cos 5θ } = 2 cos θ {2 cos 3θ cos 2θ } = 4 cos θ cos 2θ cos 3θ .
2 2
163. Ans. (c), ( sin α + cosec α ) + ( cos α + sec α ) = k + tan 2 α + cot 2 α
⇒ sin 2 α + cosec 2α + 2 + cos 2 α + sec2 α + 2 = k + tan 2 α + cot 2 α
⇒ 4 + ( sin 2 α + cos 2 α ) + cos ec 2α + sec2 α = k + tan 2 α + cot 2 α

⇒ 5 + ( cot 2 α + 1) + ( tan 2 α + 1) = k + tan 2 α + cot 2 α


⇒ 7 + tan 2 α + cot 2 α = k + tan 2 α + cot 2 α ⇒ k = 7.
sin 70° + cos 40° sin 70° + sin ( 90° − 50° )
164. Ans. (d), =
cos 70° + sin 40° cos 70° + sin ( 90° − 50° )
sin 70° + sin 50°
= ∵ sin θ = cos ( 90° − θ ) and cos θ = sin ( 90° − θ ) 
cos 70° + cos 50°
 C+D C−D 
∵ sin C + sin D = 2 sin  2  cos  2  
2sin 60° cos10°      
=
2 cos 60° cos10°  C+D  C − D 
and cos C + cos D = 2 cos   cos  
  2   2 
= tan 60° = 3.
π  π  π  π π  π 
165. Ans.(a), cos 2  + θ  − sin 2  − θ  or use sin  − θ  = cos  −  − θ   = cos  + θ 
4  4  4  2 4  4 
 π  π   π  π 
= cos  + θ  +  − θ   .cos  + θ  −  − θ   ∵ cos 2 A − sin 2 B = cos ( A + B ) .cos ( A − B ) 
 4  4   4  4 
π  π 
= cos .cos 2θ = 0 ∵ cos = 0 
2  2 
7 7 11
166. Ans. (b), Let P = sin 2 θ − cos θ + = + (1 − cos 2 θ ) − cos θ = − ( cos 2 θ + cos θ )
4 4 4

28 Office.: 606 , 6th Floor, Hariom Tower, Circular Road, Ranchi-1, Ph.: 0651-2562523, 9835508812, 8507613968
( ) BY R. K. MALIK’S NEWTON CLASSES 29
2 2
11 1  2 1  11 1  2 1 1   1
= + −  cos θ + cos θ +  = + − ( cos θ ) + 2   cos θ +    = 3 −  cos θ +  .
4 4  4 4 4  2  2    2
2
 1
Maximum value of P is obtained when  cos θ +  = 0 i.e.
 2
1
When cos θ = − and max . value of P = 3 − 0 = 3.
2
2
 1
Minimum value of P is obtained when  cos θ +  has the maximum value i.e. when cos θ = 1 and
 2
2 2
 1 3 3
So the minimum value of P = 3 −  1 +  = 3 −   =
 2 2 4
Max.value of P 3
∴ Required ratio = = = 4.
Min.value of P 3 / 4
 π  π π
167. Ans. (b), We have, 5sin θ + 3sin  θ +  + 3 = 5sin θ + 3  sin θ cos + cos θ sin  + 3
 3  3 3
3 3 3 13 3 3
= 5sin θ + sin θ + cos θ + 3 = sin θ + cos θ + 3
2 2 2 2
13 3 3
Now, the maximum value of sin θ + cos θ is
2 2
2 2
 13   3 3  169 27 196
  +   = + = = 49 = 7
2  2  4 4 4

 We know that the max .value of a sin θ + b cos θ is a 2 + b2 


 
13 3 3
∴ The maximum value of sin θ + cos θ + 3 is 10
2 2
 π
Thus, the maximum value of 5sin θ + 3sin  θ +  + 3 is 10.
 3
168. Ans. (c), tan 20°. tan 40°. tan 60°. tan 80° = 3.tan 20°. tan 40°.tan 80°.
= 3.tan 20° .tan ( 60° − 20° ) .tan ( 60° + 20° )
= 3.tan ( 3 × 20° )  Using the formula tan θ . tan ( 60° − θ ) ⋅ tan ( 60° + θ ) = tan 3θ 

= 3.tan 60° = 3. 3 = 3.
π  π 
169. Ans. (d) cos θ cos  − θ  cos  + θ 
3  3 
 π  1  1
= cos θ cos 2 − sin 2 θ  = cos θ  − sin 2 θ  = cos θ {1 − 4sin 2 θ }
 3  4  4
1 1 1 1
= cos θ {1 − 4(1 − cos 2 θ )} = cos θ (4 cos 2 θ − 3) = (4 cos3 θ − 3cos θ ) = cos 3θ .
4 4 4 4
170. Ans. (b), sin163° cos 347° + sin 73° sin167°
= sin (180° − 17° ) cos ( 360° − 13° ) + sin ( 90° − 17° ) sin (180° − 13° )
1
= sin17° cos13° + cos17° sin13° = sin (17° + 13° ) = sin 30° = .
2

Office.: 606 , 6th Floor, Hariom Tower, Circular Road, Ranchi-1, Ph.: 0651-2562523, 9835508812, 8507613968 29
30 ( ) BY R. K. MALIK’S NEWTON CLASSES
π  π 
171. Ans. (b) 2 + sin  + θ  + 2 cos  − θ 
4  4 
 1   1   π π
= 2 + + 2  (sin θ + cos θ ) = 2 +  + 2  . 2  sin θ cos + cos θ sin 
 2   2   4 4
 π  π
= 2 + (2 + 1).sin  θ +  = 2 + 3sin  θ +  .
 4  4
 π
Clearly, the maximum value of sin  θ +  is 1.
 4
π  π 
∴ The maximum value of 2 + sin  + θ  + 2 cos  − θ 
4  4 
= 2 +3.
tan θ m 2 − 1
∴ sin θ = =
sec θ m 2 + 1
π π
172. Ans. (d), A − B = ⇒ A= +B
4 4
1 + tan B 2
∴ tan A = ⇒ 1 + tan A = ⇒ (1 + tan A )(1 − tan B ) = 2.
1 − tan B 1 − tan B
173. Ans. (a), sin 600° cos 330° + cos120° sin150°
= sin ( 360° + 240° ) cos ( 360° − 30° ) + cos (180° − 60° ) sin (180° − 30° )
= sin 240° cos 30° + ( − cos 60° ) sin 30° = sin (180° + 60° ) cos 30° − cos 60° sin 30°
= − sin 60° cos 30° − cos 60° sin 30° = − ( sin 60° cos 30° + cos 60° sin 30° )
= − sin ( 60° + 30° ) = − sin 90° = −1.
174. Ans. (c), sin 2 A + sin 2 B + sin 2C = 2sin ( A + B ) cos ( A − B ) + sin 2C
π   π
= 2 sin  − C  cos ( A − B ) + 2sin C cos C ∵ A + B + C = 2 
2 
 π 
= 2 cos C cos ( A − B ) + 2sin C cos C = 2 cos C  cos ( A − B ) + sin  − ( A + B ) 
 2 
= 2 cos C cos ( A + B ) + cos ( A − B )  = 2 cos C [ 2 cos A cos B ] = 4 cos A cos B cos C.
175. Ans. (a) sin12°.sin 48°.sin 54°
1 1
= .(2sin12°.sin 48°).sin 54° = .(cos 36° − cos 60°).sin 54.
2 2
1  5 +1 1   5 +1 1  5 − 1  5 + 1  4 1
= . 
2  4
−  . 
2  4 
 = . 
2  4   4  = 32 = 8 .


x y z
176. Ans. (b), Let = = =k
cos θ  2π   2π 
cos  θ +  cos  θ − 
 3   3 
 2π   2π 
Then, x = k cos θ , y = k cos  θ +  , z = k cos  θ − 
 3   3 
  2π   2π   2π 
∴ x + y + z = k cos θ + cos  θ +  + cos  θ −   = k cos θ + 2 cos θ cos 
  3   3   3 

30 Office.: 606 , 6th Floor, Hariom Tower, Circular Road, Ranchi-1, Ph.: 0651-2562523, 9835508812, 8507613968
( ) BY R. K. MALIK’S NEWTON CLASSES 31
 2π  π π 1
= k {cos θ − cos θ } ∵ cos 3 = cos  π − 3  = − cos 3 = − 2 
   
= k . 0 = 0.
sin 2 A − sin 2 B sin 2 A − sin 2 B
177. Ans. (c), =
sin A cos A − sin B cos B 1 sin 2 A − 1 sin 2 B
( ) ( )
2 2
( sin 2
A − sin 2 B )   C + D   C − D 
=
1 ∵ sin C − sin D = 2 cos  2  sin  2  
{2 cos ( A + B ) sin ( A − B )}     
2
sin ( A + B ) sin ( A − B ) sin ( A + B )
= = = tan ( A + B )
cos ( A + B ) sin ( A − B ) cos ( A + B )
 85° + 35°   85° − 35° 
2 cos   sin  
sin 85° − sin 35°  2   2 
178. Ans. (c), =
cos 65° cos 65°
2 cos 60° ⋅ sin 25°
= ∵ cos 65° = sin ( 90° − 65° ) = sin 25° 
sin 25°
1
= 2 cos 60° = 2 × = 1
2
sin ( −660° ) tan (1050° ) sec ( −420° ) sin ( 660° ) tan (1050° ) sec ( 420° )
179. Ans. (b), =
cos ( 225° ) cosec ( 315° ) cos ( 510° ) cos ( 225° ) cosec ( 315° ) cos ( 510° )
− sin ( 720° − 60° ) tan (1080° − 30° ) sec ( 360° + 60° ) − {− sin 60°} ⋅{− tan 30°}{sec 60°}
= =
cos (180° + 45° ) cosec ( 360° − 45° ) cos ( 360° + 150° ) {− cos 45°}{− cosec45°} ⋅{cos150°}
sin 60° tan 30° ⋅ sec 60°
= [∵ cos150° = − cos 30°]
cos 45°. cosec 45°.cos 30°
3 1
. .2
2 3 2
= =
1 3 3
. 2.
2 2
A 5A
180. Ans. (d), 32sin sin = 16 ( cos 2 A − cos 3 A )  Using : 2sin A sin B = cos ( A − B ) − cos ( A + B ) 
2 2
= 16. ( 2 cos 2 A − 1) − ( 4 cos3 A − 3cos A ) 

   3 2    3 3  3     9   27 9  1 9  11
= 16.   2.   − 1 − 4.   − 3.    = 16.   − 1 −  −  = 16.  +  = 16. = 11
   4     4   4     8   16 4   8 16  16

181. Ans. (d), Let x = 4 tan 2 θ and y = 9 cot 2 θ


x+ y
Then, A.M. ≥ G.M. ⇒ ≥ xy [Where x & y both are non-negative]
2
4 tan 2 θ + 9 cot 2 θ
⇒ ≥ 4 tan 2 θ .9 cot 2 θ ⇒ 4 tan 2 θ + 9 cot 2 θ ≥ 12
2
∴ The minimum value of 4 tan 2 θ + 9 cot 2 θ is 12.
182. Ans. (d), Taking each choice separately
(a) sin A = sin {π − ( B + C )} = sin ( B + C ) = sin B cos C + cos B sin C , ∴ Option (a) is true.

Office.: 606 , 6th Floor, Hariom Tower, Circular Road, Ranchi-1, Ph.: 0651-2562523, 9835508812, 8507613968 31
32 ( )BY R. K. MALIK’S NEWTON CLASSES
(b) cos C = cos {π − ( A + B )} = − cos ( A + B ) = sin A sin B − cos A cos B.
∴ Option (b) is true. For any angles A, B and C we have
tan A + tan B + tan C − tan A tan B tan C
(c) tan ( A + B + C ) = …(i)
1 − tan A tan B − tan B tan C − tan C tan A
In a triangle, we have A + B + C = π , ∴ Equation (i) is reduced to
tan A + tan B + tan C − tan A tan B tan C
tan π = ⇒ 0 = tan A + tan B + tan C − tan A tan B tan C
1 − tan A tan B − tan B tan C − tan C tan A
⇒ tan A + tan B + tan C = tan A. tan B. tan C , ∴ Option (c) is true.
(d) However. Option (d) is false for a triangle (It can be shown to be false e.g. for A = B = C = 60° it
is false) .
183. Ans. (c), cos x + cos y + cos α = 0 ⇒ cos x + cos y = − cos α …(i)
sin x + sin y + sin α = 0 ⇒ sin x + sin y = − sin α …(ii)
 x+ y  x− y
2 cos   cos  
cos x + cos y − cos α  2   2  = cot α
Dividing (i) by (ii) we get = ⇒
sin x + sin y − sin α  x+ y  x− y
2sin   cos  
 2   2 
 x+ y
⇒ cot   = cot α .
 2 
4 xy
184. Ans. (b), sec2 θ has a value greater than or equal to 1 i.e., sec2 θ ≥ 1 ⇒ 2
≥1
( x + y)
2 2 2 2 2
⇒ ( x + y ) ≤ 4 xy ⇒ ( x + y ) − 4 xy ≤ 0 ⇒ ( x − y ) ≤ 0 ⇒ ( x − y ) = 0 ∵ ( x − y ) never be − ve 
 
⇒ x = y.
Also, when x = y and x = 0, then the value of sec2 θ becomes undefined
4 xy
Hence, sec2 θ = 2
is true if and only if x = y, x ≠ 0.
( x + y)
A B C  A B  C
185. Ans.(b), A + B + C = 180° ⇒ + = 90° − ⇒ tan  +  = tan  90° − 
2 2 2 2 2  2
A B A B
tan + tan tan + tan
⇒ 2 2 = cot C
⇒ 2 2 = 1
A
1 − tan tan
B 2 A
1 − tan tan
B
tan
C
2 2 2 2 2
A C B C A B A B B C C A
⇒ tan tan + tan tan = 1 − tan tan ⇒ tan tan + tan tan + tan tan = 1
2 2 2 2 2 2 2 2 2 2 2 2
A B
= ∑ tan tan = 1.
2 2
π   π 
186. Ans. (a), tan θ . tan  + θ  .tan  − + θ  = k tan 3θ
3   3 
 π   tan  − π  + tan θ 
 tan 3 + tan θ  
 3
 
⇒ tan θ .   .   = k tan 3θ
π
 1 − tan tan θ   1 − tan  − π  tan θ 
 3    3

32 Office.: 606 , 6th Floor, Hariom Tower, Circular Road, Ranchi-1, Ph.: 0651-2562523, 9835508812, 8507613968
( ) BY R. K. MALIK’S NEWTON CLASSES 33
 3 + tan θ   − 3 + tan θ  2
 tan θ − 3 
⇒ tan θ   .   = k tan 3θ ⇒ tan θ  2  = k tan 3θ
 1 − 3 tan θ   1 + 3 tan θ   1 − 3 tan θ 
 3 tan θ − tan 3 θ 
⇒ − 2  = k tan 3θ ⇒ − tan 3θ = k tan 3θ ⇒ k = −1.
 1 − 3tan θ 
π  π 
Alternatively : We know that tan θ . tan  − θ  .tan  + θ  = tan 3θ
3  3 
π   π 
∴ tan θ . tan  + θ  . tan  − + θ  = − tan 3θ and so, k = −1
3   3 
sin A n sin A − sin B n − 1
187. Ans. (b), sin A = n sin B ⇒ = ⇒ = [By Componendo-dividendo]
sin B 1 sin A + sin B n + 1
 A+ B   A− B   A− B 
2 cos   sin   n −1 tan  
⇒  2   2 = ⇒  2  = n − 1 ⇒ n − 1 tan  A + B  = tan  A − B  .
   
 A+ B   A − B  n +1  A + B  n +1 n +1  2   2 
2sin   cos   tan  
 2   2   2 
188. Ans. (d), cos 2 A + cos 2 B + cos 2C = {2 cos ( A + B ) cos ( A − B )} + ( 2 cos 2 C − 1)
= 2 cos (180° − C ) cos ( A − B ) + 2 cos 2 C − 1 [∵ A + B + C = 180°]
= −2 cos C cos ( A − B ) + 2 cos 2 C − 1 = −2 cos C ⋅ {cos ( A − B ) − cos C } − 1

= −1 − 2 cos C.{cos ( A − B ) − cos (180° − ( A + B ) )} [∵ A + B + C = 180°]

= −1 − 2cos C.{cos ( A − B ) + cos ( A + B )} = −1 − 2 cos C.{2 cos A cos B} = −1 − 4 cos A cos B cos C
 π
189. Ans. (c), Since α ∈  0,  , so tan α ≥ 0
 2
tan 2 α A+ B
Now, if A = x 2 + x and B = , then we have ≥ A. B [∵ AM . ≥ G.M .]
2
x +x 2
1/ 2
1 tan 2 α   2 tan 2 α  tan 2 α
⇒  x2 + x + ≥
  x + x .  ⇒ x2 + x + ≥ 2 tan α .
2 x2 + x   x2 + x  x2 + x
5 −1  5 −1   5 −1  6 − 2 5 3 − 5
190. Ans. (b), sin18° = ⇒ sin 2 18° =   .   = = .
4  4  4  16 8

5 +1  5 +1   5 +1  6 + 2 5 3 + 5
cos 36° = ⇒ cos 2 36° =   .   = = .
4  4   4  16 8
∴ If sin 2 18° and cos 2 36° are the roots of a quadratic equation, then
3− 5 3+ 5 6 3  3− 5   3+ 5  4 1
Sum of the roots = + = = and product of the roots =   .   = = .
8 8 8 4  8   8  64 16
3 1
∴ Required quadratic equation is x 2 −   x + = 0 ⇒ 16 x 2 − 12 x + 1 = 0.
4 16
A B C A B C S1 − S3
191. Ans. (b), A + B + C = 180° ⇒ + + = 90° ⇒ tan  + +  = tan 90° = ∞ ⇒ =∞
2 2 2 2 2 2 1 − S2
A B C A
Where, S1 = tan + tan + tan = ∑ tan
2 2 2 2

Office.: 606 , 6th Floor, Hariom Tower, Circular Road, Ranchi-1, Ph.: 0651-2562523, 9835508812, 8507613968 33
34 ( BY R. K. MALIK’S NEWTON CLASSES
)
 A B A B C S −S
S 2 = ∑  tan ⋅ tan  and S3 = tan tan tan and 1 3 = ∞ ⇒ 1 − S 2 = 0
 2 2 2 2 2 1 − S2
A B
⇒ S2 = 1 ⇒ ∑ tan 2 tan 2 = 1 [Note : Remember this result directly]

192. Ans. (a) sin α = sin β ⇒ sin α − sin β = 0


α + β  α − β 
⇒ 2 cos   sin  =0 …(i)
 2   2 
And cos α = cos β ⇒ cos α − cos β = 0
α + β  α − β 
⇒ − 2 sin   sin  =0 …(ii)
 2   2 
From (i) and (ii) we get:
α − β   α +β α +β 
sin  =0 ∵cos = 0, and sin = 0 both simultanevely is not possible 
 2   2 2 
α − β 
i.e., the two conditions are satisfied only if sin   =0.
 2 
193. Ans. (b) We have: cos 2 A + cos 2 B + cos 2C
= 2 cos( A + B ) cos( A − B ) + (1 − 2 sin 2 C ) = −2sin C cos( A − B ) + 1 − 2sin 2 C
= 1 − 2sin C{cos( A − B ) + sin C} = 1 − 2sin C {cos( A − B ) − cos( A + B )}
= 1 − 2sin C{2 sin A sin B} = 1 − 4sin A sin B sin C
⇒ cos 2 A + cos 2 B + cos 2C + 4sin A sin B sin C = 1 .
sec8 A − 1 (1 − cos8 A ) cos 4 A 2sin 2 4 A cos 4 A
194. Ans. (b), = . = . ∵ 1 − cos 2θ = 2 sin 2 θ 
sec 4 A − 1 cos 8 A (1 − cos 4 A) cos8 A 2sin 2 2 A

=
( 2 sin 4 A. cos 4 A) .sin 4 A =
sin 8 A. ( 2sin 2 A cos 2 A )
=
tan 8 A
.
2
cos 8 A. 2sin 2 A cos 8 A. ( 2 sin 2 A.sin 2 A ) tan 2 A
195. Ans. (b), sin 36° sin 72° sin108° sin144° = sin 36° cos18° cos18° sin 36°
∵ sin 72° = sin ( 90° − 18° ) = cos18°, sin108° = sin ( 90° + 18° ) = cos18° and sin144° 
 
 = sin (180° − 36° ) = sin 36° 
 1
1 1  ∵ sin 2
A = (1 − cos 2 A ) 
2
= (1 − cos 72° ) . (1 + cos 36° )  
2 2 and cos 2 A = 1 (1 + cos 2 A ) 
 2 

1  5 − 1   5 + 1  1 1 5
= 1 −
4 
 1 +
4  
 =
4  64
( 64
)(
5 − 5 5 + 5 = .20 = .
16
)
π   2π  1  π   1    2π  
196. Ans.(d), cos 2  + x  + cos 2  + x  = 1 + cos  2  + x    + 1 + cos  2  + x  
3   3  2  3   2    3  
1   2π   4π  1  π  1
= 1 + cos  + 2 x  + cos  + 2 x   = 1 + 2 cos (π + 2 x ) cos    = 1 − cos ( 2 x ) .
2  3   3  2  3  2
1 1
Now, 1 − cos ( 2 x ) attains a maximum value, When cos ( 2 x ) has the minimum value
2 2
i.e. when cos ( 2x ) has the minimum value i.e., when cos ( 2 x ) = −1.

34 Office.: 606 , 6th Floor, Hariom Tower, Circular Road, Ranchi-1, Ph.: 0651-2562523, 9835508812, 8507613968
( BY R. K. MALIK’S NEWTON CLASSES
) 35
1 1 3
∴ Maximum value of 1 − cos ( 2 x ) = 1 − . ( −1) = and so, the maximum value of
2 2 2
π   2π  3
cos 2  + x  + cos 2  + x  is .
3   3  2
x−h a
197. Ans. (a), Given, x = h + a sec θ ⇒ sec θ = ⇒ cos θ =
a x−h
a2 y−k b b2
⇒ cos 2 θ = 2
and y = k + b cosec θ ⇒ cosec θ = ⇒ sin θ = ⇒ sin 2 θ = 2
( x − h) b y−k ( y −k)
a2 b2
Now, cos 2 θ + sin 2 θ = 1 ⇒ 2 2
=1
( x − h) ( y − k )
π  π 
198. Ans. (a), 4 cos ( x 2 ) cos  + x 2  cos  − x 2 
3  3 
1  π  π  1 
= 4. cos ( 3 x 2 )  Using formula cos θ .cos  3 − θ  cos  3 + θ  = 4 cos 3θ 
4      
= cos ( 3x 2 ) and Clearly, −1 ≤ cos ( 3 x 2 ) ≤ 1 [ We know that − 1 ≤ cos θ ≤ 1]
⇒ The extreme values of the given expression are −1 and 1.
 π    3π  
199. Ans. (a), 6 sin 4 ( 5π + θ ) + sin 6  + θ   − 4 sin 4  + θ  + sin 6 ( 3π + θ ) 
 2    2  
 π  
∵ sin ( 5π + θ ) = − sin θ , sin  2 + θ  = − cos θ , 
 
= 6 {sin 4 θ + cos 4 θ } − 4 {cos 6 θ + sin 6 θ }  
  3π  
sin  + θ  = − cos θ , sin ( 3π + θ ) = − sin θ 
  2  

{ } {
= 6 ( sin 2 θ + cos 2 θ ) − 2sin 2 θ cos 2 θ −4 ( cos 2 θ + sin 2 θ ) − 3sin 2 θ cos 2 θ ( cos 2 θ + sin 2 θ )
2 3
}
= 6 {1 − 2sin 2 θ cos 2 θ } − 4 {1 − 3sin 2 θ cos 2 θ } = 2.
tan β cos θ
200. Ans. (c), tan β = cos θ tan α ⇒ =
tan α 1
tan α − tan β 1 − cos θ
⇒ = [By Componendo and Dividendo]
tan α + tan β 1 + cos θ
sin α sin β θ
− 2sin 2
cos α cos β 2 sin α cos β − cos α sin β θ sin (α − β ) θ
⇒ = ⇒ = tan 2 ⇒ = tan 2 .
sin α sin β
+ 2 cos 2θ sin α cos β + cos α sin β 2 sin (α + β ) 2
cos α cos β 2
3π 4π
201. Ans. (a), cos 2 + cos 2 = cos 2 108° + cos 2 144° = cos 2 ( 90° + 18° ) + cos 2 (180° − 36° )
5 5
2 2
2 2  5 −1   5 + 1  6 − 2 5 6 + 2 5
= ( − sin18° ) + ( − cos 36° ) 2 2
= sin 18° + cos 36° =   +   = +
 4   4  16 16
12 3
= = .
16 4
cos (α − β ) p +1
202. Ans. (d), = ⇒ ( p − 1) cos (α − β ) = ( p + 1) sin (α + β )
sin (α + β ) p −1

Office.: 606 , 6th Floor, Hariom Tower, Circular Road, Ranchi-1, Ph.: 0651-2562523, 9835508812, 8507613968 35
36 ( BY R. K. MALIK’S NEWTON CLASSES
)
sin (α + β ) + cos (α − β )
⇒ p cos (α − β ) − sin (α + β )  = sin (α + β ) + cos (α − β ) ⇒ p =
cos (α − β ) − sin (α + β )
sin α cos β + cos α sin β + cos α cos β + sin α sin β
⇒ p=
cos α cos β + sin α sin β − sin α cos β − cos α sin β
tan α + tan β + 1 + tan α tan β
⇒ p= [ Dividing Nr. and Dr. by cos α cos β ]
1 + tan α tan β − tan α − tan β

⇒ p=
(1 + tan α )(1 + tan β ) =  1 + tan α   1 + tan β  π  π 
⇒ p = tan  + α  .tan  + β  .
(1 − tan α )(1 − tan β )  1 − tan α   1 − tan β  4  4 
203. Ans. (c), sin 4 A − cos 2 A = cos 4 A − sin 2 A ⇒ sin 4 A + sin 2 A = cos 4 A + cos 2 A
 π 
⇒ 2 sin 3 A cos A = 2 cos 3 A cos A ⇒ sin 3 A = cos 3 A ∵ 0 < A < 4 ⇒ cos A ≠ 0 
⇒ tan 3 A = 1 = tan 45°
 π 3π 
⇒ 3 A = 45° ∵ 0 < A < 4 ⇒ 0 < 3 A < 4 and so tan 3 A = 1 ⇒ tan 3 A = tan 45° 

⇒ A = 15° ⇒ 4 A = 60° ⇒ tan 4 A = tan 60° = 3.


 B + 2C + 3 A   A− B   π − A + C + 3A   A− B 
204. Ans. (b), cos   + cos   = cos   + cos   [∵ B + C = π − A]
 2   2   2   2 
 π + C + 2A   A− B   π + (π − B ) + A   A− B 
= cos   + cos   = cos   + cos   [∵ C + A = π − B ]
 2   2   2   2 
 2π + A − B   A− B   A− B   A− B 
⇒ cos   + cos   = cos  π +  + cos  
 2   2   2   2 
 A− B   A− B 
= − cos   + cos   = 0.
 2   2 
cos 2 36° cos 2 72°
205. Ans. (b), We have cot 2 36° cot 2 72° =
sin 2 36° sin 2 72°

=
( 2 cos 2
36° )( 2 cos 2 72° )
=
(1 + cos 72° )(1 + cos144° )
( 2 sin 2
36° )( 2sin 72° )
2
(1 − cos 72° )(1 − cos144° )
=
(1 + cos 72° )(1 − cos 36° ) ∵ cos144° = cos (180° − 36° ) = − cos 36° 
(1 − cos 72° )(1 + cos 36° )
 5 − 1  5 +1
 `1 + 1 −
=  4  4 

=
3+ 5 3− 5 ( )( ) =
9−5
=
4 1
= , Hence, option (b) is correct.

1 −
5 −1  
 1 +
5 +1

5− 5 5+ 5 ( )( ) 25 − 5 20 5
 4  4` 
2 2
 5 + 1 5 −1   1 
 ×     4 2 1
cos 2 36° cos 2 72°  4 4  4
Alternatively: Given = = =  = =
sin 2 36° sin 2 72°  10 − 2 5 10 + 2 5   100 − 20   80  5
   
 4 4   16 
206. Ans. (c) We have:
tan A − tan B = x …(i) cot B − cot A = y …(ii)

36 Office.: 606 , 6th Floor, Hariom Tower, Circular Road, Ranchi-1, Ph.: 0651-2562523, 9835508812, 8507613968
( BY R. K. MALIK’S NEWTON CLASSES
) 37
tan A − tan B x
Dividing (i) by (ii) we get: =
cot B − cot A y
tan A − tan B x x
⇒ = ⇒ tan A tan B = …(iii)
1 1 y y

tan B tan A
tan A − tan B x
Now, tan( A − B ) = = [Using(i) & (iii)]
1 + tan A tan B 1 + x
y
xy x+ y 1 1
⇒ tan( A − B ) = ⇒ cot ( A − B ) = = +
x+ y xy y x

3 −1 BC Perpendicular
207. Ans. (b), ∵ tan A = = =
3 +1 AB Base C

AB 3 +1 3 +1 3 −1
∴ cos A = = =
A
( ) ( ) 2 2
AC 2 2
3 −1 + 3 +1 3 +1 B

1 Perpendicular
208. Ans. (b), ∵ tan θ = =
7 Base
1 1 7
∴ sin θ = = , cos θ = 2 2
7 +1 2 2 2 2 1
θ
2 2
∴ cosecθ = 2 2 and sec θ = 7
7
cosec2θ − sec 2 θ 8 − ( 8 / 7 ) 48 3
∴ = = =
cosec θ + sec θ 8 + ( 8 / 7 )
2 2
64 4
1
209. Ans. (d), Given, tan θ + cot θ = 2 ⇒ tan θ + = 2 ⇒ tan 2 θ + 1 = 2 tan θ
tan θ
2
⇒ tan 2 θ − 2 tan θ + 1 = 0 ⇒ ( tan θ − 1) = 0 ⇒ tan θ = 1 ∴ θ = 45°
1 1 2
∴ sin θ + cos θ = sin 45° + cos 45° = + = = 2
2 2 2
210. Ans. (a), ∵ 2 cos A ⋅ tan B − 2 cos A − tan B + 1 = 0
∴ 2 cos A ( tan B − 1) − ( tan B − 1) = 0 ∴ ( 2 cos A − 1)( tan B − 1) = 0
1
∴ cos A = = cos 60°, tan B = 1 = tan 45° ∴ A = 60°, B = 45°
2
sin 3 θ + cos3 θ sin 3 θ − cos3 θ
211. Ans. (c), Given, +
sin θ + cos θ sin θ − cos θ
( sin θ + cos θ ) ( sin 2 θ − sin θ ⋅ cos θ + cos 2 θ ) ( sin θ − cos θ ) ( sin 2 θ + sin θ ⋅ cos θ + cos 2 θ )
= +
sin θ + cos θ ( sin θ − cos θ )
= 1 − sin θ ⋅ cos θ + 1 + sin θ ⋅ cos θ = 2
212. Ans. (d), ( sec θ + tan θ − 1)( sec θ − tan θ + 1) = sec θ + ( tan θ − 1)  sec θ − ( tan θ − 1) 

= sec 2 θ − ( tan θ − 1)  = sec 2 θ − ( tan 2 θ − 2 tan θ + 1) = sec 2 θ − tan 2 θ + 2 tan θ − 1


2
 
= 1 + 2 tan θ − 1 = 2 tan θ

Office.: 606 , 6th Floor, Hariom Tower, Circular Road, Ranchi-1, Ph.: 0651-2562523, 9835508812, 8507613968 37
38 ( ) BY R. K. MALIK’S NEWTON CLASSES
2 2
213. Ans. (d), ( sin θ + cosec θ ) + ( cos θ + sec θ )
= sin 2 θ + cosec 2 θ + 2 ( sin θ )( cosec θ ) + cos 2 θ + sec 2 θ + 2 ( cos θ )( sec θ )
= ( sin 2 θ + cos 2 θ ) + 2 (1) + 2 (1) + sec 2 θ + cosec2 θ

= (1) + 2 + 2 + (1 + tan 2 θ ) + (1 + cot 2 θ ) = tan 2 θ + cot 2 θ + 7 , ∴ k = 7

214. Ans. (a), ( sin 4 θ + cos 4 θ ) + sin 2 θ cos 2 θ = ( sin 2 θ + cos 2 θ ) − 2sin 2 θ cos 2 θ  + sin 2 θ cos 2 θ
2

 
2
= (1) − 2sin 2 θ cos 2 θ + sin 2 θ cos 2 θ = 1 − sin 2 θ cos 2 θ = 1 − u 2 , ∴ u = sin θ ⋅ cos θ
215. Ans. (d), ( sin θ + cos θ )(1 − sin θ cos θ ) = ( sin θ + cos θ ) ( sin 2 θ + cos 2 θ − sin θ cos θ )

= ( sin θ + cos θ ) ( sin 2 θ − sin θ cos θ + cos 2 θ ) = sin 3 θ + cos3 θ = sin n θ + cos n θ
∴ n=3
216. Ans. (a), ∵ p ⋅ cos θ = q 2 − p 2 ⋅ sin θ
p Perpendicular
∴ tan θ = = q
q2 − p2 Base P
θ
Perpendicular p p
∴ sin θ = = = q2 − p2
Hypoteneous p + (q − p
2 2 2
) q

∴ q ⋅ sin θ = p
217. Ans. (a), ∵ sin θ = 3 ⋅ cos θ ∴ tan θ = 3 ∴ θ = 60°
 3 1
∴ 2 ( sin θ + cos θ ) − 1 = ( sin 60° + cos 60° ) − 1 = 2  +  − 1 = 3 + 1 − 1 = 3
 2 2
1 2
218. Ans. (d), sec θ + cos θ = 2 ⇒ + cos θ = 2 ⇒ cos 2 θ − 2 cos θ + 1 = 0 ⇒ ( cos θ − 1) = 0
cos θ
⇒ cos θ = 1 , ∴ sec 2 θ − sec4 θ = 1 − 1 = 0, Hence, option (d) is correct.
1 2
219. Ans. (a), tan θ + cot θ = 2 ⇒ tan θ + = 2 ⇒ ( tan θ − 1) = 0 ⇒ tan θ = 1
tan θ
∴ tan 2 θ − tan 3 θ = 1 − 1 = 0 , Also, in option (a) cot 3 θ − cot 2 θ = 1 − 1 = 0
220. Ans. (c), ∵ 1 + sin α ⋅ sin β − cos α ⋅ cos β = 0 ∴ cos α ⋅ cos β − sin α ⋅ sin β = 1
∴ cos (α + β ) = 1 ∴ α + β = 2nπ ∴ β = 2nπ − α
tan α tan α tan α
∴ tan α ⋅ cot β = = = = −1
tan β tan ( 2nπ − α ) − tan α
221. Ans. (b), ∵ A + B + C = π
B+C π A A+ B π C B+C π A A
∴ = − and = − ∴ tan = tan  −  = cot
2 2 2 2 2 2 2 2 2 2
A+ B π C  C A B+C A+ B C
cos = cos  −  = sin ∴ given expression = tan ⋅ tan + cos ⋅ cosec
2 2 2 2 2 2 2 2
 Α A  C C
=  tan ⋅ cot  +  sin ⋅ cosec  = 1 + 1 = 2
 2 2  2 2
222. Ans. (c), x 2 + y 2 + z 2 = r 2 ⋅ cos 2 A ⋅ cos 2 B + r 2 ⋅ cos 2 A ⋅ sin 2 B + r 2 ⋅ sin 2 A
= r 2 ⋅ cos 2 A ( cos 2 B + sin 2 B ) + r 2 ⋅ sin 2 A = r 2 cos 2 A + r 2 sin 2 A = r 2 ( cos 2 A + sin 2 A ) = r 2

38 Office.: 606 , 6th Floor, Hariom Tower, Circular Road, Ranchi-1, Ph.: 0651-2562523, 9835508812, 8507613968
( ) BY R. K. MALIK’S NEWTON CLASSES 39
223. Ans. (a), ∵ x = a ⋅ cos θ + b ⋅ sin θ ∴ x − a = ( a ⋅ cos θ + b ⋅ sin θ ) − a
2 2 2 2

= b ⋅ sin 2 θ − a (1 − cos 2 θ ) = b ⋅ sin 2 θ − a ⋅ sin 2 θ = ( b − a ) ⋅ sin 2 θ

and b − x = b − ( a ⋅ cos 2 θ + b ⋅ sin 2 θ ) = b (1 − sin 2 θ ) − a ⋅ cos 2 θ = b ⋅ cos 2 θ − a ⋅ cos 2 θ


= ( b − a ) ⋅ cos 2 θ

( x − a )( b − x ) = ( b − a ) ⋅ sin 2 θ ⋅ ( b − a ) ⋅ cos 2 θ = ( b − a )
2 2
∴ ⋅ sin 2 θ ⋅ cos 2 θ = ( a − b ) ⋅ sin 2 θ ⋅ cos 2 θ

∵ ( x − a )( b − x ) = c 2 sin 2 θ ⋅ cos 2 θ
2
∴ c 2 = ( a − b ) ∴ c = a − b or b − a
224. Ans. (b), Given, x = cos ( A + B ) ⋅ cos ( A − B ) and y = sin ( A + B ) ⋅ sin ( A − B )
∴ x − y = cos ( A + B ) ⋅ cos ( A − B ) − sin ( A + B ) ⋅ sin ( A − B )
= cos ( A + B ) + ( A − B )  = cos 2 A [∵ cos α ⋅ cos β − sin α ⋅ sin β = cos (α + β ) ]

π  π  π  π 
225. Ans. (a), Given, x = sin  + A  ⋅ cos  + B  and y = cos  + A  ⋅ sin  + B 
3  3  3  3 
π  π  π  π 
∴ x − y = sin  + A  ⋅ cos  + B  − cos  + A  ⋅ sin  + B 
3  3  3  3 
 π  π 
= sin  + A  −  + B   = sin ( A − B ) [∵ sin α ⋅ cos β − cos α ⋅ sin β = sin (α + β ) ]
 3  3 
A A A
sin
cos A ⋅ cos + sin A ⋅ sin
 A sin A 2 = 2 2
226. Ans. (d), 1 + tan A ⋅ tan   = 1+ ⋅
 2 cos A cos A cos A ⋅ cos
A
2 2
 A A
cos  A −  cos
 2 = 2 1
= = = sec A
cos A ⋅ cos
A
cos A ⋅ cos
A cos A
2 2
sin ( A − B )
sin A ⋅ cos B − cos A ⋅ sin B
227. Ans. (b), ∵ x ⋅ cos A ⋅ cos B = sin ( A − B ) ∴ x = =
cos A ⋅ cos B cos A ⋅ cos B
∴ x = tan A − tan B , similarly, y = tan B − tan C and z = tan C − tan A
∴ x + y + z = tan A − tan B + tan B − tan C + tan C − tan A = 0
228. Ans. (c), Given, x = sin ( B − C ) ⋅ cos A , y = sin ( C − A) ⋅ cos B, and z = sin ( A − B ) ⋅ cos C
⇒ x = sin ( B − C ) ⋅ cos A = ( sin B ⋅ cos C − cos B ⋅ sin C ) ⋅ cos A
= cos A ⋅ cos B ⋅ cos C − cos A ⋅ cos B ⋅ cos C , Similarly, y = cos A ⋅ cos B ⋅ sin C − cos B ⋅ cos C ⋅ sin A
and z = sin A ⋅ cos B ⋅ cos C − cos B ⋅ cos C ⋅ sin A
So, x + y + z = cos A ⋅ cos B ⋅ cos C − cos A ⋅ cos B ⋅ cos C + cos A ⋅ cos B ⋅ sin C − cos B ⋅ cos C ⋅ sin A
+ sin A ⋅ cos B ⋅ cos C − cos B ⋅ cos C ⋅ sin A
=0
229. Ans. (d), Given, 2 ⋅ sin ( x + 60° ) = cos ( x − 30° )
⇒ 2 [sin x ⋅ cos 60° + cos x ⋅ sin 60°] = cos x ⋅ cos 30° + sin x ⋅ sin 30°
 1 3 3 1 1
⇒ 2  sin x ⋅ + cos x ⋅
2
 = cos x ⋅
2  2
+ sin x ⋅
2
∴ sin x + 3 ⋅ cos x =
2
( 3 ⋅ cos x + sin x )

Office.: 606 , 6th Floor, Hariom Tower, Circular Road, Ranchi-1, Ph.: 0651-2562523, 9835508812, 8507613968 39
40 ( ) BY R. K. MALIK’S NEWTON CLASSES
⇒ 2 sin x + 2 3 cos x = 3 cos x + sin x ⇒ sin x = − 3 ⋅ cos x ∴ tan x = − 3
230. Ans. (b), ∴ cos x + cos (120° − x ) + cos (120° + x ) = 2 ⋅ cos120° ⋅ cos x + cos x
= 2 ⋅ cos ( 90° + 30° ) ⋅ cos x + cos x = 2 ( − sin 30° ) ⋅ cos x + cos x
 1
= 2  −  ⋅ cos x + cos x = − cos x + cos x = 0
 2
1 + tan A 1 − tan A
231. Ans. (c), tan ( 45° + A ) ⋅ tan ( 45° − A ) = ⋅ =1
1 − tan A 1 + tan A
232. Ans. (d), tan 75° + tan15° = tan ( 45° + 30° ) + tan ( 45° − 30° )
1 1
( ) ( )
2 2
1−1+ 3 +1 + 3 −1
1 + tan 30° 1 − tan 30° 3+ 3 = 3 +1 + 3 −1 = 8
= + = = =4
1 − tan 30° 1 + tan 30° 1 − 1 1 + 1 3 −1 3 +1 3 −1 2
3 3
233. Ans. (b), ∵ a = tan x ∴ c = 1 + a 2 = sec x , b = tan y ∴ d = 1 + b 2 = sec y
sin x ⋅ cos y + cos x ⋅ sin y
∴ cd ⋅ sin ( x + y ) = ( sec x )( sec y )( sin x ⋅ cos y + cos x ⋅ sin y ) =
cos x ⋅ cos y
= tan x + tan y = a + b
π  π  π  π π  π 
234. Ans. (c), ∵ cos 2  − θ  = sin 2  −  − θ   = sin 2  − + θ  = sin 2  + θ  …(i)
4  2  4  2 4  4 
π  π  π  π 
∴ cos 2  − θ  + cos 2  + θ  = sin 2  + θ  + cos 2  + θ  = 1 [Using (i)]
4  4  4  4 
11π  π  π
235. Ans. (c), ∵ cos = cos  π −  = − cos …(i)
12  12  12
9π  3π  3π
and cos = cos  π −  = − cos …(ii)
2  12  12
π 3π 9π 11π π 3π 3π π π 6π
∴ cos + cos+ cos + cos = cos + cos − cos − cos = 0 = cos = cos
12 12 12 12 12 12 12 12 2 12
Hence, option (c) is correct.
9π π π  π
236. Ans. (a), ∵ cot = cot  −  = tan
20  2 20  20
7π  π 3π  3π 5π π
cot = cot  −  = tan and cot = cot
20  2 20  20 20 4
 π π   3π 3π  π
∴ Given =  cot ⋅ tan  ⋅  cot ⋅ tan  ⋅ tan = 1×1× 1 = 1
 20 20   20 20  4
7π  π π 5π  3π  3π
237. Ans. (b), ∵ sin = sin  π −  = sin and sin = sin  π −  = sin
8  8 8 8  8  8
 π 3π 
∴ Given Expression = 2  sin 2 + sin 2 
 8 8 
3π π π  π
But, sin = sin  −  = cos …(i)
8 2 8 8
 π π
∴ Given Expression = 2  sin 2 + cos 2  = 2 (1) = 2 [Using (i)]
 8 8

40 Office.: 606 , 6th Floor, Hariom Tower, Circular Road, Ranchi-1, Ph.: 0651-2562523, 9835508812, 8507613968
( ) BY R. K. MALIK’S NEWTON CLASSES 41
θ θ θ
2 cos 2 + 2 sin ⋅ cos
238. Ans. (d),
1 + sin θ + cos θ
=
(1 + cos θ ) + sin θ
= 2 2 2 = cot θ
1 + sin θ − cos θ (1 − cos θ ) + sin θ 2 sin 2 θ + 2 sin θ ⋅ cos θ 2
2 2 2
1 − tan 2 θ
239. Ans. (a), ∵ We know that, = cos 2θ
1 + tan 2 θ
1 − tan 2 ( 45° − A )
∴ = cos  2 ( 45° − A )  = cos ( 90° − 2 A ) = sin 2 A
1 + tan 2 ( 45° − A )
sin 3θ cos 3θ sin 3θ ⋅ cos θ − cos 3θ ⋅ sin θ
240. Ans. (d), − =
sin θ cos θ sin θ ⋅ cos θ
sin ( 3θ − θ ) sin 2θ 2sin θ ⋅ cos θ
= = = =2
sin θ ⋅ cos θ sin θ ⋅ cos θ sin θ ⋅ cos θ
sin 2θ cos 2θ sin 2θ cos θ − cos 2θ sin θ sin ( 2θ − θ ) sin θ
241. Ans. (a), − = = = = sec θ
sin θ cos θ sin θ ⋅ cos θ sin θ ⋅ cos θ sin θ ⋅ cos θ
242. Ans. (d), 4 sin θ cos3 θ − 4 cos θ sin 3 θ = 4sin θ cos θ ( cos 2 θ − sin 2 θ )
= 2 ( 2sin θ cos θ )( cos 2θ ) = 2 sin 2θ ⋅ cos 2θ = sin 4θ
243. Ans. (b), 1 − 8 cos 2 θ + 8 cos 4 θ = 1 − 8 cos 2 θ (1 − cos 2 θ )

= 1 − 8 cos 2 θ ( sin 2 θ ) = 1 − 2 ( 4sin 2 θ cos 2 θ ) = 1 − 2sin 2 2θ = cos ( 2 × 2θ ) = cos 4θ


244. Ans. (b), cos 3 A + cos 5 A + cos 7 A + cos15 A = ( cos15 A + cos 3 A ) + ( cos 7 A + cos 5 A )
= 2 cos 9 A ⋅ cos 6 A + 2 cos 6 A ⋅ cos A = 2 cos 6 A ⋅ ( cos 9 A + cos A ) = 2 cos 6 A ⋅ ( 2 cos 5 A ⋅ cos 4 A )
= 4 ⋅ cos 4 A ⋅ cos 5 A ⋅ cos 6 A ⇒ m + n + p = 4 + 5 + 6 =15
245. Ans. (c), Given, sin 40° − cos 70° = k ⋅ cos80° ⇒ sin 40° − cos ( 90° − 20° ) = k ⋅ cos80°
 3
⇒ sin 40° − sin 20° = k ⋅ cos 80° ⇒ 2 cos 30° ⋅ sin10° = k cos 80° ⇒ 2   = k ⇒ k = 3
 2 
246. Ans. (c), cos 20° + ( cos100° + cos140° ) = cos 20° + 2 cos120° cos 20°
= cos 20° + 2 cos ( 90° + 30° ) .cos 20° = cos 20° + 2 ( − sin 30° ) ⋅ cos 20°
 1
= cos 20° + 2  −  ⋅ cos 20° = cos 20° − cos 20° = 0
 2
247. Ans. (a), cos 55° + cos 65° + cos175° = cos ( 90° − 35° ) + cos ( 90° − 25° ) + cos (180° − 5° )
1
= ( sin 35° + sin 25° ) − cos 5° = 2 sin 30° ⋅ cos 5° − cos 5° = 2   ⋅ cos 5° − cos 5° = cos 5° − cos 5° = 0
2
248. Ans. (c), sin10° + sin 20° + sin 40° + sin 50° = ( sin 50° + sin10° ) + ( sin 40° + sin 20° )
1 1
= 2 sin 30° ⋅ cos 20° + 2sin 30° cos10° = 2   ⋅ cos 20° + 2   ⋅ cos10°
2 2
= cos 20° + cos10° = cos ( 90° − 70° ) + cos ( 90° − 80° ) = sin 70° + sin 80°
cos 21° − sin 21° 1 − tan 21° tan 45° − tan 21°
249. Ans. (b), = = [Divide N, D by cos 21° ]
sin 21° + sin 21° 1 + tan 21° 1 + tan 45° ⋅ tan 21°
= tan ( 45° − 21° ) = tan 24° = tan ( 90° − 66° ) = cot 66°

Office.: 606 , 6th Floor, Hariom Tower, Circular Road, Ranchi-1, Ph.: 0651-2562523, 9835508812, 8507613968 41
42 ( BY R. K. MALIK’S NEWTON CLASSES
)
1 1
250. Ans. (a), sin 7θ ⋅ sin θ + sin11θ ⋅ sin 3θ = ( 2sin 7θ ⋅ sin θ ) + ( 2sin11θ sin 3θ )
2 2
1 1 1
= ( cos 6θ − cos8θ ) + ( cos8θ − cos14θ ) = ( cos 6θ − cos14θ )
2 2 2
1
= ( 2sin10θ ⋅ sin 4θ ) = sin 4θ ⋅ sin10θ
2
π 2π 3π 4π 5π  π 3π 5π  2π 4π
251. Ans. (c), 16 cos cos cos cos cos = 4  4 cos cos cos  cos cos
12 12 12 12 12  12 12 12  12 12
= 4 ( 4 cos15° cos 45° cos 75°) cos 30° cos 60°
= 4 ( 4 cos15° cos(60° − 15°) cos(60° + 15°) ) cos 30° cos 60°
= 4 cos ( 3 ×15° ) × cos 30°× cos 60° [∵ We know that 4 cos θ ⋅ cos ( 60° − θ ) ⋅ cos ( 60° + θ ) = cos 3θ ]
1 3 1 3 3
= 4× × × = =
2 2 2 2 2
252. Ans. (c), Given expression, ( sin 2 A + sin 2 B ) − sin 2C = 2sin ( A + B ) ⋅ cos ( A − B ) − sin 2C
= 2sin C ⋅ cos ( A − B ) − 2sin C ⋅ cos C = 2 sin C ⋅ cos ( A − B ) − cos C 
= 2 sin C ⋅ cos ( A − B ) −  − cos ( A + B )   = 2 sin C ⋅  cos ( A + B ) + cos ( A − B ) 
= 2sin C ⋅ ( 2 cos A ⋅ cos B ) = 4 cos A ⋅ cos B ⋅ sin C
253. Ans. (b), ( sin 2 A + sin 2 B ) + sin 2 ( A − B )
= 2sin ( A + B ) ⋅ cos ( A − B ) + 2sin ( A − B ) ⋅ cos ( A − B ) = 2 cos ( A − B ) ⋅ sin ( A + B ) + sin ( A − B ) 
= 2 cos ( A − B ) ⋅ ( 2sin A ⋅ cos B ) = 4sin A ⋅ cos B ⋅ cos ( A − B )

( tan A + cot 2 A )
2
254. Ans. (c), tan 4 A + cot 4 A + 2 = tan 4 A + cot 4 A + 2 tan 2 A cot 2 A = 2

1 1
= tan 2 A + cot 2 A = ( sec2 A − 1) + ( cosec 2 A − 1) = ( sec 2 A + cosec2 A ) − 2 = 2
+ 2 −2
cos A sin A
sin 2 A + cos 2 A 1
= 2 2
−2 = 2 2
− 2 = sec 2 A ⋅ cosec2 A − 2
sin A cos A sin A cos A
2 2

255. Ans. (b), Given Expression =


1 + tan θ 1 − tan θ
− =
(1 + tan θ ) − (1 − tan θ )
1 − tan θ 1 + tan θ (1 − tan 2 θ )
4 tan θ  2 tan θ 
= = 2 = 2 ⋅ tan ( 2θ ) = m ⋅ tan ( nθ ) ∴ m = 2, n = 2 ∴ m = n
(1 − tan θ )  1 − tan 2 θ 
2

π θ  π θ    π θ 
256. Ans. (a), Given expression, sin 2  +  − cos 2  +  = − cos  2  +  
 4 2  4 2   4 2 
[ ∵ We know that cos 2 x − sin 2 x = cos 2 x ]
π 
= − cos  + θ  = − ( − sin θ ) = sin θ
2 
a b
257. Ans. (c), Given, cot θ = ⇒ tan θ = …(i)
b a
 1 − tan 2 θ   2 tan θ 
∴ Given expression, a cos 2θ + b sin 2θ = a  2  + b 2 
 1 + tan θ   1 + tan θ 

th
42 Office.: 606 , 6 Floor, Hariom Tower, Circular Road, Ranchi-1, Ph.: 0651-2562523, 9835508812, 8507613968
( ) BY R. K. MALIK’S NEWTON CLASSES 43

 1 − tan 2 θ 2 tan θ 
 ∵ We know that cos 2θ = and sin 2θ =

2
1 + tan θ 1 + tan 2 θ 
  b 2   b
1 −    b  2 ⋅ 
= a   2  + 
a a
2
[Using (i)]
 b  b
1 +
    1 +  
 a  a
a ( a 2 − b 2 ) + 2ab 2 a  a 2 − b 2 + 2b 2  a ( a2 + b2 )
= = = =a
(a 2
+ b2 ) (a 2
+ b2 ) (a 2
+ b2 )
π 9π 3π 5π
258. Ans. (b) 2 cos cos + cos + cos
13 13 13 13
10π 8π 3π 5π  3π   5π   3π  5π
= cos + cos + cos + cos = cos  π −  + cos  π −  + cos   + cos
13 13 13 13  13   13   13  13
3π 5π 3π 5π
= − cos − cos + cos + cos =0
13 13 13 13
259. Ans. (c), Given that, radius r = 3 m and arc d = 1 m
arc 1
We know that, angle = = rad
ardius 3
260. Ans. (a), ∵ We know that cos ( 540° − θ ) = cos ( 3π − θ ) = − cos θ
 π 
and sin ( 630° − θ ) = sin  7 − θ  = − cos θ
 2 
∴ Given expression = − cos θ − ( − cos θ ) = − cos θ + cos θ = 0
1 1 − tan 2 25° 1 − x 2
261. Ans. (a), Given = tan 40° = cot 50° = = =
tan 50° 2 tan 25° 2x
Alternatively ∵ tan155° = tan (180° − 25° ) = − tan 25° = − x
−1 1
and tan115° = tan ( 90° + 25° ) = − cot 25° = =−
( tan 25° ) x
 −1 
−x −   2
∴ Given expression =  x  = 1− x
 −1  2x
1+ (−x)  
 x 
262. Ans. (b), ∵ sin (120° − α ) = sin (120° − β ) [∵ We know that sin θ = sin (180° − θ ) ]
π
∴ 120° − α = 120° − β or 120° − α = 180° − (120° − β ) ⇒ α = β or α + β = 60° =
3
263. Ans. (b), Given, 3sin 2θ = 5 + 4 cos 2θ
2
 2 tan θ   1 − tan θ   2 tan θ 1 − tan 2 θ 
⇒ 3 −
 4 =5 ∵ We know that sin 2θ = and cos 2θ =
2
 1 + tan θ
2
  1 + tan θ   1 + tan 2 θ 1 + tan 2 θ 
2
⇒ 6 tan θ − 4 + 4 tan 2 θ = 5 + 5 tan 2 θ ⇒ tan 2 θ − 6 tan θ + 9 = 0 ⇒ ( tan θ − 3) = 0 ⇒ tan θ = 3
264. Ans. (a), Given Expression = tan (160° − 110° ) = tan ( 50° ) = tan ( 90° − 40° ) = cot 40°

Office.: 606 , 6th Floor, Hariom Tower, Circular Road, Ranchi-1, Ph.: 0651-2562523, 9835508812, 8507613968 43
44 ( BY R. K. MALIK’S NEWTON CLASSES
)
1
2 1− 2
1 1 1 − tan 20° p p2 −1  1
= = = = = ∵ p = cot 20° ⇒ tan 20° = 
tan ( 2 × 20° ) 2 tan 20° 2 tan 20° 1 2p  p
2 2 
1 − tan 20°  p
sin ( x + y ) a+b
265. Ans. (b), Given, =
sin ( x − y ) a −b
sin ( x + y ) + sin ( x − y ) (a + b) + ( a − b)
⇒ = [By Componendo and Dividendo]
sin ( x + y ) − sin ( x − y ) (a + b) − ( a − b)
2sin x ⋅ cos y 2a tan x a
⇒ = ⇒ =
2 cos x ⋅ sin y 2b tan y b
1
266. Ans. (c), We note that sin15° ⋅ cos15° = ( 2 ⋅ sin15° ⋅ cos15° )
2
1 1 1 1
= ( sin 30° ) =   = , which is a rational number, Hence option (c) is correct
2 22 4
3 −1
Note that sin15° = , clearly not a rational number
2 2
3 +1
cos15° = , clearly not a rational number
2 2
2
 3 −1  2
and sin15° ⋅ cos 75° = sin15° ⋅ sin15° = sin 15° =   , Hence, options (a), (b), (d) are wrong.
 2 2 

267. Ans. (b), Given Expression =


1 − cos α
+
1 + cos α
=
(1 − cos α ) + (1 + cos α )
1 + cos α 1 − cos α 1 − cos 2 α
2 2 2
= = = , Since sin α is negative in 3rd quadrant = −2cosecα
sin α 2 sin α − sin α
268. Ans. (d), ∵ cot (α + β ) = 0 = cot 90° ⇒ α + β = 90°
∴ sin (α + 2 β ) = sin (α + β ) + β  = sin ( 90° + β ) = cos β
269. Ans. (c), Given, cosec θ − cot θ = p …(i)
1 1
As we know that cosec θ + cot θ = ⇒ cosec θ + cot θ = …(ii)
cosec θ − cot θ p
1 1 1
∴ Adding (i) and (ii) we get, 2 cosec θ = p + ⇒ cosec θ =  p + 
p 2 p
270. Ans. (d), Given, sec θ − tan θ = x …(i)
1 1
As we know that sec θ + tan θ = ⇒ sec θ + tan θ = …(ii)
sec θ − tan θ x
1 x2 + 1 2x
Now, adding we get, 2 sec θ = x + = ∴ cos θ = 2
x x x +1

 2 x  ( x + 1) − ( 2 x ) ( x 2 + 1 − 2 x )( x 2 + 1 + 2 x )  1 − x 2 
2 2 2 2
2
2 2
Then sin θ = 1 − cos θ = 1 −  2  = = = 2 
( x2 + 1) ( x2 + 1)
2 2
 x +1  1+ x 

1 − x2
⇒ sin θ =
1 + x2
th
44 Office.: 606 , 6 Floor, Hariom Tower, Circular Road, Ranchi-1, Ph.: 0651-2562523, 9835508812, 8507613968
BY R. K. MALIK’S NEWTON CLASSES
( ) 45
271. Ans. (b), Given ⇒ cos105° + sin105° = cos ( 90° + 15° ) + sin ( 90° + 15° )
 3 −1  3 +1 2 1
= − sin15° + cos15° = −   + = = = cos 45° , Hence, option (b) is correct.
 2 2  2 2 2 2 2
Alternatively : cos105° + sin105° = cos105° + sin ( 90° + 15° ) = cos105° − cos15°
 105° + 15°   105° − 15°  1 1 1
= 2 sin   ⋅ sin   = 2 sin 60° ⋅ sin 45° = 2 × × = = cos 45°
 2   2  2 2 2
Hence, option (b) is correct.
  A    A  5 A 
272. Ans. (c), Given Expression = 8  2 cos 2    − 16  2sin   ⋅ sin  
  2   2  2 
= 8 (1 + cos A ) − 16 ( cos 2 A − cos 3 A ) = 8 (1 + cos A ) − 16  2 cos 2 A − 1 − 16 ( 4 cos3 A − 3cos A )

 3   3 2    3 3 3
= 8  1 +  − 16  2 ×   − 1 − 16  4   − 3 ×  = 14 − (18 − 16 − 27 − 36 ) = 3
 4   4     4  4 
π 1 π
273. Ans. (c), ∵ sin (α + β ) = 1 ∴ α + β = …(i) and sin (α − β ) = ∴ α −β = …(ii)
2 2 6
π π
Solving (i) and (ii) we get, α = and β =
3 6
π π   2π π 
∴ tan (α + 2 β ) ⋅ tan ( 2α + β ) = tan  +  ⋅ tan  + 
3 3  3 6
2π  5π  π π  π  π  1 

=  tan

 tan
3  6 

 3

 = tan  π −  ⋅ tan  π −  =  − tan   − tan  =  −
 6  3  6   − 3 =1
3
( )
2
274. Ans. (c), Given, sin 25° + cos 25° = p ⇒ ( sin 25° + cos 25° ) = p 2 [On squaring]
∴ ( sin 2
25° + cos 2 25° ) + ( 2 sin 25° ⋅ cos 25° ) = p 2 ⇒ 1 + sin ( 2 × 25° ) = p 2 ⇒ sin 50° = p 2 − 1

⇒ cos 50° = + 1 − sin 2 50° = 1 − ( p 2 − 1) = 1 − ( p 4 − 2 p 2 + 1) = 1 − p 4 + 2 p 2 − 1


2

= 2 p2 − p4 = p ⋅ 2 − p2
c
π  c
275. Ans. (b), In the first quadrant, as the value of x goes from 0 = 0° to 90° =   the value of sin x
2
increase from 0 to 1. Now, 1 ≈ 57° so that 1° < 1 and hence, sin1° < sin1
c c c

276. Ans. (b), Given =


1 − sin x
+
1 + sin x
=
1 − sin x
+
1 + sin x
=
(1 − sin x ) + (1 + sin x ) = 2
1 + sin x 1 − sin x 1 + sin x 1 − sin x (1 − sin 2 x ) cos 2 x

2 2
= = = −2sec x [ ∵ cos x < 0 in 2nd quadrant]
cos x − cos x
277. Ans. (b), We know that sin x ⋅ cos y − cos x ⋅ sin y = sin ( x − y )
 π  π 
We get, sin  + A  −  + B   = sin ( A − B )
 3  3 
π π π
278. Ans. (d), ∵ 0 < B < ⇒− < − B < 0 also 0 < A <
2 2 2
π π
Hence, − < A− B < ⇒ A − B lies in 1st or 4th quadrant
2 4

Office.: 606 , 6th Floor, Hariom Tower, Circular Road, Ranchi-1, Ph.: 0651-2562523, 9835508812, 8507613968 45
46 BY R. K. MALIK’S NEWTON CLASSES
( )
279. Ans. (a), Given, A and B in the 1st quadrant, so all six ratios are positive, ∴ cos A > 0 and sin B > 0
2 2
∴ cos A = + 1 − sin 2 A = 1 − ( 3 / 5 ) = 4 / 5 , Also sin B = + 1 − cos 2 B = 1 − ( 9 / 41) = 40 / 41
∴ sin ( A − B ) = sin A ⋅ cos B − cos A ⋅ sin B = ( 3 / 5)( 9 / 41) − ( 4 / 5 )( 40 / 41) = −133 / 205
 sin A sin ( A / 2 ) 
280. Ans. (a), 1 + tan A ⋅ tan ( A / 2 ) = 1 +  ⋅ 
 cos A cos ( A / 2 ) 
cos A ⋅ cos ( A / 2 ) + sin A ⋅ sin ( A / 2 ) cos ( A − A / 2 ) 1
= = = = sec A
cos A ⋅ cos ( A / 2 ) cos A ⋅ cos ( A / 2 ) cos A
sin ( A − B )
sin A ⋅ cos B − cos A ⋅ sin B sin A sin B
281. Ans. (a), ∵ = = − = tan A − tan B
cos A ⋅ cos B cos A ⋅ cos B cos A cos B
sin ( B − C )
Similarly, simplify the other two terms. We get, = tan B − tan C and
cos B cos C
sin ( C − A )
= tan C − tan A , Then, the given expression becomes
cos C ⋅ cos A
= ( tan A − tan B ) + ( tan B − tan C ) + ( tan C − tan A) = 0
282. Ans. (d), ∵ 2sin ( x + 60° ) = cos ( x − 30° )
 1 3 3 1
⇒ 2 ( sin x ⋅ cos 60° + cos x ⋅ sin 60° ) = 2  sin x ⋅ + cos x ⋅  = cos x ⋅ + sin x ⋅
 2 2  2 2

1 3
∴ sin x ⋅ = − cos x ⋅ ∴ tan x = − 3
2 2
283. Ans. (c), ∵ sin (θ + φ ) = 2 ⋅ sin (θ − φ ) ∴ sin θ ⋅ cos φ + cos θ ⋅ sin φ = 2 sin θ ⋅ cos φ − 2 cos θ ⋅ sin φ
⇒ 3cos θ ⋅ sin φ = sin θ ⋅ cos φ ⇒ 3 tan φ = tan θ ⇒ tan θ = 3 tan φ , So clearly, k = 3
sin θ ⋅ sin φ cos θ ⋅ cos φ 3
284. Ans. (b), ∵ 3 tan θ ⋅ tan φ = 1 ⇒3 =1 ⇒ =
cos θ ⋅ cos φ sin θ ⋅ sin φ 1
cos θ ⋅ cos φ + sin θ ⋅ sin φ 3 + 1 cos (θ − φ )
By Componendo and dividendo, we get, = ⇒ =2
cos θ ⋅ cos φ − sin θ ⋅ sin φ 3 − 1 cos (θ + φ )
285. Ans. (b), We have cot 54° = cot ( 90° − 36° ) = tan 36° and tan 20° = tan ( 90° − 70° ) = cot 70°
cot 54° tan 20° tan 36° cot 70°
∴ Given expression = + = + = 1+1 = 2
tan 36° cot 70° tan 36° cot 70°
π π π  π  π 
286. Ans. (a), Use +θ = −  − θ  , we get, given expression = cos 2  − θ  + sin 2  − θ  = 1
4 2 4  4  4 
3ππ π  3π  π π  π 3π 3π π
287. Ans. (c), cot cot cot  π −
cot  cot  −  = cot ⋅ cot × tan × tan =1
20 20 4  20   2 20  20 20 20 20
288. Ans. (d), Given Expression
= ( sin 2 5° + sin 2 85° ) + ( sin 2 10° + sin 2 80° ) + ...... + ( sin 2 40° + sin 2 50° ) + sin 2 45° + sin 2 90°
2
 1 
= ( sin 5° + cos 5° ) + .... + ( sin 40° + cos 40° )  + 
2
 + (1)
2 2 2 2

 2
1 3 19 1
= [1 + 1 + 1 + ... + 8 times ] + + 1 = 8 + = =9
2 2 2 2
289. Ans. (c), ∵ θ is in second quadrant ∴ cos θ < 0 and sin θ > 0

th
46 Office.: 606 , 6 Floor, Hariom Tower, Circular Road, Ranchi-1, Ph.: 0651-2562523, 9835508812, 8507613968
( ) BY R. K. MALIK’S NEWTON CLASSES 47

25 12  12   −5  120
∴ sin θ = + 1 − cos 2 θ = 1 − = ∴ sin 2θ = 2sin θ ⋅ cos θ = 2     = −
169 13  13   13  169
θ 
290. Ans. (c), ∵ 270° < θ < 360° ∴ 135° <   < 180°
2
∴ (θ / 2 ) lies in the second quadrant ∴ sin (θ / 2 ) > 0 and cos (θ / 2 ) < 0

θ  1 + cos θ 1+ 3 / 7 5
∴ cos   = − =− =−
2 2 2 7
π  π  π 3
291. Ans. (b), sin  − A  ⋅ sin  + A  = sin 2 − sin 2 A = − sin 2 A
3  3  3 4
3 
∴ Given Expression = 4 sin A  − sin 2 A  = 3sin A − 4 sin 3 A = sin 3 A
4 
292. Ans. (d), Given Expression
1 1 1 1 1 1 1
= ( cos 20°⋅ cos 40°⋅ cos 80° ) ⋅ cos 60° = cos ( 3 × 20° ) ⋅ = ⋅ cos ( 3 × 20° ) = × = = 4
4 2 8 8 2 16 2
π  π 
293. Ans. (c), We know that, tan A ⋅ tan  − A  ⋅ tan  + A  = tan 3 A
3  3 
∴ tan 60 ⋅ tan 20 ⋅ tan 40° ⋅ tan 80° = 3  tan 20° ⋅ tan ( 60° − 20° ) ⋅ tan ( 60° + 20° ) 

= 3 tan ( 3 × 20° ) = 3 ⋅ 3 = 3
2 tan α 2 ( 3) 3
294. Ans. (c), tan 2α = = 2
=−
1 − tan α 1 − ( 3)
2
4
Now, dividing Nr. and Dr. of given expression by cos 2α , we get
2 tan 2α − 3 2 ( −3 / 4 ) − 3 −6 − 12 18 9
= = = =− =−
4 tan 2α + 5 4 ( −3 / 4 ) + 5 −12 + 20 8 4
π π π  π π   π
295. Ans. (b), Given Expression = cos 2 + cos 2  −  + cos 2  +  + cos 2  π − 
8 2 8 2 8  8
π π π π  π π
= cos 2 + sin 2 + sin 2 + cos 2 = 2  cos 2 + sin 2  = 2 (1) = 2
8 8 8 8  8 8
296. Ans. (b), The given expression is = ( cos80° + cos 40° ) + cos (180° − 20° ) + cos (180° + 60° )
1 1 1 1
= 2 cos 60° ⋅ cos 20° − cos 20° − cos 60° = 2 × cos 20° − cos 20° − = cos 20° − cos 20° − = −
2 2 2 2
297. Ans. (c), ∵ cos 40° = cos ( 90° − 50° ) = sin 50° and sin 40° = sin ( 90° − 50° ) = cos 50°
sin 70° + sin 50° 2 ⋅ sin 60° ⋅ cos10°
∴ Given expression = = = tan 60° = 3
cos 70° + cos 50° 2 ⋅ cos 60° ⋅ cos10°
sin α 5 sin α + sin β 5 + 3
298. Ans. (d), ∵ = ∴ By Componendo and Dividendo =
sin β 3 sin α − sin β 5 − 3
α + β  α − β  α + β 
2sin   ⋅ cos   8 tan  
∴  2   2 = ∴  2 =4
α + β  α − β  2 α − β 
2 cos   ⋅ sin   tan  
 2   2   2 
299. Ans. (b), cos172° = cos (180° − 8° ) = − cos 8°

Office.: 606 , 6th Floor, Hariom Tower, Circular Road, Ranchi-1, Ph.: 0651-2562523, 9835508812, 8507613968 47
48 ( ) BY R. K. MALIK’S NEWTON CLASSES
∴ Given Expression = cos 52° + ( cos 68° − cos8° ) = cos 52° − 2 ⋅ sin 38° ⋅ sin 30°
1
= cos 52° − 2 ⋅ sin ( 90° − 52° ) ⋅ = cos 52° − cos 52° = 0
2
300. Ans. (a), We have N = sin 4 α + sin 2 α ⋅ cos 2 α + cos 2 α
= sin 2 α ( sin 2 α + cos 2 α ) + cos 2 α = sin 2 α + cos 2 α = 1
Similarly, we can show that D = 1 ∴ Given Expression 1 /1 = 1
301. Ans. (c), The given expression is = ( sin α + cosec 2α + 2 ) + ( cos 2 α + sec 2 α + 2 ) − tan 2 α − cot 2 α
2

= 4 + ( sin 2 α + cos 2 α ) + ( sec2 α − tan 2 α ) + ( cosec 2 α − cot 2 α ) = 4 + (1) + (1) + (1) = 7


302. Ans. (b), sin163° = sin (180° − 17° ) = sin17° , cos 347° = cos ( 2 × 180° − 13° ) = cos13°
and sin 73° = sin ( 90° − 17° ) = cos17° , sin167° = sin (180° − 13° ) = sin13°
∴ Given Expression = sin17° ⋅ cos13° + cos17° ⋅ sin13° = sin (17° + 13° ) = sin 30° = 1 / 2
303. Ans. (d), Note that sin 99 a + sin 99 b + sin 99 c = 3 is possible iff sin a = sin b = sin c = 1, i.e.,
100 100 100
a = b = c = 90° ∵ cos 90° = 0 ∴ cos a + cos b + cos c = 0+0+0 = 0
304. Ans. (c), N = sin A − sin B = sin ( A + B ) ⋅ sin ( A − B ) , D = sin A ⋅ cos A − sin B ⋅ cos B
2 2

1 1 1
= ( 2sin A ⋅ cos A ) − ( 2sin B ⋅ cos B )  = [sin 2 A − sin 2 B ] =  2 ⋅ cos ( A + B ) ⋅ sin ( A − B ) 
2 2 2
N sin ( A + B ) ⋅ sin ( A − B )
= cos ( A + B ) ⋅ sin ( A − B ) ∴ = = tan ( A + B )
D cos ( A + B ) ⋅ sin ( A − B )
1
305. Ans. (c), Given, sin ( A + B + C ) = 1 ⇒ A + B + C = 90° and tan ( A − B ) =
3
⇒ A − B = 30° , also sec ( A + C ) = 2 ⇒ A + C = 60°
1
⇒ cos ( A + C ) = ∴ B = 30°, A = 60° and C = 0°
2
306. Ans. (c), Given, A = 90° ∴ B + C = 90° ⇒ C = 90° − B [∵ A + B + C = 180°]
Operating cos on both sides we get, cos C = cos ( 90° − B ) = sin B
⇒ cos 2 B + cos 2 C = cos 2 B + sin 2 B = 1
307. Ans. (c, d),
Each one of the sequences, x, y and z is an infinite G.P. with first term 1 and common ratio < 1.

1 1  a 2 
∴ x = ∑ cos 2 n θ = = ∵ S∞ = 1 − r and a = 1, r = cos θ 
n=0 1 − cos θ sin 2 θ
2


1 1  a 2 
y = ∑ sin 2 n θ = = ∵ S∞ = 1 − r and a = 1, r = sin θ 
n=0 1 − sin θ cos 2 θ
2


1  a 2 
z = ∑ cos 2 n θ sin 2 n θ = 2 2
2
∵ S∞ = 1 − r and a = 1, r = cos θ sin θ 
n =0 1 − cos θ sin θ
1 1
Note that sin 2 θ + cos 2 θ = 1 ⇒ + = 1 ⇒ x + y = xy …(i)
x y
1 1 1
Now to get relation in x, y, z put sin 2 θ = and cos 2 θ = in z =
x y 1 − sin θ ⋅ cos 2 θ
2

th
48 Office.: 606 , 6 Floor, Hariom Tower, Circular Road, Ranchi-1, Ph.: 0651-2562523, 9835508812, 8507613968
( ) BY R. K. MALIK’S NEWTON CLASSES 49
1
We get, x = ⇒ xyz = xy + z , Hence option (c) is true.
1 1
1− ⋅
x y
Using (i) we get, xyz = ( x + y ) + z , Hence option (d) is also correct.
308. Ans. (c), Given that diameter of circular wire = 10 cm
arc 10π π
∴Length of wire = 10π Hence, required angle = = = rad
radius 50 5
309. Ans. (c), we know that − a 2 + b 2 ≤ a sin + b cos θ ≤ a 2 + b 2
Maximum value of 3cos θ + 4sin θ = 32 + 42 = 52 = 5
310. Ans. (c), Given that, sin θ + cos θ = m …….. (i)
And sec θ + cosecθ = n …. (ii)
Now, n ( m + 1)( m − 1) = n ( m 2 − 1) = ( sec θ + cosec θ ) 2sin θ cos θ (∵ m 2
= 1 + 2 sin θ cos θ )
sin θ + cos θ
= ⋅ 2sin θ cos θ = 2m
sin θ cos θ
311. Ans. (b), We have, sin 420°⋅ cos 390° + cos ( −300° ) ⋅ sin ( −330° )
= sin 60°⋅ cos 30° − cos ( 360° − 60° ) ⋅ sin ( 360° − 30° )
= sin 60°⋅ cos 30° − cos 60°⋅ ( − sin 30° ) = sin 60° ⋅ cos 30° + cos 60° ⋅ sin 30°
= sin ( 60° + 30° ) = sin 90° = 1
312. Ans. (c), Now, cot ( 45° + θ ) cot ( 45° − θ ) = tan ( 90° − 45° − θ ) cot ( 45° − θ )
= tan ( 45° − θ ) cot ( 45° − θ ) = 1
313. Ans. (c), Now, sin A sin ( 60° − A ) cot ( 60° + A ) = sin A ( sin 2 60° − sin 2 A )
3
3  3sin A − 4 sin A sin 3 A
= sin A  − sin 2 A  = =
4  4 4
sin A cos A
314. Ans. (d), f ( A) = sin A ⋅ cos A ⋅ + cos A ⋅ sin A ⋅ = sin 2 A + cos 2 A = 1
cos A sin A
315. Ans. (c), Let f ( x ) = ( 3cos x + 4sin x ) + 5

We know that, − a 2 + b 2 ≤ a cos x + b sin x ≤ a 2 + b 2


⇒ − 32 + 42 ≤ 3cos x + 4sin x ≤ 32 + 42 ⇒ − 25 ≤ 3cos x + 4sin x ≤ 25
⇒ − 5 ≤ 3cos x + 4sin x ≤ 5 ⇒ −5 + 5 ≤ ( 3cos x + 4sin x + 5 ) ≤ 5 + 5 ⇒ 0 ≤ f ( x ) ≤ 10
∴ The maximum value of f ( x ) is 10
2
1 + cos x sin x + (1 + cos x ) 2 (1 + cos x )
2
sin x 1 + 1 + 2 cos x 2
316. Ans. (b), + = = = =
1 + cos x sin x sin x (1 + cos x ) sin (1 + cos x ) sin x (1 + cos x ) sin x
1
=2 = 2 cosec x
sin x
317. Ans. (a), Let f (θ ) = sin 3θ ⋅ cos 2θ + cos 3θ ⋅ sin 2θ
= sin ( 3θ + 2θ ) = sin 5θ ∵ sin ( A + B ) = sin A.cos B + cos A ⋅ sin B 

We know that −1 ≤ sin 5θ ≤ 1 ⇒ − 1 ≤ f (θ ) ≤ 1 So, the maximum value of f (θ ) is 1

Office.: 606 , 6th Floor, Hariom Tower, Circular Road, Ranchi-1, Ph.: 0651-2562523, 9835508812, 8507613968 49
50 ( BY R. K. MALIK’S NEWTON CLASSES
)
cos12° − sin12° sin147°
318. Ans. (c), +
cos12° + sin12° cos147°
1 − tan12°
= + tan147° = tan ( 45° − 12° ) + tan (180° − 33° ) = tan 33° − tan 33° = 0
1 + tan12°
1 99°× π 11π
319. Ans. (d), Let ∠A = rad, ∠B = 99° = =
2 180° 20
We know that, ∠A + ∠B + ∠C = π
1 11π 11π 1 9π − 10
⇒ + + ∠C = π ⇒ ∠C = π − − =
2 20 20 2 20
π  1 
320. Ans. (c), clearly sec2 θ + cos 2 θ ≥ 2, ∀ 0 < θ < ∵we know that x + ≥ 2 if x > 0 
2  x 
3 12
321. Ans. (a), ∵ tan A = and tan B = −
4 5
1 1 + tan A tan B
∴ cot ( A − B ) = =
tan ( A − B ) tan A − tan B
Which shows that cot ( A − B ) has only one value of A and B
3π 3π 3π 1 1
sin + cos − tan − +1
322. Ans. (b), 4 4 4 = 2 2 =1
3π 3π 3π − 2 + 2 + 1
sec + cosec − cot
4 4 4
323. Ans. (b), We know that, 1° < 1 rad ⇒ sin1° < sin1
cos15° sin15° cos 45° cos15°
324. Ans. (c), ×
cos 45° sin 45° sin 45° sin15°
= ( sin 45° cos15° − cos 45° sin15° ) × ( cos 45° sin15° − sin 45° cos15° )
1 1 1
= sin ( 45° − 15° ) × sin (15° − 45° ) = − sin 30°× sin 30° = − × = −
2 2 4
325. Ans. (c), ∵ 4 sin 2 x + 4 cos x − 1 = 0
⇒ 4 − 4 cos 2 x + 4 cos x − 1 = 0 ⇒ − 4 cos 2 x + 4 cos x + 3 = 0
⇒ 4 cos 2 x − 4 cos x − 3 = 0 ⇒ 4 cos 2 x − 6 cos x + 2 cos x − 3 = 0
3
⇒ ( 2 cos x − 3)( 2 cos x + 1) = 0 ⇒ cos x = (which is not possible)
2
1
And cos x = −
2
1
⇒ cos = − = cos 210° ( ∵ A lies in IIIrd quadrant )
2
⇒ A = 210°
1 1 1
326. Ans. (b), Given that, cos θ =  x +  ⇒ x + = 2 cos θ …(i)
2 x x
2
1  1 2
We know that, x + 2 =  x +  − 2 = ( 2 cos θ ) − 2 = 4 cos 2 θ − 2
2

x  x
= 2 cos 2θ [From eq. (i)]

th
50 Office.: 606 , 6 Floor, Hariom Tower, Circular Road, Ranchi-1, Ph.: 0651-2562523, 9835508812, 8507613968
(BY R. K. MALIK’S NEWTON CLASSES
) 51
1 2 1  1
∴  x + 2  = × 2 cos 2θ = cos 2θ
2 x  2
327. Ans. (a), Let f ( x ) = 3 cos x + sin x
 3 1   π
⇒ f ( x ) = 2  cos x + sin x  = 2sin  x + 
 2 2   3

 π π π π
Since, −1 ≤ sin  x +  ≤ 1 Hence, f ( x ) is maximum, if x + = ⇒ x = = 30°
 3 3 2 6
Or directly use − a 2 + b 2 ≤ a sin θ + b cos θ ≤ a 2 + b 2
Hence − 1 + 3 ≤ sin x + 3 cos x ≤ 1 + 3
328. Ans. (d), Given that, ABCD is a cyclic quadrilateral
So, A + C = 180° ⇒ A = 180° − C ⇒ cos A = cos (180° − C ) = − cos C
⇒ cos A + cos C = 0 … (i)
Similarly, cos B + cos D = 0 …. (ii)
On adding eqs. (i) and (ii), we get cos A + cos B + cos C + cos D = 0
1 − 3 tan 2 A 1 1  41π 
329. Ans. (b), Clearly = = ∵ A = 
3 tan A − tan A tan 3 A tan 41π
3
 12 
4
1 π
= = tan =1
 π 4
tan  10π + 
 4
π
330. Ans. (a), Arc length of circle rθ = 5 ×15°× ( ∵ θ = 15° )
180°

= cm
12
331. Ans. (d), 1 − sin10° sin 50° sin 70°
1 1 1 7
= 1 − ⋅ 4 sin10° sin ( 60° − 10° ) sin ( 60° + 10° ) = 1 − sin 30° = 1 − =
4 4 8 8
5 99
332. Ans. (a), ∵ sin θ = and sin φ =
13 101
∴ cos {π − (θ + φ )} = − cos (θ + φ ) = − {cos θ cos φ − sin θ sin φ}
 25  99 
2
5 99  12 20 5 99   240 495  255
= −  1− 1−   − ×  = − × − ×  = − − =
 169  101  13 101  13 101 13 101  1313 1313  1313
333. Ans. (a), ∵ x = sin θ cos θ and y = sin θ + cos θ
2
∴ y 2 − 2 x = ( sin θ + cos θ ) − 2sin θ cos θ
= sin 2 θ + cos 2 θ + 2 sin θ cos θ − 2sin θ cos θ = sin 2 θ + cos 2 θ = 1
334. Ans. (d), sin 4 x − cos 4 x = p (given)
⇒ ( sin 2 x − cos 2 x )( sin 2 x + cos 2 x ) = p

⇒ sin 2 x − cos 2 x = p ⇒ − cos 2 x = p ⇒ cos 2 x = − p ∵ −1 ≤ cos 2 x ≤ 1 ⇒ cos 2 x ≤ 1

Office.: 606 , 6th Floor, Hariom Tower, Circular Road, Ranchi-1, Ph.: 0651-2562523, 9835508812, 8507613968 51
52 ( ) BY R. K. MALIK’S NEWTON CLASSES
∴ p ≤1
1 1
335. Ans. (d), Given that sin A = and sin B =
10 5
We know that sin ( A + B ) = sin A cos B + cos A sin B

1 1 1 1 1 4 9 1 1 5 1
= 1− + 1− = + ⇒ ( 2 + 3) = =
10 5 10 5 10 5 10 5 50 50 2
π π
⇒ sin ( A + B ) = sin ⇒ A+ B =
4 4
sin1° sin1°
336. Ans. (b), We know that, 1c = 57°17′ ∴ = < 1 [ ∵ sin1° < sin 57°17′]
sin1 sin ( 5727 ) °
c

337. Ans. (a), The given expression is,


  2π   4π 
sin θ + sin (θ + 120° ) + sin (θ + 240° ) = sin θ + sin  θ +  + sin  θ + 
  3   3 
π
= sin θ + 2sin (θ + π ) cos = sin θ + sin (π + θ ) = sin θ − sin θ = 0
3
338. Ans. (b), We know that 1c = 57°17′ (approximately)
⇒ 2c = 114°34′ Hence correct choice is (b)
339. Ans. (c), X + Y = sin ( A + B ) sin ( A − B ) + cos ( A + B ) cos ( A − B )
= cos {( A + B ) − ( A − B )} ⇒ X + Y = cos 2 B
(a) for 0° < B < 45°, obviously X + Y > 0 (b) for B = 45°, we see that X + Y = 0
(c) for 45° < B ≤ 90°, clearly X + Y < 0
∴ Only option (c) is incorrect for any A and B, the expression dosen’t give any rational value
for X + Y
π   180° 
340. Ans. (a), tan   = tan   = tan15°
 12   12 
1
( )
2
1− 3 −1
tan 45° − tan 30° 3 = 3 −1 × 3 −1 = 3 +1− 2 3
= tan ( 45° − 30° ) = = = = 2− 3
1 + tan 45° ⋅ tan 30° 1 + 1 3 +1 3 −1 3 −1 2
3
341. Ans. (a), sin 2π ≠ sin ( −2π ) ∵ sin ( −2π ) = − sin 2π
342. Ans. (a), Given that, s = a sec θ cos φ , y = b sec θ sin φ , z = c tan θ
x 2 y 2 z 2 a 2 sec2 θ cos 2 φ b 2 sec 2 θ sin 2 φ c 2 tan 2 θ
+ − = + −
a2 b2 c2 a2 b2 c2
= sec2 θ ( cos 2 φ + sin 2 φ ) − tan 2 θ = sec 2 θ − tan 2 θ = 1
π
343. Ans. (c), Since, A + B =
2
π  1
∴ cos A cos B = cos  − B  cos B = sin B cos B = sin 2 B
2  2
1 1
∴ This greatest and the least values are and − respectively
2 2

th
52 Office.: 606 , 6 Floor, Hariom Tower, Circular Road, Ranchi-1, Ph.: 0651-2562523, 9835508812, 8507613968
( BY R. K. MALIK’S NEWTON CLASSES
) 53
344. Ans. (d), Given p = tan α + tan β , q = cot α + cot β
 1 1  1 1   1 tan α tan β 
∴  − = −  = − 
 p q   tan α + tan β cot α + cot β   tan α + tan β tan α + tan β 
1 − tan α tan β 1
= = = cot (α + β )
tan α + tan β tan (α + β )
cosec (π + θ ) cot {( 9π / 2 ) − θ } cosec 2 ( 2π − θ )   9π  π  
345. Ans. (b),  ∵ cot  − θ  = cot  − θ  = tan θ 
cot ( 2π − θ ) ⋅ sec (π − θ ) sec {( 3π / 2 ) + θ } 
2
 2  2  

=
( −cosec θ )( tan θ )( cosecθ ) =
tan 2 θ ⋅ cosec2θ sin 2 θ cos 2 θ
= ⋅ =1
( − cot θ ) ( sec2 θ ) ( cosecθ ) sec 2 θ cos 2 θ sin 2 θ
346. Ans. (c), sin 20° sin 40° sin 80°
1
= sin 20° sin 60° ( 2sin 40° sin 80° )
2
1 1 3  1
= sin 20° sin 60° ( cos 40° − cos120° ) = ⋅ sin 20°  1 − 2 sin 2 20° + 
2 2 2  2
3 3 3 3 3
=
8
( 3sin 20° − 4sin 3 20° ) =
8
sin 60° = ⋅ =
8 2 16
1
Or diretly use the paraula sin θ sin ( 60 − θ ) ⋅ sin ( 60 + θ ) = sin 3θ
4
  3π    π  
347. Ans. (b), We have, 3 sin 4  − α  + sin 4 ( 3π + α )  −2 sin 6  + α  + sin 6 ( 5π − α ) 
  2    2  
4 4
= 3 ( − cos α ) + ( − sin α )  − 2  cos 6 α + sin 6 α 
 
−2 ( cos 2 α + sin 2 α ) − 3cos 2 α sin 2 α ( cos 2 α + sin 2 α ) 
3

 
= 3 − 6sin 2 α cos 2 α − 2 + 6sin 2 α cos 2 α = 3 − 2 = 1
348. Ans. (a), tan15° ⋅ tan195° = tan15°⋅ tan (180° + 15° ) = tan15° ⋅ tan15°
= tan 2 (15° ) …. (i)
2 tan15° 1  2 tan θ 
Clearly tan ( 2 (15° ) ) = tan 30° = 2
=  ∵ tan 2θ = 
1 − tan 15° 3  1 − tan 2 θ 
⇒ 1 − tan 2 15° = 2 3 tan15° ⇒ tan 2 15° + 2 3 tan15° − 1 = 0
−2 3 ± 12 + 4 −2 3 ± 4
⇒ tan15° =
2 (1)
=
2
(
= − 3 ± 2 ⇒ tan15° = − 3 + 2 rejecting − 3 − 2 )
( ) ( )
2
From Eq. (i), we get tan15° ⋅ tan195° = 2 − 3 = 4+3− 4 3 = 7 − 4 3

349. Ans. (d), Let x = 2 + 2 + 2 + .....∞ ∴ x = 2+ x


On squaring both sides, we get x 2 = 2 + x
⇒ x 2 − x − 2 = 0 ⇒ ( x + 1)( x + 2 ) = 0 ⇒ x = 2, x = −1 ∴ x = 2 is correct ( ∵ x is positive )
1
∴ 2 = cosecθ ⇒ sin θ =
2
350. Ans. (a), The squares of the tangents of the angles 30°, 45° and 60° are in GP

Office.: 606 , 6th Floor, Hariom Tower, Circular Road, Ranchi-1, Ph.: 0651-2562523, 9835508812, 8507613968 53
54 ( ) BY R. K. MALIK’S NEWTON CLASSES
2
 1  1
( 3)
2
2 2 2
⇒ tan 30°, tan 45°, tan 60° are GP ⇒   , 1, are in GP = , 1, 3 are in GP
2
 3
( ∵ Condition for GP, b 2 = ac is satisfied )
1
i.e., 12 = × 3 ⇒ 1 = 1
2
−1 −1
351. Ans. (d), I. ( sec θ + tan θ ) = ( sec1200° + tan1200° )
−1 −1
= sec ( 6π + 120° ) + tan ( 6π + 120° )  = ( sec120° + tan120° )
1
( )
−1 −1
= ( −cosec 30° − cot 30° ) = −2 − 3 =− , negative
2+ 3
II. cosecθ − cot θ = cosec ( 6π + 120° ) − cot ( 6π + 120° )
2
= cosec120° − cot120° = sec 30° + tan 30° = + 3, positive
3
Hence, both the statements are incorrect,
tan 45° − tan 30°
352. Ans. (b), A. tan15° = tan ( 45° − 30° ) =
1 + tan 45° ⋅ tan 30°
1
1−
= 3 = 3 −1 × 3 −1 = 3 + 1 − 2 3 = 2 − 3
1 3 +1 3 −1 2
1+
3
1
1+
tan 45° + tan 30° 3 = 3 +1 × 3 +1 = 2 + 3
B. tan 75° = tan ( 45° + 30° ) = =
1 − tan 45° ⋅ tan 30° 1 − 1 3 −1 3 +1
3
tan 60° + tan 45° 3 +1 1+ 3 4+2 3
C. tan (105° ) = tan ( 60° + 45° ) = = × = = −2 − 3
1 − tan 60° ⋅ tan 45° 1 − 3 1 + 3 −2
353. Ans. (b), Given, θ = 18°
 5 −1 
Now, we have 4sin 2 θ + 2sin θ = 4sin 2 (18° ) + 2sin (18° )  ∵ sin18° = 
 4 

 5 − 1 
= 4
2
 5 − 1  5 +1− 2 5 ( ) ( 5 −1 ) = 3− 5 5 −1
 + 2  = 4⋅ + + =1
 4   4  16 2 2 2

π 3 −1
354. Ans. (a), I. sin = sin15° =
12 2 2
3 1 3 −1
∵ sin15° = sin ( 45° − 30° ) = sin 45° ⋅ cos 30° − cos 45° ⋅ sin 30° = − =
2 2 2 2 2 2
π 3 +1
II cos = cos15° =
12 2 2
3 1 3 +1
∵ cos15° = cos ( 45° − 30° ) = cos 45° ⋅ cos 30° + sin 45° ⋅ sin 30° = + =
2 2 2 2 2 2
π
III. cot = cot15° = 2 + 3
12
th
54 Office.: 606 , 6 Floor, Hariom Tower, Circular Road, Ranchi-1, Ph.: 0651-2562523, 9835508812, 8507613968
( BY R. K. MALIK’S NEWTON CLASSES
) 55
1
1+
∵ cos15° = tan 75° = tan ( 45° + 30° ) =
tan 45° + tan 30°
= 3 = 3 +1 = 2 + 3
1 − tan 45° ⋅ tan 30° 1 − 1 3 −1
3
∴ The correct sequence is III > II > I
π 3.14
355. Ans. (a), Ist, 1° = radian = = 0.017 radian
180 180
IInd , we know that I radian 57°17′ Which is greater than 45°
4 16 3
356. Ans. (a), sin A = ⇒ cos A = − 1 − =−
5 25 5
12 144 5
cos B = − ⇒ sin B = + 1 − =
13 169 13
4  12  3 5 −48 − 15 63
∴ sin ( A + B ) = sin A cos B + cos A sin B = ×  −  − × = =−
5  13  5 13 65 65
π 3π 5π
357. Ans. (b,d) , We have, < 2c < and 2π < 7 c <
2 4 2
⇒ 2c is in second quadrant and 7c is in first quadrant

⇒ a = cos 2c < 0 and b = sin 7 c > 0 ⇒ ab < 0


2

358. Ans. (c) We have,


1 − sin θ
=
(1 − sin θ ) ⇒
1 − sin θ 1 − sin θ
=
2
1 + sin θ 1 − sin θ 1 + sin θ cos θ

1 − sin θ 1 − sin θ  π 3π 
Hence
1 + sin θ
=
− cos θ ∵ 2 < θ < 2 ⇒ cos θ < 0 

1 − sin θ
⇒ = − sec θ + tan θ
1 + sin θ
2
 θ θ θ θ
 cos + sin  cos + sin
y +1 1 + sin θ y +1 2 2 1 +
=   ⇒ y 2 2
359. Ans. (b) , We have, = , ⇒ =
1− y 1 − sin θ 1− y  θ θ
2
1 − y θ θ
 cos − sin  cos − sin
 2 2 2 2

θ θ  π π π
1+ y
cos + sin ∵ 0 < θ < 2 ⇒ 0 < 2 < 4 
⇒ = 2 2  
1 − y cos θ − sin θ ⇒ cos θ > sin θ 
2 2  2 2 

θ
1 + tan
1+ y 2 θ
⇒ = ⇒ y = tan
1 − y 1 − tan θ 2
2
360. Ans. (d) Clearly, sec α − tan α is not defined for α = ± π / 2.
2

Now,
1 − sin α
=
(1 − sin α ) ⇒
1 − sin α 1 − sin α
= = sec α − tan α , if cos α > 0
2
1 + sin α 1 + sin α 1 + sin α cos α

Office.: 606 , 6th Floor, Hariom Tower, Circular Road, Ranchi-1, Ph.: 0651-2562523, 9835508812, 8507613968 55
56 ( ) BY R. K. MALIK’S NEWTON CLASSES
Clearly, cos α > 0 ⇒ α ∈ ( −π / 2, π / 2 )

1
361. Ans. (a)We have, a − b = sin x cos x ( cos 2 x − sin 2 x ) = sin 4 x
4
π
⇒ a − b > 0 for 0 < x <
4
1 π
Also, a + b = sin x cos x = sin 2 x ⇒ a + b > 0 for 0 < 2x < π i.e. 0 < x <
2 2
sin 3 A − cos3 A sin A
362. Ans. (b) We have, + − 2 tan A cot A.
sin A − cos A 1 + tan 2 A
sin A
= ( sin 2 A + cos 2 A + sin A cos A ) + − 2 = 1 + sin A cos A − sin A cos A − 2 = −1.
sec A

363. Ans. (b) We have, tan θ < 0


⇒ θ lies either in II quadrant of IV quadrant ⇒ sin θ > 0 or, sin θ < 0
4 4 4 4
∴ tan θ = − ⇒ sin θ = ± ⇒ sin θ = or, sin θ = −
5 5 5 5
sin ( x + y ) a+b
364. Ans. (b) We have, =
sin ( x − y ) a −b

sin ( x + y ) + sin ( x − y ) (a + b) + (a − b) 2 sin x cos x 2a tan x a


⇒ = ⇒ = ⇒ =
sin ( x + y ) − sin ( x − y ) ( a + b ) − ( a − b ) 2 cos x sin y 2b tan y b

x
365. Ans. (d), Given tan = cos ec x − sin x
2
x x
1 + tan 2
2 tan
x 2− 2 ⇒ 2t (1 + t ) = (1 − t )2 , where t = tan 2 x ⇒ t 2 + 4t − 1 = 0
⇒ tan =
2 2 tan
x
1 + tan 2
x 2
2 2
 2 x 
⇒ t = −2 + 5 ∵ t = tan 2 > 0 

A 5A
366. Ans. (c) We have, 32sin sin
2 2
 9 27 3
= 16 ( cos 2 A − cos 3 A ) = 16 ( 2 cos 2 A − 1 − 4 cos3 A + 3cos A ) = 16  2 × − 1 − 4 × + 3 ×  = 11
 12 64 4
367. Ans. (a) We have, sin 2θ = cos 3θ
⇒ 2 sin θ cos θ = 4 cos3 θ − 3cos θ ⇒ 2 sin θ = 4 (1 − sin 2 θ ) − 3

5 −1 − 5 −1
⇒ 4 sin 2 θ + 2sin θ − 1 = 0 ⇒ sin θ = Rejecting ∵ θ is acute
4 4
2π 4π 8 14π
368. Ans. (d) We have, cos cos cos cos
15 15 15 15

th
56 Office.: 606 , 6 Floor, Hariom Tower, Circular Road, Ranchi-1, Ph.: 0651-2562523, 9835508812, 8507613968
( ) BY R. K. MALIK’S NEWTON CLASSES 57

 π  16π 16π
sin  24 ⋅  sin sin
π 2π 4π 8π  15 =− 15 = − 15 = 1
= − cos cos cos cos =−
15 15 15 15 π π π 16
24 sin 16sin 16 sin
15 15 15
π 2π 3π 4π 5π 6π 7π
369. Ans. (b) We have, cos cos cos cos cos cos cos
15 15 15 15 15 15 15
 π 2π 4π 7π   3π 6π  5π
=  cos cos cos cos  ×  cos cos  × cos
 15 15 15 15   15 15  15

 π 2π 4π  8π    3π 6π  π
= cos cos cos cos  π −    cos cos  × cos
 15 15 15  15    8 15  3

 π 2π 4π 8π   3π 6π  1
=  − cos cos cos cos  ×  cos cos ×
 15 15 15 15   15 15  2

 π   3π  16π sin  12π 


sin  24 ×  sin  2 2 ×  sin  
 15  15  1  15  × 1 = 1 × 1 × 1 = 1
=− ×  × =− 15 ×
π 3π 2 π 3π 2 16 4 2 27
24 sin 22 sin 16sin 4sin
15 15 15 15
S1 − S3 + S5 − S7 + ....
370. Ans. (a) We know that tan (θ1 + θ 2 + ..... + θ n ) =
1 − S 2 + S 4 − S6 + ....
5
C1 tan θ −5 C3 tan 3 θ + 5 C5 tan 5 θ
∴ tan 5θ =
1 −5 C2 tan 2 θ + 5 C4 tanθ

π α 
371. Ans. (c) We have, 4sin 4 α + sin 2 2α + 4 cos 2  − 
4 2
 π 
1 + cos  2 − α  
= 4sin 2 α ( sin 2 α + cos 2 α ) + 4    = 2 sin α + 2 1 + sin α
 ( )
 2 
 

  3π 
= −2sin α + 2 (1 + sin α ) = 2. ∵ sin α < 0 for α ∈  π , 2 
  
2
α α x −1  x −1 
2sin cos 2 1−  
sin α 2 2 = 2x  2x 
372. Ans. (c) We have, tan α = = = x2 −1
cos α 2 α  x −1 
1 − 2 sin 1− 2 
2  2x 
373. Ans. (c) We have, sin A + cos A = m and sin 3 A + cos3 A = n.
3
Now, sin A + cos A = m ⇒ ( sin A + cos A ) = m3 ⇒ sin 3 A + cos3 A + 3sin A cos A ( sin A + cos A ) = m3

⇒ n + 3sin A cos Am = m3 …(i)

Again, sin A + cos A = m ⇒ sin 2 A + cos 2 A + 2 sin A cos A = m 2

m2 − 1
⇒ sin A cos A = …(ii)
2

Office.: 606 , 6th Floor, Hariom Tower, Circular Road, Ranchi-1, Ph.: 0651-2562523, 9835508812, 8507613968 57
58 ( ) BY R. K. MALIK’S NEWTON CLASSES

from (i) and (ii), we have n + 3m


(m 2
− 1)
= m3 ⇒ 2n + 3m3 − 3m = 2m3 ⇒ m3 − 3m + 2n = 0
2
374. Ans. (d) cos 2 A + cos 2 B + cos 2C
 3π 
= 2 cos ( A + B ) cos ( A − B ) + cos 2C = 2 cos  − C  cos ( A − B ) + cos 2C
 2 
= −2sin C cos ( A − B ) + 1 − 2sin 2 C = 1 − 2sin C {cos ( A − B ) + sin C }

{ }
= 1 − 2sin C cos ( A − B ) + sin ( 3π / 2 − ( A + B ) ) = 1 − 2sin C {cos ( A − B ) − cos ( A + B )}

= 1 − 4sin A sin B sin C.


 π  π  π π
375. Ans. (a) We have, y = sin  x +  + cos  x +  = 2 cos  x + − 
 6  6  6 4

 x π π
⇒ y = 2 cos  x −  ⇒ y is maximum for x − = 0 i.e. x =
 12  12 12
2

376. Ans. (d) We have,


1 + cos θ
=
(1 + cos θ ) ⇒
1 + cos θ 1 + cos θ
=
2
1 − cos θ 1 − cos θ 1 − cos θ sin θ

1 + cos θ 1 + cos θ
⇒ = [∵ π < θ < 2π ⇒ sin θ < 0]
1 − cos θ − sin θ

1 + cos θ
⇒ = − cos ec θ − cot θ
1 − cos θ

1 − sin θ 1 + sin θ 1 − sin θ + 1 + sin θ


377. Ans.(b)We have, + =
1 + sin θ 1 − sin θ 1 − sin 2 θ
2 2 ∵ π / 2 < θ < π 
= = = −2sec θ ∴ cos θ < 0 
cos θ − cos θ  
tan α + tan β
+ {cos (α − β ) sec (α + β ) + 1} = 1
−1
378. Ans. (a) We have,
cot α + cot β
sin (α + β ) sin α sin β cos (α + β )
⇒ × + =1
cos α cos β sin (α + β ) cos (α − β ) + cos (α + β )

cos α cos β − sin α sin β 1 1


⇒ tan α tan β + = 1 ⇒ tan α tan β + = 1 ⇒ tan α tan β = 1
2 cos α cos β 2 2
379. Ans. (a) We have,
tan 70° − tan 20° sin ( 70° − 20° ) cos 50° 2 cos 50° 2 cos 50°
= × = = =2
tan 50° cos 70° cos 20° sin 50° 2 cos 70° cos 20° cos 90° + cos 50°
380. Ans. (a) We have, −5 ≤ 3sin θ − 4 cos θ ≤ 5 for all θ
1 1 1
⇒ 2 ≤ 3sin θ − 4 cos θ + 7 ≤ 12 for all θ ⇒ ≤ ≤ for all θ
12 3sin θ − 4 cos θ + 7 2

th
58 Office.: 606 , 6 Floor, Hariom Tower, Circular Road, Ranchi-1, Ph.: 0651-2562523, 9835508812, 8507613968
( BY R. K. MALIK’S NEWTON CLASSES
) 59

 cos x 1 − sin x 
381. Ans. (c) We have, cos x  + 
 1 − sin x cos x 

 cos 2 x + (1 − sin x )2  2 − 2 sin x


= cos x   = = 2 for all x ∈ R Hence, required value = 2.
 (1 − sin x ) cos x  1 − sin x

b
382. Ans. (b) We have, tan x =
a

b b
1+ 1−
a+b a −b a + a 1 + tan x 1 − tan x cos x + sin x cos x − sin x
∴ + = = + = +
a −b a+b 1−
b
1+
b 1 − tan x 1 + tan x cos x − sin x cos x + sin x
a a
cos x + sin x + cos x − sin x cos x + sin x + cos x − sin x 2 cos x
= = =
2
cos x − sin x 2 2
cos x − sin x 2
cos 2 x
1 − sin 2 x
383. Ans. (c) We have, sec 2 x − tan 2 x =
cos 2 x
π 
1 − cos  − 2 x 
 2  2sin 2 (π / 4 − x ) π 
⇒ sec 2 x − tan 2 x = ⇒ sec 2 x − tan 2 x = = tan  − x 
π  2sin ( π / 4 − x ) cos (π / 4 − x ) 4 
sin  − 2 x 
2 
cos θ sin θ b
384. Ans. (a) , We have, = ⇒ tan θ =
a b a


a
+
b
= a cos 2θ + b sin 2θ = a
1 − tan 2 θ
+b
2 tan θ (
=a 2
a 2 − b 2 ) 2ab 2
+ 2 =a
sec 2b cos ec 2θ 1 + tan 2 θ 1 + tan 2 θ a + b2 a + b2

385. Ans. (a) sin 2 α + sin 2 β − sin 2 γ

= sin 2 α + sin ( β − γ ) sin ( β + γ ) = sin 2 α + sin (π − α ) sin ( β + γ ) [∵ α + β − γ = π ]


= sin α {sin α + sin ( β + γ )} = sin α {sin ( β − γ ) + sin ( β + γ )} ∵ α = π − ( β − γ ) 

= 2 sin α sin β cos γ

 απ   βπ 
386. Ans. (d) We have, tan   = cot  
 4   4 
 απ   π βπ  π π π
⇒ tan   = tan  −  ⇒ α = nπ +  − β  ⇒ α = 2 ( 2n + 1) − β ⇒ α + β = 2 ( 2n + 1)
 4  2 4  4 2 4
cos A sin A
387. Ans. (a) We haves, = n and =m
cos B sin B
sin ( A + B ) sin ( A − B ) sin 2 A − sin 2 B
∴ m 2 − n 2 = ( m + n )( m − n ) = ⇒ m2 − n2 =
cos 2 B sin 2 B cos 2 B sin 2 B
sin 2 A − sin 2 B cos 2 B − cos 2 A cos 2 A
⇒ ( m 2 − n 2 ) sin 2 B = 2
= 2
⇒ ( m 2
− n 2
) sin 2
B = 1 − 2
= 1 − n2
cos B cos B cos B

Office.: 606 , 6th Floor, Hariom Tower, Circular Road, Ranchi-1, Ph.: 0651-2562523, 9835508812, 8507613968 59
60 ( BY R. K. MALIK’S NEWTON CLASSES
)
1 cos (θ − φ )
388. Ans.(c) We have, cos (θ + φ ) = m cos (θ − φ ) ⇒ =
m cos (θ + φ )

1 + m 2 cos θ cos φ 1− m 1− m
⇒ = ⇒ tan θ tan φ = ⇒ tan θ = cot φ
1 − m 2 sin θ sin φ 1+ m 1+ m

389. Ans. (a), As cos 2 x + sin 2 x = 1 Hence a 2 cos 2 y + b 2 sin 2 y = 1


∴ a 2 (1 − sin 2 y ) + b 2 sin 2 y = 1 ∴ a 2 − ( a 2 − b 2 ) sin 2 y = 1 ∴ ( a 2 − b 2 ) sin 2 y = a 2 − 1

390. Ans. (d), 2 + 2 (1 + cos 4θ ) = 2 + 2 cos 2θ

π 3π
= 2 (1 − cos 2θ ) = 2 ⋅ 2sin 2 θ = 2 sin θ = 2sin θ as <θ <
2 4
Hence, (d) is the correct answer.
3cos θ + cos 3θ 4 cos3 θ
391. Ans. (c), = 3
= cot 3 θ
2sin θ − sin 3θ 4sin θ
3π 4π
392. Ans. (d), cos 2 + cos 2 = cos 2 108° + cos 2 144°
5 5
2 2
2 2
 5 −1   5 +1  2 ( 5 + 1) 3
= sin 18° + cos 36° =   +   = =
 4   4  16 4

α + β 
tan  
sin α 5 sin α + sin β 8  2 =4
393. Ans. (d) 3sin α = 5sin β ⇒ = ⇒ = ⇒
sin β 3 sin α − sin β 2 α − β 
tan  
 2 
π 1 π
394. Ans. (b),Given sin ( A + B ) = 1 ⇒ A + B = and sin ( A − B ) = ⇒ A− B =
2 2 6
π π
Hence we have A = and B =
3 6
395. Ans. (c), Now, tan ( A + 2 B ) ⋅ tan ( 2 A + B )
π π   2π π   2π   5π 
= tan  +  ⋅ tan  +  = tan   ⋅ tan  
3 3  3 6  3   6 
π π  π π   π  π  1 
= tan  +  ⋅ tan  +  =  − cot   − cot  = − 3 ⋅  −
2 6 2 3  6 3 
 =1
3
( )
396. Ans. (b), Now, sin 2 A − sin 2 B = sin 2 (π / 3) − sin 2 (π / 6 )
2
 3   1 2 3 1 2 1
=   −   = − = =
 2  2 4 4 4 2

th
60 Office.: 606 , 6 Floor, Hariom Tower, Circular Road, Ranchi-1, Ph.: 0651-2562523, 9835508812, 8507613968

You might also like